SlideShare a Scribd company logo
1 of 220
Question 1: Robert Mugabe has ruled Zimbabwe since the _________
achieved independence in 1980.
A. national B. nation C. nationality D. nationally
- nation (n): đất nước
- national (adj): thuộc đất nước
- nationally (adv)
- nationality (n): quốc tịch
Sau “the” cần danh từ và nghĩa câu B hợp lý nhất
Question 2: It is time to begin, _________?
A. isn’t it B. are they C. do I D. is time
Cấu trúc câu hỏi đuôi, Subject + verb + (object) + (…) + , + trợ động
từ + subject/ đại từ tương ứng với subject + ?
Question 3: Otherwise, you give a(n) ________ impression that you
are too anxious for a settlement.
A. wrong B. false C. improper D. incorrect
- give a false impression (collo): để lại ấn tượng sai lầm
Question 4: Grinder, a 35-year military veteran, was named to the
post ______ Thursday, Jan. 13
A. on B. in C. of D. with
Question 5: Poteet will receive a $100 cash ________ and $200 for
his school to purchase books.
A. bounty B. award C. gift D. prize
- prize : giải thưởng, phần thưởng (ít trang trọng, dùng cho người có
hành động tốt)
- award: giải thưởng, phần thưởng ( dùng cho người nào đó làm
hành động lớn lao )
- gift: món quà
- bonus: phần thưởng thêm
Question 6: Kids without guidance getting into trouble __ there
was nothing else to do
A. because B. though C. because of D. despite
- because + clause: bởi vì
- because of + N: bởi vì
- though + clause: mặc dù
- despite + N: mặc dù
Question 7: Tonight, Hana wore a __________ dress at the party
A. gorgeous red Chinese B. gorgeous Chinese red
C. red Chinese gorgeous D. Chinese red gorgeous
Trật tự: opinion -> color -> origin
Question 8: Don't _________ when he is telling the story.
A. stop in B. stop off C. break in D. break into
- stop in: ở nhà
- stop off: ngừng chuyến đi/hành trình
- break in: chen lời
- break into: đột nhập
Question 9: When my father ________ TV, my friend suddenly came
to my house.
A. has watched B. watches C. watched D. was watching
Hành động đang xảy ra liên tiếp trong quá khứ thì có hành động khác
chen ngang
Question 10: When I see him, I ___________ him your regards.
A. will give B. give C. would give D. gave
Sau mệnh đề chỉ thời gian dùng hiện tại ( vế trước dùng tương lai)
Question 11: She said to the mouse, you must __________ me a
favor, and once more manage the house for a day alone.
A. make B. do C. put D. go
- do sb a favor: giúp đỡ ai đó việc gì
Question 12: The doctor said that she might never be able to
_________ children.
A. care B. tolerate C. bear D. feed
- bear (v): chịu đựng/sinh ra
- tolerate (v): chịu đựng
- care (v): quan tâm
- feed (v): cho ai ăn
=> về nghĩa thì sẽ chọn bear (sinh ra)
Question 13: This house ____________ in the 18th century.
A. is building B. were build C. was built D. is built
- Câu bị động trong quá khứ: S + was/were + V3/ed
Question 14: _______ her homework, she watched her favourite
movie
A. Having finished B. finish C. being finished D. she
finish
Nếu 2 hành động ở 2 mệnh đề xảy ra theo thứ tự trước sau, ta lược
bỏ chủ từ của mệnh đề có hành động xảy ra trước rồi sau đó chia
động từ trong mệnh đề đó ở dạng Having + V3/ Ved.
Question 15: The sooner JK take your medicine,_________ he will
feel
A. better B. the better C. the good D. well
So sánh tăng tiến: The + comparative + S + V + the + comparative + S
+ V
Question 16: Ann and Peter are doing homework together.
– Ann: “Could you lend me that book”
– Peter: “____________”
A. No, I’m busy B. That’s what I think
C. Why did you say that? D. Of course, here it is
Câu giao tiếp: lời nhờ đưa đồ và cách đáp lại (theo nghĩa)
Question 17: Linda is thanking Daniel for his birthday present.
Linda: “Thanks for the book. I’ve been looking for it for months.”
Daniel: “____________”
A. You can say that again. B. Thank you for looking at it
C. I like reading books. D. I’m glad you like it.
Câu giao tiếp: cảm ơn quà và đáp lại (theo nghĩa)
Question 18: A. permission B. suggestion
C. refusal D. possible
A,B,C: âm 2
D: âm 1
Question 19: A. forest B. succeed C. homeless
C. nation
B: âm 2
A,C,D: âm 1
Question 20:
A. stopped B. wanted C. decided D. visited
Question 21:
A. head B. please C. heavy C. measure
Question 22: They tell us she isn't allowed to leave unless she gets
the doctor's permission
A. disapproval B. objection C. agreement D.
refusal
- Permission = agreement (n): sự cho phép
- Disapproval (n): sự không chấp thuận
- Objection = refusal (n): sự từ chối- abide = obey (v): nghe lời
Question 23: Public schools must abide by these rules, so why
shouldn't charter schools, too?
A. obey B. tolerate C. dislike D. allow
- abide = obey (v): nghe lời
- tolerate (v): chịu đựng
- allow (v): cho phép
Question 24: I would love to go back to college, but unfortunately,
that ship has sailed.
A. it was late B. it was timely
C. it was inconvenient D. it was traditional
That ship has sailed: đã quá trễ rồi
Question 25: A comfortable working environment will increase
productivity.
A. promote B. raise C. decrease D. go up
Increase = go up = raise = promote (v): tăng >< decrease (v): giảm
Question 26: It was not until they had reached Dak Lak that they realized how
far they had gone.
A. Not until they had reached Dak Lak did they realize how far they had gone.
B. Not until had they reached Dak Lak they realized how far they had gone.
C. Not until they reached Dak Lak had they realized how far they had gone.
D. Not until they realized how far they had gone had they reached Dak Lak
Đảo ngữ Not until + clause ………… trợ động từ + S + V
- Dịch: vừa mới…….thì…….
Question 27: They had a lot of homework last Monday.
A. They wish they had not a lot of homework last Monday
B. They wish they didn’t have a lot of homework last Monday
C. They wish they had had little homework last Monday
D. They wish they wouldn’t have a lot of homework last Monday
Điều ước không có thật ở quá khứ: S + wish + S + had + V3/ed
Question 28: Let's start by reviewing what we do with those
friends last week.
A. start B. reviewing C. do D. friend
“do” sai: thì quá khứ, có trạng ngữ chỉ thời gian “last week” cuối câu
Sửa: did
Question 29: Lily and her mother went to hospital last week to check
our health.
A. her B. went C. last D. our health
“our health” sai: tính từ sở hữu không đúng theo nghĩa
Sửa: their health
Question 30: I must complement you on your handling of a very
difficult situation.
A. must B. complement C. handling D. very
Complement: bổ ngữ => sai vì nhầm lẫn về nghĩa
Sửa: Compliment (khen)
Question 31: She advised him to go to the dentist immediately.
A. He would go to the dentist immediately
B. She should go to the dentist immediately
C. He should go to the dentist immediately
D. He must go to the dentist immediately
Lời khuyên => should
Question 32: She said, “ Don’t laugh, Jessica. Be serious.”
A. She said not to laugh and be serious.
B. She said Jessica not to laugh and be serious.
C. She told Jessica not to laugh and asked him to be serious.
D. She told Jessica do not laugh and be serious
Câu yêu cầu trong tường thuật: tell sb (not) to do something
Question 33: The last time I played the piano was 10 years ago.
A. I have not played the piano for 10 years ago
B. I have not played the piano for 10 years
C. I have not played the piano since 10 years
D. I did not play the piano since 10 years
Quá khứ đơn chuyển thành hiện tại hoàn thành
Bỏ “ago” sau thời gian, và đây là khoảng thời gian nên dùng “for”
Question 34:
Plants and animals will find it difficult to escape from or adjust to the
effect of global warming. Scientists have already observed shifts in
the life cycles of (34) ________plants and animals
A. many B. each C. much C. little
Plants và animals là số nhiều đếm được => many
Question 35:
such as flowers (35)______________earlier and birds hatching
earlier in the spring.
A. increasing B. blooming C. growing D.
swelling
- bloom (v): nở (dùng cho hoa)
- grow (v): lớn lên
- swell (v): phồng lên
Question 36:
Many species have begun shifting (36)_____________ they live or their
annual migration patterns due to warmer temperatures.
A. that B. what C. which D. where
Mệnh đề chỉ nơi chốn => hợp nghĩa và ngữ cảnh
Question 37:
Species living in unique ecosystems, such as those found in polar and
mountaintop regions, are especially at risk (37)___________ migration
to new habitats is not possible.
A. because B. but C. and D. although
Question 38:
For example, polar bears and marine mammals in the Arctic are already
threatened by(38) ________________ sea ice but have nowhere farther
to go.
A. decreasing B. falling C. deepening D. dwindling
Dwindling sea ice: suy giảm băng biển
Question 39: What is the passage mainly about?
A. Brain Waves Tell the Story
B. People with fixed mindsets
C. The feedbacks are important
D. People with a growth mindset
Có thể dùng phương pháp loại trừ:
B: Sai vì có nói thêm people with a growth mindset
C: Sai vì trong bài không đề cập
D. Sai vì nói có nói thêm people with a fixed mindset
=> Chọn A
Question 40: According to paragraph 3, what is the result when
people with a fixed mindset were presented with information that
could help them learn?
A. They paid close attention to B. They were reluctant
C. They were angry D. They showed no interest
Dẫn chứng: “But when………..sign of interest” ở câu đầu đoạn 3
Question 41: Which of the following is NOT mentioned in paragraph 2 and 3 as for
people with a fixed mindset?
A. They were not interested in new knowledge
B. They only paid attention to the the feedbacks
C. Learning made them get a headache
D. They were inattentive to information that helps them learn
Dùng phương pháp dò từng câu có trong bài:
A: Có ở câu đầu đoạn 3 “ but when they were presented…… no sign of interest”
B: Có ở câu đầu đoạn 2 “ people with fixed mindset….ability”
C. Không đề cập (chọn)
D. như câu A
Question 42: The word growth in paragraph 4 is closest in meaning to _______.
A. decline B. development C. height D. reduction
Growth (n) = development : phát triển
Question 43: The word they in paragraph 1 refers to _______.
A. People with both mindsets B. Hard questions
C. Brainwaves D. feedback
Dẫn chứng: “they” trong đoạn 1 chỉ người ở số nhiều => đứng trước nó chỉ có
“people with both mindsets”.
Question 44: Which of the following is the best title for the passage?
A. The open-water swimming record of Diana Nyad
B. Every Sport is a Team Sport
C. Swimming is difficult
D. The strength of trainers to athletes
Dẫn chứng: Câu đầu đoạn 1 và xuyên suốt bài nói về vấn đề sự hợp
tác của nhiều người dù là môn thể thao cá nhân
Question 45: According to paragraph 1, what type of sports has a
team?
A. individual sports B. team sportsC. underwater sports D. both
A&B
Dẫn chứng: câu đầu đoạn 1 có cụm “every sport”, tức là chỉ tất cả các
loại môn thể thao
Question 46: The word arduous in paragraph 2 is closest in meaning
to _______.
A. heavy B. easy C. strenuous D. quick
Arduous (adj) = strenuous: cật lực, tốn nhiều công sức
Question 47: The word it in paragraph 1 refers to _______.
A. every sport
B. tennis
C. golf
D. a team sport
“it” trong đoạn văn đứng sau 2 danh từ số ít : “every sport” và “a
team sport”
=>Loại B,C
- Xét về nghĩa của câu chứa “it” => chọn every sport
Question 48: The word coaches in paragraph 2 is closest in meaning to
_______.
A. partners B. instructors C. pupils D. carriages
Coach (n) = instructor: người hướng dẫn, huấn luyện viên
Question 49: Which of the following is NOT true according to the
passage?
A. A team of guides measure the winds and the current, and watch for
obstacles
B. Divers teach her how to swim properly
C. NASA experts guide on nutrition and endurance
D. Trainers who talked her through uncontrollable shivers, nausea,
hallucinations, and despair
A. đúng ( … a team of guides ( for meassuring….), câu 6 từ dưới lên
Divers look for sharks => Câu B sai
C. NASA experts…. dòng 4 từ dưới lên
D. Trainers who talked…………….. (dòng 2 từ dưới lên)
Question 50: Which of the following can be referred from the
passage?
A. Diana Nyad lost so much weight, which led to hallucinations
B. A few months of training were not enough for her to be ready
C. Diana Nyad’s trainers contributed mostly to her achievements
D. Diana Nyad were supported by over 50 people
Dẫn chứng: câu cuối đoạn văn: “but it took fifty-one other people to
do it: 51 người khác làm nó.
Question 1. Children can be encouraged to work together as a
team by playing _______ sports.
A. compete B. competitively C. competitive D.
competition
Kiến thức: Từ loại
Giải thích:
- Ta thấy chỗ trống ở trước danh từ nên từ cần điền là một tính từ.
Xét các đáp án :
A. compete (v): thi/ thi đấu B. Competitively (adv): cạnh tranh
C. competitive (a): cạnh tranh D. competition (n): cuộc thi
Vậy đáp án là C
Tạm dịch: Trẻ em có thể được khuyến khích làm việc cùng
nhau như một đội bằng cách chơi các môn thể thao cạnh
tranh.
Question 2. Michael rarely returns to his hometown, _______?
A. doesn’t heB. hasn’t he C. does he D. has he
Kiến thức: Câu hỏi đuôi
Giải thích:
Ta có chú ý trong khi thành lập câu hỏi đuôi như sau: Nếu trong
câu dạng khẳng định có rarely, barely, hardly, never thì thành
lập câu hỏi đuôi như đối với câu phủ định.
Như vậy ở đây ta mượn trợ động từ là does. Chọn đáp án
đúng là C.
Tạm dịch. Michael hiếm khi về quê, có phải không?
Question 3. Pesticide residues in fruit and vegetable can be _______
to health.
A. crucial B. supportive C. receptive D. destructive
Kiến thức: Từ vựng
Giải thích:
Xét các đáp án:
A. crucial (adj): cực kỳ quan trọng B. supportive
(adj): khuyến khích, cổ vũ
C. receptive (adj): dễ tiếp thu D. destructive (adj):
phá hoại, gây hại
Phù hợp ngữ nghĩa nhất của câu là D.
Tạm dịch: Dư lượng thuốc trừ sâu trong trái cây và rau quả có
thể gây hại sức khoẻ.
Question 4. When you grow up, you must learn how to become
independent _______ your parents.
A. withB. in C. of D. on
Kiến thức: Giới từ
Giải thích:
Ta có cấu trúc: be/ get/ become independent of: tự lập/ không phụ
thuộc vào
Vậy đáp án đúng là C
Tạm dịch: Khi lớn lên, bạn phải học cách tự lập không phụ thuộc vào
cha mẹ.
Question 5. I don’t feel like buying a _______ in a poke; we’d
better check the content.
A. pig B. cattle C. buffalo D. ox
Kiến thức: Thành ngữ
Giải thích:
Ta có thành ngữ: buy a pig in a poke: mua vô tội vạ
Vậy đáp án đúng là A
Tạm dịch: Tôi không muốn mua vô tội vạ; tốt hơn chúng ta nên kiểm
tra nội dung.
Question 6. Dang Van Lam was absent from Vietnam’s World
Cup qualifiers in June _______ his contact with a Covid-19
case.
A. despite B. because C. although D. because of
Kiến thức: Liên từ
Giải thích:
Xét các đáp án:
A. despite (+ V-ing/ noun phrase): mặc dù B. because (+
clause): bởi vì
C. although (+ clause): mặc dù D. because of (+ V-ing/
noun phrase): bởi vì
Ta thấy sau chỗ trống là là một cụm danh từ nên loại đáp án B và C
Dựa vào nghĩa của câu, đáp án D là đúng
Tạm dịch: Đặng Văn Lâm đã vắng mặt ở vòng loại World Cup của Việt
Nam vào tháng Sáu vì nhiễm Covid-19.
Question 7. I remember she wore a _______ dress to go out with her
boyfriend last week.
A. cotton white Vietnamese B. Vietnamese white cotton
C. white Vietnamese cotton D. white cotton Vietnamese
Kiến thức: Trật tự tính từ
Giải thích:
Theo quy tắc trật tự tính từ trong câu: OSASCOMP
white – Color; Vietnamese – Origin; cotton – Material
Vậy đáp án đúng là C
Tạm dịch: Tôi nhớ cô ấy đã mặc một chiếc váy vải cotton Việt Nam
màu trắng để đi ra ngoài với bạn trai tuần trước.
Question 8. She _______ quite often as a guest on popular
television shows, as well as in television and bigscreen movies.
A. looked up B. looked after C. turned up D. turned on
Kiến thức: Cụm động từ
Giải thích:
Xét các đáp án:
A. look up: tra cứu (từ điển) B. look after: chăm sóc
C. turn up: đến/ xuất hiện D. turn on: bật (điện/ công tắc
điện)
Dựa vào nghĩa và ngữ cảnh, đáp án C là đúng
Tạm dịch: Cô xuất hiện khá thường xuyên với tư cách khách mời trên
các chương trình truyền hình nổi tiếng, cũng như trong các bộ phim
truyền hình và màn ảnh rộng.
Question 9. The students _______ about their summer vacation
when their teacher came in.
A. talked B. were talking C. have talked D. talk
Kiến thức: Thì quá khứ tiếp diễn
Giải thích:
Ta thấy mệnh đề trạng ngữ chỉ thời gian bắt đầu bằng WHEN và động
từ CAME ở quá khứ đơn nên động từ ở mệnh đề chính ở thì quá khứ
tiếp diễn (một hành động đang diễn ra thì bị một hành động khác
xen vào).
Công thức: S1 + was/ were + V1-ing + when + S2 + V2-simple past
Vậy đáp án đúng là: B (were talking)
Tạm dịch: Các học sinh đang nói về kỳ nghỉ hè của họ khi giáo
viên của họ đến
Question 10. _______, they will have done their homework.
A. By the time you finish cooking
B. By the time you finished cooking
C. When you finish cooking
D. While you finish cooking
Question 10. Đáp án A
Kiến thức: Sự phối hợp các thì (Tương lai & hiện tại)
Giải thích:
- Mệnh đề chính chia ở thì tương lai hoàn thành nên chọn đáp án A.
* Chú ý : By the time + S + V(htt), S + will have + Vp2
By the time + S + V(qkđ), S + had + Vp2
Tạm dịch : Đến lúc bạn hoàn thành việc nấu ăn, chúng sẽ đã hoàn
thành bài tập của chúng.
Question 11. Due to the nature of the earthquake, a much larger
_______ of the population might be affected.
A. density B. totality C. segment D. division
Kiến thức: Cụm từ cố định
Giải thích:
Ta có các cụm từ:
- density of the population = population density: mật độ dân số (danh
từ không đếm được)
- segment of the population = population segment: phân khúc dân số/
bộ phận dân số (danh từ đếm được)
Vậy đáp án đúng là C
Tạm dịch: Do tính chất của trận động đất, một bộ phận dân số lớn
hơn nhiều có thể bị ảnh hưởng.
Question 12. Volunteers may be required to obtain Red Cross
_______ in order to serve through hospitals and healthcare
organizations or provide disaster relief.
A. diploma B. certification C. license D. degree
Kiến thức: Từ vựng
Giải thích:
A. Diploma /dɪˈpləʊ.mə/ (n): chứng chỉ, bằng cấp (dành
cho các khóa học kéo dài từ 1-2 năm)
B. Certification /ˌsɜː.tɪ.fɪˈkeɪ.ʃən/ (n): giấy chứng nhận
C. Licence /ˈlaɪ.səns/ (n): giấy phép
D. Degree /dɪˈɡriː/ (n): bằng cấp (dành cho các khóa học
kéo dài từ 3-4 năm)
Dựa vào nghĩa và ngữ cảnh đáp án đúng là B
Tạm dịch: Các tình nguyện viên có thể được yêu cầu có giấy
chứng nhận Hội chữ thập đỏ để mà phục vụ ở bệnh viện và các
tổ chức chăm sóc sức khỏe hoặc là cung cấp sự cứu nạn.
Question 13. Last night Laura _______ a big diamond ring by
her husband-to-be at her birthday party.
A. was givingB. gave C. gives D. was given
Kiến thức: Câu bị động
Giải thích:
Ta thấy trạng ngữ chỉ thời gian LAST NIGHT nên động từ chia ở thì
quá khứ đơn nên loại đáp án A và C.
Ta có công thức : give Sb St : cho/ tặng ai cái gì
Hơn nữa, ta thấy trong câu có BY HER HUSBAND-TO-BE nên động từ
chia ở bị động – loại B
Vậy đáp án đúng là D
Tạm dịch : Đêm qua, Laura đã được chồng sắp cưới tặng một chiếc
nhẫn kim cương lớn trong bữa tiệc sinh nhật.
Question 14. ________ the homework, he was allowed to go out
with his friends.
A. Finishing B. Finish C. To finish
D. Having finished
Kiến thức : Rút gọn mệnh đề trạng ngữ
Giải thích:
Khi hai mệnh đề cùng chủ ngữ, và câu muốn nhấn mạnh hành
động phía trước được hoàn thành xong trước rồi hành động
phía sau mới xảy ra thì chúng ta dùng công thức: Having + P2,
S+Ved.
Vậy đáp án đúng D.
Tạm dịch: Sau khi hoàn thành xong bài tập về nhà, anh ấy được
phép đi chơi với những người bạn của mình.
Question 15. _______the time passes, _______I feel! The deadline of
my thesis is coming, but I have just finished half of it.
A. The faster / the nervous B. The more fast / the nervous
C. The fast / the more nervous D. The faster / the more nervous
Kiến thức: So sánh kép
Giải thích:
A. The faster / the nervous: sai công thức
B. The more fast / the nervous: sai công thức
C. The fast / the more nervous: sai công thức
D. The faster / the more nervous: đúng công thức so sánh kép
The + comparative + S1+ V1, the comparative + S2 + V2: càng ……, càng
……
Tạm dịch: Thời gian trôi qua càng nhanh, tôi càng cảm thấy hồi hộp.
Question 16. Tom is in Ho Chi Minh city and asks a passer-by the way to the railway
station.
- Tom: “Can you show me the way to the railway station, please?”
- Passer-by: “_______”
A. No way. B. Just round the comer over there.
C. Look it up in a dictionary! D. There’s no traffic near here.
Kiến thức: Tình huống giao tiếp (Đáp lại lời đề nghị)
Giải thích:
Tình huống giao tiếp:
Tom đang ở thành phố Hồ Chí Minh và hỏi người qua đường đường tới nhà
ga xe lửa.
- Tom: Làm ơn chỉ cho tôi đường đến nhà ga xe lửa.
- Người đi đường: _______.
Xét các đáp án:
A. Không đời nào B. Nó chỉ ở quanh góc kia thôi
C. Hãy tra cứu nó trong cuốn từ điển! D. Không có giao
thông ở gần đây
Xét về nghĩa và tình huống giao tiếp thì chọn được B
Bản word phát hành từ website Tailieuchuan.vn
Question 17. Marry is talking to Linda over the phone.
- Mary: “Thank you for helping me prepare for the party.”
- Linda: “_______”
A. My pleasure B. The meal was out of
this world
C. Never mention me D. Of course not
Kiến thức: Tình huống giao tiếp (Đáp lại lời cảm ơn)
Giải thích:
Tình huống giao tiếp:
Marry đang nói chuyện với Linda qua điện thoại.
- Mary: "Cảm ơn bạn đã giúp tôi chuẩn bị cho bữa tiệc."
- Linda: “_______”
Xét các đáp án:
A. Niềm vui của tôi B. Bữa ăn đã ra khỏi thế giới này
C. Không bao giờ đề cập đến tôi D. Tất nhiên là không
Xét về nghĩa và tình huống giao tiếp thì chọn được A
Question 18. A. uncertain B. arrogant C. familiar
D. impatient
Kiến thức: Trọng âm của từ 3 âm tiết
Giải thích:
- uncertain /ʌnˈsɜːtn/: trọng âm rơi vào âm tiết thứ hai vì theo quy tắc
tiền tố UN- không làm thay đổi trọng âm chính.
- arrogant /ˈærəɡənt/ : trọng âm rơi vào âm tiết thứ nhất vì theo quy
tắc trọng âm ưu tiên rơi vào âm chứa nguyên âm dài.
- familiar /fəˈmɪlɪə/: trọng âm rơi vào âm tiết thứ hai
- impatient /ɪmˈpeɪʃnt/: trọng âm rơi vào âm tiết thứ hai vì theo quy
tắc tiền tố IM- không làm thay đổi trọng âm chính.
Vậy đáp án đúng là B
Question 19. A. precede B. offer C. visit D. finish
Question 19. Đáp án: A
Kiến thức: Trọng âm của từ 2 âm tiết
Giải thích:
A. precede (v) /prɪˈsiːd/ : động từ nên trọng âm rơi vào
thứ 2
B. offer (v) /ˈofə/ : động từ 2 âm tiết nhưng trọng âm rơi
vào 1 vì có đuôi -ER
C. visit (v) /ˈvizit/ : động từ 2 âm tiết mà cả 2 âm tiết đều
là /I/ nên trọng âm rơi vào 1
D. finish (v) /ˈfiniʃ/ : động từ 2 âm tiết nhưng trọng âm
rơi vào 1 vì có đuôi -ISH
Vậy đáp án là A
Question 20. A. looked B. laughed C. opened D. stepped
Kiến thức : Cách phát âm đuôi -ED
Giải thích:
– Phát âm là /t/ khi từ có tận cùng bằng các phụ âm vô thanh:
/θ/, /p/, /k/, /f/, /s/, /ʃ/, /tʃ/
– Phát âm là /id/ khi từ có tận cùng là các âm: /t/, /d/
– Phát âm là /d/ khi các từ có tận cùng là nguyên âm và các
phụ âm hữu thanh còn lại
Xét các đáp án:
A. looked /lʊkt/ B. laughed /lɑː /
C. opened /ˈəʊ.pənd/ D. stepped /stept/
Vậy đáp án đúng là C
Question 21. A. tangle B. dangerous C. battle D. calculate
Question 21. Đáp án B
Kiến thức: Cách phát âm của nguyên âm
Giải thích:
A. tangle /ˈtæŋɡl/ B. dangerous /ˈdeɪndʒərəs/
C. battle /ˈbætl/ D. calculate /ˈkælkjuleɪt/
Vậy đáp án đúng là B
Question 22. I knew she was only flattering me because she wanted
to borrow some money.
A. teasing B. threatening C. praising D. helping
Kiến thức: Đồng nghĩa (từ đơn)
Giải thích:
Ta có: flatter (v): xu nịnh, tâng bốc, khen ngợi
Xét các đáp án:
- tease (v): trêu chọc - threaten (v): đe dọa
- praise (v): khen ngợi, tán dương - help (v): giúp đỡ
Vậy: flatter = praise
Dịch: Tôi biết cô ấy chỉ đang nịnh tôi vì cô ấy muốn mượn tiền.
Question 23. he student was ecstatic when he found out he
received the highest achievable score on the
test.
A. excited B. worried C. tragic D. disappointed
Kiến thức: Đồng nghĩa (từ đơn)
Giải thích:
Ta có: - ecstatic (a): ngất ngây/ vui mừng/ hào hứng
Xét các đáp án:
A. excited (a): hào hứng/ vui mừng B. worried (a): lo lắng
C. tragic (a) bi kịch/ thảm kịch D. disappointed (a):
thất vọng
Vậy đáp án đúng là A
ecstatic ~ excited
Tạm dịch: Sinh viên vui mừng khi biết mình nhận được số điểm cao
nhất có thể đạt được trong bài kiểm tra.
Question 24. If you want to be a successful gardener, of course you’ve got to have
green fingers.
A. lack skills in gardening B. have a good head for gardening
C. be better at doing the gardening D. make plants grow well
Kiến thức: Trái nghĩa (cụm từ hoặc thành ngữ)
Giải thích:
Ta có thành ngữ: have green fingers: mát tay/ có tài làm vườn/ có khiếu làm vườn
Xét các đáp án:
A. lack skills in gardening: thiếu kỹ năng làm vườn
B. have a good head for gardening: có tài/ khiếu làm vườn
C. be better at doing the gardening: giỏi làm vườn hơn
D. make plants grow well: làm cho cây cối phát triển tốt
Vậy đáp án đúng: A
have green fingers >< lack skills in gardening
Tạm dịch: Nếu bạn muốn trở thành một người làm vườn thành công, tất nhiên bạn
phải có khiếu làm vườn.
Question 25. 2008 is a memorable year for people around the world because of the
global financial recession that hit practically every sector of world economy.
A. prosperity B. downturn C. crisis D. depression
Kiến thức: Trái nghĩa (từ đơn)
Giải thích:
Ta có: Recession (n) sự khủng hoảng
Xét các đáp án:
A. Prosperity (n): sự thịnh vượng (về kinh tế)
B. crisis (n) cơn khủng hoảng
C. Downturn (n): sự suy sụp (trog hoạt động kinh tế)
D. depression (n) sự trì trệ
Vậy đáp án đúng là A
recession = crisis >< prosperity
Tạm dịch: 2008 là năm đáng nhớ với mọi người trê toàn thế giới vì sự khủng
hoảng kinh tế toàn cầu, cái mà đã đánh thẳng vào các ngành kinh tế thế
giới.
Question 26. Linda's final exam is important. She realizes little of it
A. Under no circumstances does Linda realize how important her
final exam is.
B. Rarely does Linda realize how important her final exam is.
C. Little does Linda realize how important her final exam is.
D. Never does Linda realize how important her final exam is.
Kiến thức: Kết hợp câu – đảo ngữ
Giải thích:
Cấu trúc:
Under no circumstances/rarely/little/never + mệnh đề đảo ngữ
Mệnh đề đảo ngữ: trợ động từ/tobe/ động từ khuyết thiếu/ have/has + S + V
Đề bài: Linda's final exam is important. She realizes little of it.
Bài kiểm tra cuối khóa của Linda rất quan trọng. Cô ấy gần như không hiểu
được điều đó.
= C. Little does Linda realize how important her final exam is.
Hầu như Linda không nhận ra được kỳ thi cuối cùng của cô quan trọng như
thế nào.
Question 27. I deeply regret having spoken to her so severely yesterday. She was badly
hurt.
A. If only I could apologize to her for having spoken to her so severely yesterday.
B. I wish I hadn’t spoken to her so severely yesterday.
C. She must have been badly hurt because I had spoken to her so severely yesterday.
D. If I hadn’t spoken to her so severely yesterday, she wouldn’t be badly hurt.
Kiến thức: Kết hợp câu – câu ước
Giải thích:
Câu đề bài: Tôi vô cùng hối hận vì đã nói rất nặng lời với cô ấy đêm qua. Cô ấy bị
tổn thương nặng nề.
Các phương án:
A. Giá mà tôi có thể xin lỗi cô ấy về việc đã nói rất nặng lời với cô ấy đêm
qua. (sai về cấu trúc - ở đây cần cấu trúc lời ước cho quá khứ nhưng đây lại là lời
ước ở hiện tại.)
B. Tôi ước tôi đã không nói rất nặng lời với cô ấy đêm qua. (đáp án này
đúng - cấu trúc lời ước cho quá khứ)
C. Cô ấy chắc chắn bị tổn thương nặng nề bởi vì tôi đã nói rất nặng lời với
cô ấy đêm qua. (sai về thì của động từ ở mệnh đề trạng ngữ chỉ lý do –
YESTERDAY – thì động từ chia ở quá khứ đơn)
D. Nếu tôi đã không nói rất nặng lời với cô ấy đêm qua, cô ấy không bị tổn
thương nặng nề. (Sai về cấu trúc – việc tổn thương này đã xảy ra nên ko thể là câu
điều kiện hỗn hợp)
Vậy đáp án đúng là B
Question 28. We are going to visit our grandparents when we will
finish our final exams.
A. are going to B. our C. when D. will finish
Kiến thức: Lỗi sai – Thì của động từ
Giải thích:
Động từ của mệnh đề chính chia ở thì tương lai gần, thì động
từ của mệnh đề trạng ngữ chỉ thời gian chia ở thì hiện tại nên
sai ở đáp án D.
Sửa: will finish → finish
Tạm dịch: Chúng tôi sẽ đi thăm ông bà khi chúng tôi hoàn
thành kì thi cuối cùng.
Question 29. Ozone has his origin in a number of sources, a
prime one being the automobile engine.
A. his B. prime C. being D. the
Kiến thức: Lỗi sai – Đại từ nhân xưng/ tính từ sở hữu
Giải thích:
- Tính từ sở hữu thay thế cho danh từ “Ozone” (không phải
danh từ chỉ người) là “ITS” nên sai ở đáp án A.
Sửa lỗi: his → its
Tạm dịch: Ozone có nguồn gốc từ một số nguồn, nguồn chính
là động cơ ô tô.
Question 30. It took the spouses five years to discover that
their tastes were diverging and their tempers were
incompatible.
A. spouses B. tastes C. diverging D. incompatible
Kiến thức: Lỗi sai – Từ vựng
Giải thích:
Ta có: diverging (V-ing): khác nhau/ bất đồng ý kiến
divergent (a): khác nhau/ bất đồng ý kiến
Dùng sau động từ TASTE + adj (linking verb + adj/n)
Vậy đáp án đúng là C
Sửa lỗi: diverging → divergent
Tạm dịch: Hai vợ chồng đã mất 5 năm để phát hiện ra rằng sở thích
của họ khác nhau và tính khí của họ
không tương thích.
Question 31. I'm sure Luisa was very disappointed when she failed
the exam.
A. Luisa must be very disappointed when she failed the exam.
B. Luisa must have been very disappointed when she failed
the exam.
C. Luisa may be very disappointed when she failed the exam.
D. Luisa could have been very disappointed when she failed
the exam.
Kiến thức: Câu đồng nghĩa – Động từ khuyết thiếu
Giải thích: Must be... : dùng để diễn tả những suy luận ở hiện tại
Must + have + Vp2: dùng để diễn tả những suy luận ở trong quá khứ
Could + have + Vp2: dùng để diễn tả những điều có thể đã xảy ra
nhưng trên thực tế là không
Đề bài: Tôi chắc chắn Luisa đã rất thất vọng khi cô ấy trượt kỳ thi.
= B. Luisa hẳn đã rất thất vọng khi cô thi trượt.
Question 32. “Would you like to come out to dinner with me tonight, Jenny?” Paul
said.
A. Paul suggested that Jenny go out to dinner with him that night.
B. Paul insisted on Jenny going out to dinner with him that night.
C. Paul invited Jenny to go out to dinner with him that night.
D. Pau offered Jenny to go out to dinner with him that night
Question 32. Đáp án: C
Kiến thức: Câu đồng nghĩa – Câu tường thuật
Giải thích:
Câu đề bài: “Em có muốn ra ngoài ăn tối với anh hôm nay không Jenny?” Paul
hỏi. (Đây là cấu trúc dùng để mời).
Xét các đáp án:
A. Paul gợi ý rằng Jenny nên ra ngoài ăn tối với anh ấy hôm đó. → sai ý
B. Paul nằng nặc muốn Jenny ra ngoài ăn tối với anh ấy hôm đó. → sai ý
C. Paul mời Jenny ra ngoài ăn tối với anh ấy hôm đó. → đúng
D. Không dịch vì sai cấu trúc: Động từ “offer” không có cấu trúc “offer sb to do
sth”.
Đáp án C đúng ý đề bài cho. Các đáp án còn lại sai ý hoặc sai cấu trúc.
Question 33. he last time I ate spaghetti was five months ago.
A. I didn’t eat spaghetti five months ago. B. I would eat
spaghetti in five months.
C. I have eaten spaghetti for five months. D. I haven’t eaten
spaghetti for five months.
Kiến thức: Câu đồng nghĩa – Thì của động từ
Giải thích:
Ta có công thức:
S + haven’t/ hasn’t + V- pp + O + for + time
= S + (last) + V-simple past + O + time + ago
= It’s + time + since + S + (last) + V-simple past + O
= The last time + S + V-simple past + O + was + time + ago
Vậy đáp án đúng là D
The last time I ate spaghetti was five months ago.
= D. I haven’t eaten spaghetti for five months.
Tạm dịch: Lần cuối cùng tôi ăn mì Ý là năm tháng trước.
= D. Tôi đã không ăn mì Ý được năm tháng.
Question 34.
The 2020 Summer Games were postponed due to the coronavirus
pandemic but organizers have been determined to (34) _______ they
go ahead in July.
A. acclaim B. ensure C. remain D. assure
Kiến thức: Đọc điền từ
Giải thích:
Xét các đáp án:
A. acclaim (v): hoan nghênh B. ensure (v): đảm bảo (+ st/ that
+ clause)
C. remain (v): còn lại D. assure (v): đảm bảo (+ st)
Dựa vào nghĩa và ngữ cảnh đáp án B là đúng
Thông tin: The 2020 Summer Games were postponed due to the
coronavirus pandemic but organizers have been determined to (34)
___ ensure ___ they go ahead in July.
Tạm dịch: Thế vận hội mùa hè 2020 đã bị hoãn do đại dịch coronavirus
nhưng các nhà tổ chức vẫn quyết tâm đảm bảo rằng họ sẽ diễn ra vào
tháng 7.
Question 35
It began in Fukushima Prefecture, (35) _______ was devastated by the triple disasters
of the 2011 earthquake, tsunami and nuclear meltdown.
A. who B. where C. which D. whom
Kiến thức: Đọc điền từ
Giải thích:
A. who: thay thế cho danh từ chỉ người có chức năng làm chủ ngữ hoặc tân ngữ
trong mệnh đề quan hệ.
B. where: thay thế cho danh từ chỉ nơi chốn và có chức năng làm trạng từ chỉ nơi
chốn trong mệnh đề quan hệ.
C. which: thay thế cho danh từ chỉ vật có chức năng làm chủ ngữ hoặc tân ngữ
trong mệnh đề quan hệ.
D. whom: thay thế cho danh từ chỉ người có chức năng làm tân ngữ trong mệnh
đề quan hệ.
Ta cần đại từ quan hệ thay thế cho danh từ “Fukushima Prefecture” – danh từ chỉ nơi
chốn và làm chủ ngữ trong mệnh đề quan hệ không xác định.
Thông tin: It began in Fukushima Prefecture, (32) ___ which ___ was devastated
by the triple disasters of the 2011 earthquake, tsunami and nuclear meltdown.
Tạm dịch: Nó bắt đầu ở tỉnh Fukushima, nơi bị tàn phá bởi ba thảm họa động đất, sóng thần
và thảm họa hạt nhân năm 2011.
Question 36.
although (36) _______ people say claims of recovery are a little
premature.
A. much B. every C. a lot D. many
Question 36. Đáp án: D
Kiến thức: Đọc điền từ
Giải thích:
A. much: nhiều (dùng với danh từ không đếm được)
B. every: mọi (dùng với danh từ không đếm được)
C. a lot (+ of : dùng với cả danh từ số nhiều và danh từ không
đếm được)
D. many: nhiều (dùng với danh từ số nhiều)
Ta thấy, sau chỗ trống là danh từ số nhiều (people) nên đáp án D là
đúng
Question 37
. (37) _______ must wear face masks, and are being urged to clap rather
than cheer.
. A. Audiences B. Viewers C. Spectators D. Watchers
Question 37. Đáp án: C
Kiến thức: Đọc điền từ Giải thích:
Xét các đáp án:
A. Audiences: khán giả (xem ở rạp) B. Viewers: người xem/ khán giả
(xem TV)
C. Spectators: khán giả (xem ở sân vận động) D. Watchers: người xem
Ta thấy, đoạn văn nói về thế vận hội mùa hè được tổ chức ở Nhật, nên đáp án C là
đúng (spectators: khan giả xem ở sân vận động)
Thông tin: Japanese people are being encouraged to look at live broadcasts
of the relay and refrain from travelling to watch it amid fears of a spike in
COVID 19 cases. (37) ___ Spectators ___ must wear face masks, and are
being urged to clap rather than cheer.
Tạm dịch: Người dân Nhật Bản đang được khuyến khích xem các chương trình phát
sóng trực tiếp của cuộc tiếp sóng và hạn chế đi lại để xem nó trong bối cảnh lo ngại
về sự gia tăng đột biến các trường hợp mắc COVID 19. Khán giả phải đeo khẩu
trang và được khuyến khích vỗ tay hơn là cổ vũ.
Question 38. A. in spite of B. because of C. due to
D. with a view to
Question 38. Đáp án: A
Kiến thức: Đọc điền từ
Giải thích:
Xét các đáp án:
A. in spite of (+ V-ing/ N.P): mặc dù B. because of (+ V-
ing/ N.P): bởi vì
C. due to (+ V-ing/ N.P): bởi vì D. with a view to (+ V-
ing/ N.P): với ý định
Dựa vào nghĩa và ngữ cảnh, đáp án đúng là A
Thông tin: Japan's Prime Minister Yoshihide Suga reiterated his
commitment to stage a “safe and secure” Olympics (38) ___
in spite of ___ the pandemic.
Tạm dịch: Thủ tướng Nhật Bản Yoshihide Suga nhắc lại cam kết tổ
chức một Thế vận hội “an toàn và bảo mật” bất chấp đại dịch.
Question 39. What is the passage mainly about?
A. The natural homes for zoos’ animals. B. The suffering zoos bring
captured animals.
C. The things zoo officials concern. D. The comfort for zoo visitors.
Question 39: Đáp án B
Kiến thức: Đọc hiểuGiải thích: Ý chính của bài là gì?
Xét các đáp án:
A. Ngôi nhà tự nhiên của các loài động vật trong vườn thú.
B. Các vườn thú đau khổ mang theo những con vật bị bắt.
C. Những điều các quan chức sở thú quan tâm.
D. Sự thoải mái cho khách tham quan vườn thú.
Thông tin: Zoo officials say that they are concerned about animals. However,
most zoos remain "collections" of interesting "items" rather than protective
habitats. Zoos teach people that it is acceptable to keep animals in captivity.
However, animals in zoos are bored, limited, lonely, and far from their natural
homes.
Tạm dịch: Các quan chức sở thú nói rằng họ lo ngại về động vật. Tuy nhiên, hầu
hết các vườn thú vẫn là "bộ sưu tập" các "vật phẩm" thú vị hơn là môi trường
sống bảo vệ. Các vườn thú dạy mọi người rằng việc nuôi nhốt động vật được chấp
nhận. Tuy nhiên, các loài động vật trong vườn thú cảm thấy buồn chán, hạn chế,
cô đơn và xa nhà tự nhiên của chúng.
Question 40. The word “observe” in paragraph 2 is closest
in meaning to ______.
A. watch B. feed C. raise D. touch
Kiến thức: Đọc hiểu
Giải thích: Từ “observe” trong đoạn 2 gần nghĩa nhất
với ______.
Observe: xem/ quan sát
Xét các đáp án:
A. watch: xem B. feed: nuôi
C. raise: nuôi D. touch: chạm vào
Vậy đáp án đúng là A
Từ đồng nghĩa: observe = watch
Thông tin: Most zoo areas are quite small, and visitors can
rarely observe animals’ normal behavior in these
unnatural spaces.
Tạm dịch: Hầu hết các khu vực vườn thú đều khá nhỏ và
du khách hiếm khi có thể quan sát hành vi bình thường
của động vật trong những không gian phi tự nhiên này.
Question 41. According to the passage, zoochosis is popular among animals in
______.
A. the wild B. natural homes C. cages D. large spaces
Question 41: Đáp án C
Kiến thức: Đọc hiểu
Giải thích: Theo đoạn văn, zoochosis phổ biến ở các loài động vật trong ______.
Xét các đáp án:
A. hoang dã B. ngôi nhà tự nhiên
C. lồng D. không gian rộng lớn
Thông tin: This results in abnormal and self-destructive behavior called zoochosis.
A worldwide study of zoos found that zoochosis is common among animals in small
spaces or cages. For example, Bears spend 30 percent of their time walking back
and forth. These are all signs of distress.
Tạm dịch: Điều này dẫn đến hành vi bất thường và tự hủy hoại được gọi là
zoochosis. Một nghiên cứu trên toàn thế giới về các vườn thú cho thấy rằng bệnh
zoochosis là phổ biến ở các loài động vật trong không gian nhỏ hoặc lồng. Ví dụ:
Gấu dành 30 phần trăm thời gian của chúng để đi đi lại lại. Đây là tất cả các dấu
hiệu của sự đau khổ.
Question 42. The word “them” in paragraph 4 refers to ______.
A. signs B. their habitats C. endangered animals
D. zoos
Kiến thức: Đọc hiểu
Giải thích: Từ “them” trong đoạn 4 đề cập đến ______.
Xét các đáp án:
A. dấu hiệu B. môi trường sống của chúng
C. động vật có nguy cơ tuyệt chủng D. vườn thú
Thông tin: However, zoos that breed endangered animals, such as big
cats and Asian elephants, often do not release them to the wild.
Tạm dịch: Tuy nhiên, các vườn thú nuôi các loài động vật có nguy cơ
tuyệt chủng, chẳng hạn như mèo lớn và voi châu Á, thường không
thả chúng về tự nhiên.
Question 43. Which of the following is TRUE according to the passage?
A. Zoos educate people by showing them animals’ natural behavior.
B. Zoos remain animals’ protective habitats.
C. Zoos always release endangered animals to the wild.
D. Zoos keep animals in small spaces
Question 43: Đáp án: D
Kiến thức: Đọc hiểu
Giải thích: Điều nào sau đây là ĐÚNG theo đoạn văn?
Xét các đáp án:
A. Các vườn thú giáo dục con người bằng cách cho họ thấy hành vi
tự nhiên của động vật.
B. Các vườn thú vẫn là môi trường sống bảo vệ của động vật.
C. Các vườn thú luôn thả các loài động vật có nguy cơ tuyệt chủng
về tự nhiên.
D. Vườn thú nuôi nhốt động vật trong không gian nhỏ
Thông tin: The animals are kept together in small spaces, with no privacy
and little opportunity for mental stimulation or physical exercise.
Tạm dịch: Các con vật được nhốt chung với nhau trong những không gian
nhỏ, không có sự riêng tư và ít có cơ hội để kích thích tinh thần hoặc rèn
luyện thể chất.
Question 44. Which of the following could be the main idea of the passage?
A. Being optimistic is an effective way to get over bad situations.
B. Keeping positive or negative thoughts is the own choice of each person.
C. Positive thoughts are necessary conditions to be successful.
D. There seems to have more pessimists than optimists.
Kiến thức: Đọc hiểu
Giải thích: Câu nào trong các câu sau đây có thể là ý chính của đoạn văn?
A. Lạc quan là một cách rất hiệu quả để vượt qua các hoàn cảnh khó khăn.
B. Giữ suy nghĩ tích cực hay tiêu cực là lựa chọn riêng của mỗi người.
C. Suy nghĩ tích cực là điều kiện cần thiết để thành công.
D. Dường như là có nhiều người bi quan hơn lạc quan.
Thông tin 1: Have you ever seen people who have just endured an awful situation?
Some focus on what they have lost, and this is easy to understand. But other people
focus on what they did not lose, and they start thinking about a better future.
Tạm dịch: Bạn đã từng thấy những người phải trải qua một tình huống tồi tệ? Vài
người chỉ chăm chú vào những gì họ đã mất, và điều này cũng dễ hiểu. Nhưng
những người khác chỉ tập trung vào những điều họ chưa mất, và họ bắt đầu nghĩ về
1 tương lai tốt hơn.
Thông tin 2: It may sound like a cliché. While a positive attitude may not be the
answer to every problem, it can certainly give you an advantage in surviving most of
life’s minor setbacks.
Tạm dịch: Nó nghe như 1 câu sáo rỗng. Mặc dù thái độ tích cực không phải là chìa
khóa cho mọi vấn đề, nhưng nó chắc chắn sẽ cho bạn cơ hội vượt qua mọi khó
sốngQuestion 45. The word “grief” in paragraph 3 is closest in
meaning to _______.
A. problem B. sorrow C. disappointment D. damage
Kiến thức: Đọc hiểu
Giải thích: Từ “grief” trong đoạn 3 gần nghĩa nhất với từ
___________.
A. vấn đề B. nỗi buồn
C. sự thất vọng D. sự thiệt hại
Từ đồng nghĩa: grief (nỗi đau buồn) = sorrow
Thông tin: Both have lost their homes and all their belongings in
a devastating storm. One family cannot mask their grief. They
feel that everything they hold dear has been destroyed.
Tạm dịch: Cả 2 gia đình đều mất nhà và toàn bộ của cải trong
trận bão kinh hoàng. Một gia đình không thể che giấu nỗi buồn
của họ. Họ cảm thấy rằng mọi thứ họ có đã bị phá hủy hoàn
toàn.
Question 46. What does the word “they” in paragraph 3 refer to?
A. tragic events B. all of the people C. the first
family D. the second family
Kiến thức: Đọc hiểu
Giải thích: Từ “they” trong đoạn 3 đề cập đến từ nào?
A. thảm kịch B. tất cả mọi người
C. gia đình thứ nhất D. gia đình thứ hai
Thông tin: However, the second family certainly seems to be
better off. They are thinking about making progress rather than
focusing on the tragic events.
Tạm dịch: Tuy nhiên, gia đình thứ 2 chắc chắn có vẻ tốt hơn. Họ
đang suy nghĩ về việc làm sao để cải thiện tình hình chứ không
tập trung vào thảm kịch.
Question 47. It can be inferred from the third paragraph that _______.
A. disappointment and sadness are all people’s common emotions in terrible
situations.
B. your attitude in terrible situations is more important than how serious the
problems are.
C. optimists often suffer less terrible situations than pessimists.
D. your attitude will decide the way you react to terrible situations.
Kiến thức: Đọc hiểu
Giải thích: Có thể suy ra từ đoạn 3 rằng _______.
A. Sự thất vọng và nỗi buồn là cảm xúc chung của tất cả mọi người trong hoàn cảnh
khó khăn.
B. Thái độ của bạn trong các hoàn cảnh khó khăn thì quan trọng hơn là việc vấn đề đó
nghiêm trọng đến mức nào.
C. Người lạc quan thường ít gặp phải các hoàn cảnh khó khăn hơn người bi quan.
D. Thái độ của bạn sẽ quyết định cách bạn phản ứng lại các hoàn cảnh khó khăn.
Thông tin: Imagine two families: Both have lost their homes and all their belongings in a
devastating storm. One family cannot mask their grief. They feel that everything they
hold dear has been destroyed. They cannot imagine how they will ever be able to
replace things and start over again. Their normal life seems to have been completely
lost. In contrast, a second family is crying with joy. All of the people in their family are
unharmed and safe. This family is just happy that everyone has survived. This family is
already trying to figure out how they can recover. You can’t really blame the first family
for experiencing a very normal reaction to a terrible situation. However, the second
family certainly seems to be better off. They are thinking about making progress rather
Question 48. The word “scenario” in paragraph 4 mostly means
_______.
A. trouble B. background C. circumstance D.
imagination
Question 48. Đáp án C
Kiến thức: Đọc hiểu
Giải thích: Từ “scenario” trong đoạn 4 có nghĩa là _______.
A. khó khăn B. bối cảnh
C. tình huống D. sự tưởng tượng
Từ đồng nghĩa: scenario (kịch bản, hoàn cảnh, tình huống) =
circumstance
Thông tin: Though this scenario is extreme, everyone
experiences setbacks that seem just awful at the time.
Tạm dịch: Mặc dù tình huống này hơi cực đoan, nhưng dường
như ai cũng có lúc trải qua khó khăn trong cuộc đời.
Question 49. According to the passage, which of the following is NOT true?
A. The thing people have to remember is managing their own reaction to
bad situations.
B. Everyone will suffer some terrible experiences in their life.
C. Paying attention to the solutions of the setback is better than focusing
on the damage it causes.
D. To have a good foundation for the future, you should not undergo bad
situations today.
Question 49. Đáp án D
Kiến thức: Đọc hiểu
Giải thích: Theo đoạn văn, câu nào là KHÔNG đúng?
A. Điều mà mọi người phải nhớ là kiểm soát phản ứng của mình trong các hoàn
cảnh khó khăn.
B. Mỗi người cũng sẽ trải qua các khó khăn trong cuộc đời của họ.
C. Tập trung vào các giải pháp giải quyết khó khăn tốt hơn là chăm chú vào các
thiệt hại do khó khăn gây ra.
D. Để có 1 nền tảng tốt trong tương lai, bạn không nên trải qua các hoàn cảnh khó
khăn hôm nay.
Thông tin 1: One good piece of advice to remember is that you cannot always
control situations or other people. The only thing you can control is your own
personal reaction to bad situations.
Thông tin 2: Nobody gets through life without having some bad things happen. In
these situations, try to focus on the steps you can take to remedy the situation,
instead of how awful the setback is. By doing this, you will be laying the foundation
for a better tomorrow. And you will not suffer as much pain today. (Đoạn 4)
Question 50. According to paragraph 5, what is the major thing you should
do when you have troubles?
A. Be optimistic and make out a plan. B. Ask other people for
help when necessary.
C. Control your emotions. D. Determine how serious the
problem is.
Kiến thức: Đọc hiểu
Giải thích: Theo đoạn 5, điều cốt yếu bạn nên làm khi gặp khó khăn là gì?
A. Hãy lạc quan và vạch ra 1 kế hoạch.
B. Hãy nhờ người khác giúp đỡ khi cần thiết.
C. Hãy kiểm soát cảm xúc của bạn.
D. Hãy xác định xem khó khăn của bạn nghiêm trọng tới mức nào.
Thông tin: The bottom line is, no matter what the problem is, you are more
likely to fix it if you can stay positive and work out a plan.
Tạm dịch: Điểm mấu chốt là, cho dù khó khăn của bạn là gì đi nữa, bạn đều
có thể vượt qua được nếu bạn luôn lạc quan và vạch ra 1 kế hoạch.
Question 1: Lan wants to learn English because it is
an____________ language.
A. internationally B. international
C. internationalize D. internationalism
A. Internationally /inte’næ∫nəli/ (adv): cấp độ toàn thế giới
B. International /,intə’næ∫nəl/ (a): thuộc về quốc tế
C. Internationalize /intə’næʃnəlaiz/ (v): quốc tế hóa
D. Internationalism /intə’næ∫nəlizəm/ (n): chủ nghĩa quốc tế
Căn cứ vào danh từ “language” thì vị trí còn trống cần một tính từ để
tạo thành một cụm danh từ. Ta loại được các phương án A, C, D.
Question 2: Let’s go to the cinema to watch “End game”,
_______?
A. will you B. don’t you C. shall we D. don’t we
Dịch nghĩa: “Chúng ta cùng đi đến rạp chiếu phim để xem bộ
phim “End game”, phải không nào?”
Câu bắt đầu với let’s có ý rủ rê thì thành lập hỏi đuôi sẽ dùng
shall we.
Question 3. He will apply for a job ________ .
A. when he is graduating from university
B. until he graduated from university.
C. after he had graduated from university
D. as soon as he graduates from university.
Kiến thức về mệnh đề trạng ngữ Xét các đáp án ta thấy:
- Động từ của vế câu cho trước ở trong đề bài ở thì tương lai →
diễn tả một hành động chưa xảy ra hay sẽ xảy ra ở trong tương lai
nên ở vế sau, không thể chia động từ ở thì quá khứ, quá khứ tiếp
diễn hay quá khứ hoàn thành.
→ loại A, B, C → 4 đáp án D đúng (phối thì hiện tại đơn với tương
lai đơn)
Dịch: Anh ta sẽ đi xin việc ngay sau khi anh ta tốt nghiệp
Question 4: Not many people are aware ____ male preference in
this company.
A. with B. on C. about D. of
* Dịch nghĩa: Không nhiều người nhận thức được sự ưu tiên nam
giới trong công ty này.
* Căn cứ vào cấu trúc:
- to be aware of: nhận thức về.
Question 5: They had to wait ten minutes for the anesthetic to
take________ before they stitched up the cut.
A. effect B. impact C. influence D. affect
A. effect /1 fekt/ (n): có kết quả, có hiệu lực, có tác dụng ánh huởng
B. impact /impækt/ (n). có lác động, ảnh huờng mạnh mé (thường là
cái gi mơi mé)
C. influence/ influans/ (n): sức mạnh, ảnh huởng, tác dộng đến ai
D. affect /e Tekl/ (v) tác động, ảnh hưởng dễn ai
*Cụm từ: Take offect. phát tác, có tác dung (thuốc thang) hoặc tao
ra.dat dược kết quả như mong quôn Tạm dịch: Họ phải dợi mười phút
để thuốc mê có hiệu luc trước khi khâu vét cát.
Question 6: ____ many international agreements affirming their
human rights, women are still much more likely than men to be poor
and illiterate.
A. Although B. Despite C. Because D. Since
Dịch nghĩa: Mặc dù nhiều xác nhận các thỏa thuận quốc tế quyền
con người của họ, nữ giới vẫn nhiều hơn nam giới về số lượng nghèo
và mù chữ.
Xét các đáp án:
A. Although + clause: mặc dù
B. Despite + N/Ving: mặc dù
C. Because + clause: bởi vì
D. Since = because + clause: bởi vì
*Note: “affirming” là danh động từ, có chức năng là một danh từ
chứ không phải Ving, cùng với “agreements” tạo thành cụm danh từ,
và cả cụm “many….rights” là một cụm danh từ hoàn thiện, do đó ta
chọn B.
Question 7: “What is the groom wearing?” “He dresses in a
_______________.”
A. light suit summer B. light summer suit
C. suit summer light D. summer suit light
*Theo quy tắc trật tự tính từ trong câu: OSASCOMP
light (a) - C (color)
*Note: summer suit (phr.v): trang phục/ bộ com-lê mùa hè
- Tính từ luôn đứng trước danh từ để bổ nghĩa cho danh từ
Tạm dịch: “Chú rể sẽ mặc gì?” – “Anh ấy mặc bộ com-lê mùa hè màu
nhạt”
Question 8: Can you tell me how the problem _________________
in the first place?
A. fell through B. brought round C. got out D. came about
A. fell through: thất bại
B. brought round: làm cho tỉnh
C. got out: rời nhà, đi chơi với
D. came about: xảy đến, xảy ra
Dịch nghĩa: Bạn có thể cho tôi biết ngay từ đầu vấn đề đã xảy ra
như thế nào không?
Question 9: When we went out, the sun _______.
A. was shining B. is shining C. shines D. shone
Kiến thức: Thì quá khứ tiếp diễn
Giải thích:
Dấu hiệu: “when” (khi), động từ ở mệnh đề trạng ngữ chỉ thời
gian ở thì quá khứ đơn => loại B, C (vì chia thì hiện tại).
Thì quá khứ tiếp diễn dùng để diễn tả một hành động đang diễn
ra tại một thời điểm trong quá khứ (chia quá khứ tiếp diễn), nếu
có hành động khác xen vào thì chia hành động đó ở quá khứ
đơn.
Công thức: S + was/were Ving.
Phối hợp thì: When + S + V-quá khứ đơn, S + was/were +
V_ing.
Tạm dịch: Khi chúng tôi đi ra ngoài, mặt trời đã chói chang.
Question 10: She will have finished the preparations for the meeting
_______.
A. by the time her boss arrives B. after her boss had arrived
C. as soon as her boss had arrived D. when her boss arrived
Kiến thức: Mệnh đề trạng ngữ chỉ thời gian
Giải thích:
Do mệnh đề chính thì tương lai nên mệnh đề trạng ngữ chỉ thời
gian ở thì hiện tại => loại B, C, D (vì chia quá khứ).
Công thức: By the time S + V(s/es), S + will have Ved/V3.
Tạm dịch: cô ta sẽ hoàn thành công việc chuẩn bị cho cuộc
họp vào lúc sếp đến.
Question 11: The two people________badminton seemed to be at
it quite intensely.
A. going B. playing C. doing D. practicing
A. go /gou/ (v) dùng trước tên của các môn thể thao thuờng có
dang "Ving
B. play /pler/ (v); dùng trước tên của các môn thể thao đồng đội
C. do /du / dùng trước tên của các môn thể thao cá nhân
D. practise / præktıs/ (v): thực hành, luyện tập
Tạm dịch: Hai người đang chơi cầu lông ở đó có vẻ khá mãnh liệt.
Question 12. She is a kind of woman who does not care much of
work but generally ________ meals, movies or late nights at a club
with her colleagues.
A. supposes B. discusses C. attends D. socializes
Kiến thức về từ vựng
A. suppose (v): giả sử
B. discuss (v): bàn luận
C. attend (v): tham dự
D. socialize (v): hoà nhập với ai + with
Dịch: Cô ấy là kiểu phụ nữ không quan tâm nhiều đến công việc
mà thường giao lưu ăn uống, xem phim hay đi ăn khuya tại câu lạc
bộ với đồng nghiệp.
Question 13: Many people ______ against COVID-19 last year.
A. vaccinated B. were vaccinating C. were vaccinated
D. vaccinate
Kiến thức: Câu bị động
Giải thích:
Người không thể tự thực hiện hành động “vaccinate” (tiêm
vắc xin), mà là do bác sĩ tiêm => động từ ở dạng bị động =>
loại A, B, D
Câu bị động thì quá khứ đơn: S + was/were Ved/V3.
vaccinate => vaccinated
Tạm dịch: Nhiều người đã được tiêm vắc xin phòng chống
COVID-19 năm ngoái.
Question 14: _______ all the exercises, I went out with my best friend.
A. To do B. Having done C. Being done D. Had done
Kiến thức: Rút gọn mệnh đề đồng ngữ
Giải thích:
2 mệnh đề có cùng chủ ngữ (she), có thể rút gọn một mệnh đề về
dạng:
- Ving: khi mệnh đề dạng chủ động và hai hành động xảy ra liên
tiếp
- Ved: khi mệnh đề dạng bị động
- Having Ved/V3: khi mệnh đề dạng chủ động, hành động được
rút gọn xảy ra trước và là nguyên nhân dẫn đến hành động còn
lại => Dạng bị động: Having been Ved/V3.
Câu đầy đủ: I had done all the exercises. I. went out with my best
friend.
Câu rút gọn: Having done all the exercises, I went out with my
best friend.
Tạm dịch: Sau khi hoàn thành tất cả các bài tập, Tôi đi chơi với
bạn thân của mình.
Question 15: My neighbor is driving me mad! It seems that the later
it is at night, __________ he plays his music!
A. the more loud B. less
C.the more loudly D. the louder
Dịch nghĩa: Hàng xóm của tôi đang khiến tôi phát điên! Có vẻ như
càng về đêm, anh ấy càng chơi nhạc to hơn!
Xét các đáp án:
A. the more loud → Sai cấu trúc so sánh hơn (loud là từ 1 âm tiết)
B. less → Dựa vào nghĩa ta thấy câu B sai
C. the more loudly → Sai cấu trúc so sánh hơn (late là từ 1 âm tiết)
D.the louder → Cấu trúc so sánh kép khi nói về 2 người hoặc sự vật:
The more/-er + S + V, the more/-er + S + V
Question 16: - Kate: How lovely your cats are!
- David: “ ________ ”
A. I love them, too B. Thank you, it is nice of you to say so
C. Can you say it again D. Really? They are
Kiến thức: Hội thoại giao tiếp
A. Tôi cũng yêu chúng.
B. Cảm ơn bạn, bạn thật tử tế khi nói như vậy.
C. Bạn có thể nói lại được không.
D. Thật sao? Chúng dễ thương thật.
Tạm dịch:
- Kate :”Lũ mèo của bạn đáng yêu quá!”.
- David: “Cảm ơn bạn, bạn thật tử tế khi nói như vậy”.
Question 17: -John: “Everyone should learn more about how to treat
the environment well."
- Jack: “________ ”
A. That's not true B. I am sure about that.
C. I don't think so. D. It's not true
Kiến thức: Hội thoại giao tiếp
A. Điều đó không đúng
B. Tôi chắc chắn về điều đó
C. Tôi không nghĩ vậy
D. Nó không đúng
Tạm dịch:
- John: “Mỗi người nên tìm hiểu nhiều hơn về cách đối xử tử tế với
môi trường”
- Jack: “Tôi không nghĩ vậy”.
Question 18:
A. sacrifice B. understand C. integrate D. recognize
Kiến thức: Trọng âm
A. /'sækrɪfaɪs/, trọng âm rơi vào âm tiết thứ 1
B. /,ʌdə'stænd/, trọng âm rơi vào âm tiết thứ 2
C. /’ɪntɪgreɪt/, trọng âm rơi vào âm tiết thứ 1
D. /’rekəgnaɪz/, trọng âm rơi vào âm tiết thứ 1
Question 19: A. social B. mature C. secure D. polite
Kiến thức: Trọng âm
A. /'səufəl/ trọng âm rơi vào âm tiết thứ 1
B. /mə’tʃuə/ trọng âm rơi vào âm tiết thứ 2
C. /sɪ’kuə/ trọng âm rơi vào âm tiết thứ 2
D. /pə’laɪt/ trọng âm rơi vào âm tiết thứ 2
Question 20:
A. coughed B. crooked C. cooked D. laughed
Kiến thức: phát âm đuôi ed
A. /kɔft/ B. /'krukɪd/ C. /kukt/ D. /lɑ:ft/
Question 21: A. band B. hand C. sand D. bank
Kiến thức: Phát âm
A. /bænd/
B. /hænd/
C. /sænd/
D. /bæŋk/
Question 22: The guards were ordered to get to the king's room on the
double.
A. in a larger number B. very quickly
C. on the second floor D. every two hours
Question 23: We can use either verbal or non-verbal forms of
communication.
A. using verbs B. using facial expressions
C. using speech D. using gesture
Kiến thức: Cụm từ
• on the double: nhanh chóng, không trì hoãn = very quickly
Tạm dịch: lính canh được lệnh đến phòng vua ngay lập tức
Kiến thức: Từ vựng
A. sử dụng động từ B. sử dụng nét mặt
C. sử dụng lời nói D. sử dụng cử chỉ
• Verbal (adj): (thuộc) lời nói = using speech
Tạm dịch: Chúng ta có thể sử dụng các hình thức giao tiếp bằng lời nói
hoặc không lời.
Question 24: You should put yourself on the back for having achieved such
a high score in the graduation exam.
A. wear a backpack B. praise yourself
C. criticize yourself D. check up your back
Kiến thức: Cụm từ
A. đeo ba lô B. khen ngợi bản thân
C. phê bình bản thân D. kiểm tra lưng của bạn
• Put yourself on the back: tự hào về bản thân >< criticize yourself
Tạm dịch: Bạn nên tự hào về bản thân vì đã đạt được số điểm cao như vậy
trong kỳ thi tốt nghiệp.
Question 25: The new policy will help generate more jobs.
A. produce B. bring out C. form D. terminate.
Kiến thức: Từ vựng
A. Produce = bring out = form: sản xuất
• Generate (v): Sản xuất, tạo ra >< Terminate (v): chấm dứt
Tạm dịch: Chính sách mới sẽ giúp tạo ra nhiều việc làm hơn
Question 26: She didn’t read the reference books. She wouldn’t be able to finish the
test.
A. Had she read the reference books, she would have been able to finish the test.
B. If she had read the reference books, she could finish the test.
C. Although she didn’t read the reference books, she was able to finish the test.
D. Not having read the reference books, she couldn’t finish the test.
Dịch nghĩa: Cô đã không đọc những cuốn sách tham khảo. Cô ấy sẽ không thể hoàn
thành bài kiểm tra.
Xét các đáp án:
A. Had she read the reference books, she would have been able to finish the test. →
Trong đảo ngữ với câu điều kiện loại 3 ta sử dụng cấu trúc: If-clause = Had + S + Vp2.
B. If she had read the reference books, she could finish the test. → Sai cấu trúc đảo
ngữ với câu điều kiện loại 3.
C. Although she didn’t read the reference books, she was able to finish the test. →
Mặc dù cô ấy không đọc sách tham khảo, cô ấy đã có thể hoàn thành bài kiểm tra, sai
nghĩa.
D. Not having read the reference books, she couldn’t finish the test. → Sai về cách
dùng của của couldn’t. Vì “could” là quá khứ của “can”-dùng để diễn tả khả năng có
thể làm gì như một bản năng, thường là do bẩm sinh mà có. Trong khi đó, câu gốc
đang dùng “be able to Vo”- dùng để diễn tả một khả năng có thể làm gì do cố gắng, nỗ
lực mới có được
Question 27. She helped us a lot with our project. We couldn’t continue without her.
A. Unless we had her contribution, we could continue with the project.
B. But for her contribution, we could have continued with the project.
C. If she hadn’t contributed positively, we couldn’t have continued with the project.
D. Provided her contribution wouldn’t come, we couldn’t continue with the project.
Kiến thức: Câu điều kiện loại 3
Giải thích:
Câu điều kiện loại 3 diễn tả điều giả định ngược với quá khứ Cấu trúc: If + S + had
Ved/ V3, S + would have Ved/ V3
= But for/ Without + noun phrase, S + would have Ved/ V3
Tạm dịch: Cô ấy giúp chúng tôi rất nhiều với dự án. Chúng tôi không thể thành công
nếu không có cô ấy.
A. sai ngữ pháp: câu điều kiện loại 2 => loại 3
B. Nếu không vì những giúp đỡ của cô ấy, chúng tôi sẽ đã tiếp tục với dự án. => sai
nghĩa
C. Nếu cô ấy không cống hiến 1 cách tích cực, chúng tôi sẽ đã không thể tiếp tục dự
án. => đúng
D. sai ngữ pháp: câu điều kiện loại 2 => loại 3
Question 28: The wooden fence surrounded the factory is beginning to fall
down because of the rain
A. wooden B. surrounded C. to fall down D. the rain
Kiến thức: rút gọn mệnh đề quan hệ
• Vì câu mang nghĩa chủ động nên rút gọn mệnh đề quan hệ về dạng Ving.
Sửa: Surrounded → surrounding
Tạm dịch: Hàng rào gỗ bao quanh nhà máy đang bắt đầu đổ vì trời mưa
Question 29: Experts in climatology and other scientists are becoming
extreme concerned about the changes to our climate which are taking place.
A. in B. extreme C. about D. are
Kiến thức: Từ loại
• Trạng từ đứng trước tính tử để bổ nghĩa cho tính từ đó.
Sai: extreme → extremely
Tạm dịch: Chuyên gia khí hậu học và những nhà khoa học khác đang cực kì lo
ngại về những thay đổi đang diễn ra đối với khí hậu của chúng ta.
Question 30: Her passion for helping people has motivated her to
found his own charity organization.
A. for B. has C. his D. organization
Kiến thức: sự hòa hợp giữa chủ ngữ và động từ
• Chủ ngữ là “her passion” → chia ngôi thứ 3 số ít.
Sửa: his → her
Tạm dịch: Niềm đam mê giúp đỡ người khác đã thúc đẩy cô ấy
thành lập tổ chức từ thiện của riêng mình.
S + have/ has + not + Ved/3………before
 It/This/That is the first time + S + have/ has + Ved/3
Question 31: I have never read such a good book before.
A. This is the first time I read a good book .
B. This is the first time I have read a good book.
C. This book is the best I have never read.
D. This book is the best I read .
Kiến thức: biến đổi thì
Sử dụng công thức:
Question 32: “He should to go to bed before 11p: m everyday”, I said
to him.
A. I advised him to go to bed before 11p: m everyday
B. He shouldn’t go to bed before 11p: m everyday
C. I prayed him to to go to bed before 11 p: m everyday
D. I ordered him you should stay in bed.
Kiến thức câu trực tiếp ở trần thuật
Advise sb to V = had better + V+ St / should + V +St
Question 33: Smoking is not allowed in the museum.
A. You mustn’t smoke in the museum.
B. You can smoke in the museum
C. You don’t have to smoke in the museum
D. You may smoke in the museum
Kiến thức Modal verbs với inf to V
Do “is not allowed” = must not
Question 34:
According to a new report, today's generation of children are in
danger of getting so little sleep that they are putting their mental
and physical health at (34) ________ .
A. risk B. threat C. danger D. jeopardy
Kiến thức: Cụm từ
• put sth at risk: đặt cái gì vào tình trạng nguy hiểm
Tạm dịch: Theo một báo cáo mới, thế hệ trẻ em ngày nay có nguy
cơ ngủ ít đến mức khiến sức khỏe tinh thần và thể chất của chúng
gặp nguy hiểm
Question 35:
Adults can easily survive on seven to eight hours' sleep a night, (35)
________ teenagers require nine or ten hours.
A. so B. or C. whereas D. because
Kiến thức: Từ vựng
A. vì thế
B. hoặc
C. trong khi
D. bởi vì
Tạm dịch: Người lớn có thể dễ dàng tồn tại khi ngủ từ bảy đến tám giờ
mỗi đêm, trong khi thanh thiếu niên cần chín hoặc mười giờ
Question 36:
Research has shown that losing as little as half an hour's sleep a
night can have profound (36) ________ on how children perform the
next day
A. effective B. effectively C. affect D. effect
Kiến thức: Từ vựng
Vị trí cần điền đứng sau tính từ nên sẽ là một danh từ
• Effect (n): tác động, ảnh hưởng
Tạm dịch: Nghiên cứu đã chỉ ra rằng mất ngủ ít nhất nửa tiếng mỗi
đêm có thể ảnh hưởng sâu sắc đến hoạt động của trẻ vào ngày hôm
sau
Question 37:
A good night's sleep is also crucial for teenagers because it is while
they are asleep (37) ________ they release a hormone that is
essential for their 'growth spurt' (the period during teenage years
when the body grows at a rapid rate).
A. that B. which C. when D. where
Kiến thức: Câu chẻ
• Câu chẻ hay còn được gọi là câu nhấn mạnh. Chúng được sử dụng
khi muốn nhấn mạnh vào một đối tượng hay sự việc nào đó.
• It is/was + trạng ngữ + that + S + V + O
Tạm dịch: Một giấc ngủ ngon cũng rất quan trọng đối với thanh thiếu
niên vì chính trong khi ngủ, chúng tiết ra một loại hormone cần thiết
cho quá trình tăng trưởng nhanh (giai đoạn ở tuổi thiếu niên khi cơ
thể phát triển với tốc độ nhanh).
Question 38:
It's true that they can, to(38) ________ extent, catch up on sleep at
weekends, but that won't help them when they are dropping off to
sleep in class on a Friday afternoon.
A. any B. some C. one D. every
Kiến thức: Cụm từ
• To some extent: ở một phạm vi, mức độ, chừng mực nào đó
Tạm dịch: Đúng là ở một mức độ nào đó, chúng có thể ngủ bù vào
cuối tuần, nhưng điều đó sẽ không giúp ích gì khi chúng ngủ gật
trong lớp vào buổi chiều thứ 6
39. What is the objective of the things Dr. Wray has done?
A. To make the environment less polluted and to have children from her marriage
with her husband.
B. To help people feel less lonely and overcome the bad emotions from climate
anxiety.
C. To describe and demonstrate the different kinds of eco-distress.
D. To provide people in some remote areas with the access to the counseling for
climate grief.
Kiến thức: Đọc hiểu
Giải chi tiết:
Mục tiêu của những việc Tiến sĩ Wray đã làm là gì?
A. Để môi trường bớt ô nhiễm và có con từ cuộc hôn nhân với chồng.
B. Để giúp mọi người cảm thấy bớt cô đơn và vượt qua những cảm xúc tồi tệ do lo
lắng về khí hậu.
C. Để mô tả và chứng minh các loại đau đớn sinh thái khác nhau.
D. Cung cấp cho người dân ở một số vùng sâu vùng xa khả năng tiếp cận với dịch
vụ tư vấn về các vấn đề khí hậu.
Thông tin: "My overall goal is to help people feel less alone," Dr. Wray said. "We
need to restore ourselves so we don't burn out and know how to be in this crisis
for the long haul that it is."
Question 40. What does the word "prevalence" in paragraph 3
mean?
A. impact B. appearance C. popularity D. recognition
Kiến thức: Đọc hiểu
Giải chi tiết:
Từ "prevalence" trong đoạn 3 có nghĩa là gì?
A. impact (n): tác động
B. appearance (n): sự xuất hiện
C. popularity (n): sự phổ biến
D. recognition (n): sự công nhận
=> prevalence (n): sự phổ biến = popularity
Thông tin: But as the prevalence of climate anxiety has grown, so
has the number of people working to alleviate it, both for
themselves and those around them.
Tạm dịch: Nhưng khi sự phổ biến của chứng lo âu về khí hậu ngày
càng gia tăng, thì số lượng người làm việc để giảm bớt nó, cho cả
bản thân và những người xung quanh cũng tăng lên.
Question 41. According to the passage, which of the following is TRUE?
A. Jennifer Atkinson couldn't sleep for fear of a mass extinction.
B. More and more people are worried about the climate change's effect on their
mental health
C. The anxiety about climate change is not widespread in less wealthy countries.
D. People can easily access the consultancy for climate distress in most parts of
the world.
Kiến thức: Đọc hiểu
Giải chi tiết:
Theo đoạn văn, điều nào sau đây là ĐÚNG?
A. Jennifer Atkinson không thể ngủ vì lo sợ về sự tuyệt chủng hàng loạt.
B. Ngày càng có nhiều người lo lắng về ảnh hưởng của biến đổi khí hậu đối với
sức khỏe tâm thần của họ.
C. Sự lo lắng về biến đổi khí hậu không phổ biến ở các nước kém giàu có hơn.
D. Mọi người có thể dễ dàng tiếp cận tư vấn về các vấn đề khí hậu ở hầu hết các
nơi trên thế giới.
Thông tin: But as the prevalence of climate anxiety has grown, so has the number
of people working to alleviate it, both for themselves and those around them.
Question 42. What is the main idea of the passage?
A. Types of eco-distress
B. When eco-distress affects human beings
C. The risk of an impending mass extinction
D. How specialists help address eco-distress
Kiến thức: Đọc hiểu
Giải chi tiết:
Ý chính của đoạn văn là gì?
A. Các dạng khó khăn về sinh thái
B. Khi tình trạng khó khăn về hệ sinh thái ảnh hưởng đến con người
C. Nguy cơ sắp xảy ra tuyệt chủng hàng loạt
D. Cách các chuyên gia giúp giải quyết vấn đề sinh thái
Thông tin: Distress over global warming is increasing, but formal and
informal support networks are springing up, too… "My overall goal is to
help people feel less alone," Dr. Wray said. "We need to restore ourselves
so we don't burn out and know how to be in this crisis for the long haul
that it is." Dr. Atkinson, in hopes of assuaging her feelings and those of her
students, designed a seminar on eco-grief and climate anxiety.
Question 43. What does the word "they" in paragraph 1 refer to?
A. activists B. children C. humanities D. students
Kiến thức: Đọc hiểu
Giải chi tiết:
Từ "they" trong đoạn 1 chỉ điều gì?
A. nhà hoạt động B. trẻ em C. nhân văn D. sinh viên
Thông tin: Jennifer Atkinson, an associate professor of environmental
humanities at the University of Washington, Bothell, became
depressed after students told her they couldn't sleep because they
feared social collapse or mass extinction.
Tạm dịch: Jennifer Atkinson, phó giáo sư về nhân văn môi trường tại
Đại học Washington, Bothell, trở nên trầm cảm sau khi sinh viên nói
với cô rằng họ không thể ngủ được vì sợ xã hội sụp đổ hoặc tuyệt
chủng hàng loạt.
Question 44. What is this passage mainly about?
A. gardens in Madrid, New York, and Chicago
B. people growing plants on roofs and walls
C. private gardens in the city's unused spaces
D. how to grow your own food in the city
44.
Đoạn này chủ yếu là về?
A. những khu vườn ở Madrid, New York và Chicago
B. người trồng cây trên mái nhà và tường
C. những khu vườn riêng trong những không gian chưa được sử dụng
của thành phố
D. cách tự trồng lương thực trong thành phố
Có thể thấy cả bài viết này nói về những người trồng cây trên mái
nhà và trên những bức tường. Các đáp án còn lại không chính xác
hoặc không bao quát được ý chính của cả bài.
Question 45. According to the passage, people ________ as cities
have little space for gardens.
A. grow plants in their apartments B. go to the countryside
at the weekends
C. paint their walls and roofs green D. grow trees and flowers
on top of roofs
Theo đoạn văn, mọi người ________ vì các thành phố có rất ít không
gian cho vườn.
A. trồng cây trong căn hộ của họ B. về quê vào cuối tuần
C. sơn tường và mái nhà màu xanh lá cây D. trồng cây và hoa trên
mái nhà
Thông tin này nằm ở câu thứ hai và câu thứ ba của đoạn đầu tiên:
"With limited space for parks and gardens... One creative solution is to
grow plants on unused areas like walls or rooftops." (Với không gian
dành cho công viên và vườn có hạn... Một giải pháp sáng tạo là trồng
cây ở những khu vực không sử dụng đến như tường hay mái nhà)
Question 46. Why is the green wall in Madrid mentioned in the
passage?
A. It is the most expensive green wall in the world.
B. It is located on the side of a government building.
C. It is a famous example of a green wall.
D. It was made to provide jobs for homeless people.
Tại sao bức tường xanh ở Madrid lại được nhắc đến trong đoạn văn?
A. Đó là bức tường xanh đắt nhất thế giới.
B. Nó nằm bên hông một tòa nhà chính phủ.
C. Đó là một ví dụ nổi tiếng về bức tường xanh.
D. Nó được tạo ra để cung cấp việc làm cho những người vô gia cư.
Thông tin này có thể được tìm thấy ở câu thứ 3 của đoạn 2:
"The CaxiaForum art gallery in Madrid, Spain, is a famous example - one
of its walls is covered with 15,000 plants from over 250 different
species." (Viện bảo tàng CaxiaForum ở Madrid, Tây Ban Nha là một ví
dụ nổi bật - một trong những bức tường ở đó được bao phủ bởi 15,000
cây với hơn 250 loài khác nhau).
Do đó "bức tường xanh" ở Madrid được nhắc đến như một ví dụ nổi
tiếng về bức tường xanh".
Question 47. Which is NOT mentioned as a benefit of a rooftop garden?
A. It becomes a park that the community can use.
B. Having one might mean paying less taxes.
C. Children can use it to learn about the environment.
D. Growing plants on a roof keeps buildings cooler.
Điều nào KHÔNG được đề cập như một lợi ích của khu vườn trên sân thượng?
A. Nó trở thành một công viên mà cộng đồng có thể sử dụng.
B. Có một có thể có nghĩa là phải trả ít thuế hơn.
C. Trẻ em có thể sử dụng nó để tìm hiểu về môi trường.
D. Trồng cây trên mái nhà giúp các tòa nhà mát mẻ hơn.
Đáp án B nằm ở thông tin đoạn 3: Many cities offer tax discounts to businesses with these
features. (Nhiều thành phố giảm giá thuế cho các doanh nghiệp có các tính năng này.)
Đáp án C nằm ở thông tin đoạn 4:
In addition to saving the school money, teachers and parents love the gardens because of
their educational value
- it's a fun and healthy way for their kids to investigate the world around them.
(Ngoài việc tiết kiệm tiền học, giáo viên và phụ huynh còn yêu thích những khu vườn vì giá trị
giáo dục của chúng - đó là một cách thú vị và lành mạnh để con họ khám phá thế giới xung
quanh.)
Đáp án D thông tin nằm ở đoạn 3:
Using plants to cover walls and rooftops can also keep cities cooler in the summer. (Sử dung
thực vật để che phủ các bức tường và mái nhà cũng có thể giữ cho các thành phố mát mẻ
hơn vào mùa hè.)
→ Chọn đáp án A
Question 48. Because food can now be grown in cities, ________ .
A. the food at expensive restaurants is cheaper
B. governments are making many rules about city gardens
C. farmers in the countryside are moving to the city
D. there's less pollution caused by transporting food
48.
Bởi vì lương thực hiện có thể được trồng ở các thành phố, ________ .
A. thức ăn ở nhà hàng đắt tiền rẻ hơn
B. chính phủ đang đưa ra nhiều quy tắc về vườn thành phố
C. nông dân ở nông thôn đang chuyển đến thành phố
D. ít ô nhiễm hơn do vận chuyển thực phẩm
Thông tin này có thể được tìm thấy ở câu thứ 3 của đoạn 5:
"This is based on the concept that locally grown food reduces pollution
since it does not have to be transported far." (Điều này dựa trên qian
niệm rằng thực phẩm được trồng tại địa phương làm giảm ô nhiễm môi
trường bởi nó không cần vận chuyển xa).
Question 49. The word "it" in paragraph 5 refers to ________ .
A. a rooftop garden B. the food in general
C. locally grown food D. green space
Từ "it" trong đoạn 5 đề cập đến ________ .
A. một khu vườn trên sân thượng
B. thức ăn nói chung
C. thực phẩm trồng tại địa phương
D. không gian xanh
It ở đây được dùng thay thế cho "locally grown food" được nhắc đến ở
vế trước của cậu.
Question 50. Why did Chef Bayless name his dish "Rooftop Salsa"?
A. He got the idea while cooking on his rooftop.
B. He buys the salsa from other rooftop gardeners.
C. It is made from food grown in his rooftop garden.
D. The money earned from the dish is given to rooftop gardeners.
Tại sao đầu bếp Bayless lại đặt tên món ăn của mình là "Rooftop Salsa"?
A. Anh ấy có ý tưởng khi đang nấu ăn trên sân thượng của mình.
B. Anh ấy mua salsa từ những người làm vườn trên sân thượng khác.
C. Nó được làm từ thực phẩm trồng trong khu vườn trên sân thượng của
anh ấy.
D. Số tiền kiếm được từ món ăn được đưa cho những người làm vườn trên
sân thượng.
Thông tin này có thể được suy ra từ câu cuối cùng của đoạn 5:
"Chef Rick Bayless serves “Rooftop Salsa” at his restaurant in Chicago, USA,
using only ingredients grown in his rooftop garden." (Đầu bếp Rick Bayless
phục vụ món “Salsa sân thượng” ở nhà hàng của mình ở Chicago, Mỹ, chỉ
sử dụng các nguyên liệu được trồng trong khu vườn trên mái nhà của
mình).
Câu 1. They are going to _______ the pool to 1.8 meter.
A. deepen B. depth C. deep D. deeply
Câu 1
Kiến thức về từ loại
Khoảng trống cần “động từ” do phía trước khoảng trống là “be
going to + Vo”
Trong đó:
A. deepen (v) B. depth (n) C. deep (a) D. deeply (adv)
 nên ta chọn A
Question 2. It’s a lovely day__________?
A. isn’t it B. was it C. doesn’t it D. did it
Kiến thức về câu hỏi đuôi: phía trước khẳng định của “to be” và chủ
ngữ là “it”
Question 3: The _____ time was a charm for Korea’s Pyeongchang
2018 Winter Olympic bid committee.
A. third B. first C. second D. forth
A. third /θɜːd/: thứ ba B. first /ˈfɜːst/: đầu tiên
C. second /ˈsekənd/: thứ hai D. forth /fɔːθ/: thứ tư
Ta có: Third time is a charm: lần thứ ba sẽ thành công, quá tam ba bận
Tạm dịch: Lần thứ ba sẽ thành công cho ủy ban đấu thầu Olympic mùa
đông Hàn Quốc Pyeongchang 2018.
Question 4. Simon Lake drew the inspiration ____ La submarine of
undersea travel and exploration from Twenty Thousand Leagues Under
the Sea.
A. at B. by C. for D. of
Kiến thức về giới từ
* Ta có cấu trúc sau:
Draw the inspiration for sth (to sb) from sth: lấy nguồn cảm hứng cho
cái gì (cho ai) từ cái gì
Question 5. The final winner will be the one who breaks through
______ and survives till the last minutes.
A. obstacles B. difficulty C. hindrance D. impediment
Kiến thức về từ vựng
A. obstacle /ˈɑːbstəkəl/ (n): khó khăn, trở ngại
B. difficulty /ˈdɪ əlt̬i/ (n): khó khăn
C. hindrance /'hindrәns/ (n): sự cản trở, ngăn cản, khó khăn
D. impediment /ɪmˈpedəmənt/ (n): sự cản trở, trở ngại, khó khăn
* Nhận thấy các danh từ đều thuộc cùng một trường nghĩa, tuy nhiên
cả 4 đáp án đều là danh từ đếm được, trong khi trước chỗ trống trong
đề bài không có mạo từ “a/an” nên vị trí này cần điền một dạng danh
từ số nhiều
→ Loại B, C, D
Tạm dịch: Người chiến thắng trong trận chung kết sẽ là người có bước
đột phá để vượt qua những khó khăn trở ngại và tiếp tục tồn tại cho
đến những phút giây cuối cùng.
* Note: Break through sth (phr.v): vượt qua, tạo ra bước đột phá cái gì
Question 6: Many students work to earn money ____________ their
parents are rich
A. because of B. despite C. however D. although
Dịch nghĩa: Nhiều sinh viên làm việc để kiếm tiền mặc dù cha mẹ họ
giàu có.
Xét các đáp án:
A. because of → Liên từ chỉ nguyên nhân, sau because of (bởi vì) là
noun/noun phrase (DT/cụm DT), không phải clause (mệnh đề)
B. despite → Liên từ chỉ sự đối lập, sau Despite (mặc dù) = In spite of
là noun/noun phrase/V_ing (DT/cụm DT/V_ing), không phải clause
(mệnh đề)
C. however → Liên từ chỉ sự đối lập, sau however (tuy nhiên) là
clause (mệnh đề), dịch cả câu: Nhiều sinh viên làm việc để kiếm tiền
tuy nhiên cha mẹ họ giàu có. (không hợp lý)
D. although → Liên từ chỉ sự đối lập, sau Although (mặc dù) = Even
though = Though = In spite that là clause (mệnh đề)
Question 7: She is going to marry a _______ man next year and
they’ll give birth to two children after getting married.
A. tall pretty English B. English tall pretty
C. tall English pretty D. pretty tall English
*Theo quy tắc trật tự tính từ trong câu: OSASCOMP
: pretty - Opinion; tall - Shape; English - Origin
Dịch: Cô ấy sẽ cưới một anh chàng người Anh cao đẹp năm tới và họ
sẽ sinh 2 con sau khi kết hôn.
Question 8. There are other problems of city life which I don't
propose to ____ at the moment.
A. go into B. go around C. go for D. go up
Kiến thức về cụm động từ
A. go into (phr.v): bắt đầu làm gì; thảo luận, kiểm chứng, mô tả, giải
thích một cách chi tiết và thận trọng
B. go around (phr.v): đủ cho mọi người trong nhóm; đến thăm ai; cư
xử tệ
C. go for (phr.v): chọn, say mê, cố gắng
D. go up (phr.v): tăng lên
Tạm dịch: Còn nhiều vấn đề khác về cuộc sống thành thị cái mà tôi
không muốn đi vào sâu để thảo luận kỹ ngay lúc này.
Question 9: When I last saw him, he________in the living room
A. is sitting B. has been sitting C. was sitting D. sitting
Kiến thức: Sự phối hợp về thì của động từ
Giải thích: Thì quá khứ đơn và quá khứ tiếp diễn kết hợp trong câu:
Diễn tả hành động đang xảy ra trong quá khứ thì một hành động khác
xen vào, hành động đang xảy ra chi thì quá khứ tiếp diễn, hành động
xen vào chia thì quá khứ đơn
Cấu trúc: S + was/ were + V_ing + when + S + Ved/bqt
Tạm dịch: Khi tôi gặp anh ta, anh ta đang ngồi trong phòng khách.
Question 10. Richard will look for a job __________.
A. after he had passed his exams B. before he passed his exams
C. while he was passing his exams D. as soon as he passes his exams
Kiến thức về mệnh đề trạng ngữ chỉ thời gian
* Ta có mệnh đề:
Richard will look for a job: chia thì tương lai đơn nên mệnh đề chỉ thời gian
phải ở thì hiện
Question 11: It is always difficult for designer sportswear to _____into the
market because there is too much competition from leading brands like
Adidas.
A. invade B. cut C. break D. interfere
A. invade /ɪnˈveɪd/ (v): xâm lược B. cut /kʌt/ (v): cắt
C. break /breɪk/ (v): làm gãy, vỡ D. interfere /ˌɪntəˈfɪər/ (v): cản trở
=> Cấu trúc: break into the market: thâm nhập thị trường
Dịch nghĩa: Luôn luôn khó khăn cho các nhà thiết kế đồ thể thao thâm nhập thị
trường vì có quá nhiều sự cạnh tranh từ các thương hiệu hàng đầu như Adidas.
Question 12. Though she lost her job last month, she still wanted to
save_____ so she said that she had left it willingly.
A. mouth B. face C. reputation D. fame
Kiến thức về cụm từ cố định
Ta có cụm từ:
Save face (idm): giữ thể diện, danh dự cá nhân, tránh để những người khác
không mất đi sự tôn trọng dành cho mình
Tạm dịch: Mặc dù cô ta bị mất việc từ tháng trước, nhưng cô ta vẫn muốn gìn
giữ thể diện cá nhân nên cô đã nói rằng mình tình nguyện bỏ công việc đó.
Question 13. The story................... by Agatha Christie
A. were written B. was written C. was written from D. wrote by
Phương pháp giải:
Kiến thức: Câu bị động:
Giải chi tiết: Câu bị động thì Quá khứ đơn. Hành động được nhấn mạnh là
câu chuyện được viết bởi Agatha Christie. Chủ ngữ là the story (số ít)=> chon
B
S + động từ tobe + V-ed/V3
Tạm dịch : Câu chuyện được viết bởi Agatha
Question 14: _________the instruction,she started to do the
assignment.
A. Read B. Having read C.Being read D.Being done
Kiến thức: Mệnh đề phân tử / Rút gọn mệnh đề đồng ngữ
Giải thích:
Khi 2 mệnh đề có cùng chủ ngữ (she) thì có thể rút gọn 1 trong 2
mệnh đề về dạng:
- V-ing / Having P2: nếu mệnh đề được rút gọn mang nghĩa chủ động
- P2 (quá khứ phân từ): nếu mệnh đề được rút gọn mang nghĩa bị
động
Tạm dịch: Sau khi đọc hướng dẫn, cô ta bắt đầu thực hiện bài tập.
Question 15____ the brushwork is in Stevenson’s landscapes, the more
vitality and character the painting seems to possess.
A. The loose B. loose
C. The loosest D. The looser
Dịch nghĩa: Nét vẽ càng mềm mại trong bức tranh phong cảnh của
Stevenson, thì nó càng có sức sống và có cá tính.
Xét các đáp án:
D. The looser the brushwork is → Cấu trúc so sánh kép khi nói về 2
người hoặc sự vật:
The more/-er + S + V, the more/-er + S + V
Question 16: Joana and David, two lectures, are talking about
library skills.
- Joana: “ I think we should teach our students how to use the
library” - David:”_______’
A. You're absolutely wrong. B. You must be kidding.
C. I couldn't agree with you more.D. That's not a good idea.
Tình huống: Joana và David, 2 giảng viên, đang nói chuyện về những kỹ
năng thư viện
- Joana: “Mình nghĩ chúng ta nên dạy học sinh sử dụng thư
viện như thế nào.”
- David: “______________.”
A. Bạn hoàn toàn sai.
B. Chắc hẳn bạn đang đùa.
C. Tôi hoàn toàn đồng ý với bạn.
D. Đó không phải là một ý kiến tốt.
Question 17: A: "Have you ever done any volunteer work?"
B: “_______.”
A. I'm doing a part-time job to support my student life.
B. You see, earning money is difficult these days.
C. Sure. When I was a student, I helped in the hospital.
D. I have been trying to work with all my heart.
HD: “Bạn đã từng làm việc tình nguyện chưa?”
A. Tôi đang làm một công việc bán thời gian để hỗ trợ cuộc sống sinh
viên của mình.
B. Bạn thấy đấy, ngày nay thì kiếm tiền rất khó.
C. Chắc chắn rồi. Khi tôi còn là học sinh, tôi đã làm giúp trong bệnh
viện.
D. Tôi đang cố gắng làm việc với tất cả trái tim của mình.
Question 18.A. approach B. panda C. income D. current
Kiến thức về trọng âm
A. approach /əˈproʊtʃ/: từ này có trọng âm rơi vào âm tiết thứ hai. Vì
theo quy tắc, trọng âm không bao giờ rơi vào nguyên âm /ə/.
B. panda /ˈpændə/: từ này có trọng âm rơi vào âm tiết thứ nhất. Vì
theo quy tắc, trọng âm không bao giờ rơi vào nguyên âm /ə/.
C. income /ˈɪnkʌm/: từ này có trọng âm rơi vào âm tiết thứ nhất. Vì
theo quy tắc, danh từ có hai âm tiết thì trọng âm thường rơi vào âm
tiết đầu.
D. current /ˈkɝrənt/: từ này có trọng âm rơi vào âm tiết đầu. Vì theo
quy tắc, trọng âm không bao giờ rơi vào nguyên âm /ə/.
Question 19.
A. confidence B. celebrate C. effective D. handicapped
Kiến thức về trọng âm
A. confidence /ˈkɑːnfədəns/: từ này có trọng âm rơi vào âm tiết đầu.
Vì theo quy tắc, trọng âm không bao giờ rơi vào nguyên âm /ə/.
B. celebrate /ˈseləbreɪt/: từ này có trọng âm rơi vào âm tiết đầu. Vì
theo quy tắc, đuôi –ate làm trọng âm dịch chuyển ba âm tính từ cuối
lên.
C. effective /əˈfektɪv/: từ này có trọng âm rơi vào âm tiết thứ hai. Vì
theo quy tắc, đuôi –tive làm trọng âm rơi vào âm tiết ngay trước nó.
D. handicapped /ˈhændɪkæpt/: từ này có trọng âm rơi vào âm tiết
thứ nhất.
Vì theo quy tắc, nếu tất cả các âm mà ngắn hết thì trọng âm sẽ rơi
vào âm tiết thứ nhất.
DE PHAT TRIEN THEO CAU TRUC DE MINH HOA 2022 MON TIENG ANH DE 16 20.pdf
DE PHAT TRIEN THEO CAU TRUC DE MINH HOA 2022 MON TIENG ANH DE 16 20.pdf
DE PHAT TRIEN THEO CAU TRUC DE MINH HOA 2022 MON TIENG ANH DE 16 20.pdf
DE PHAT TRIEN THEO CAU TRUC DE MINH HOA 2022 MON TIENG ANH DE 16 20.pdf
DE PHAT TRIEN THEO CAU TRUC DE MINH HOA 2022 MON TIENG ANH DE 16 20.pdf
DE PHAT TRIEN THEO CAU TRUC DE MINH HOA 2022 MON TIENG ANH DE 16 20.pdf
DE PHAT TRIEN THEO CAU TRUC DE MINH HOA 2022 MON TIENG ANH DE 16 20.pdf
DE PHAT TRIEN THEO CAU TRUC DE MINH HOA 2022 MON TIENG ANH DE 16 20.pdf
DE PHAT TRIEN THEO CAU TRUC DE MINH HOA 2022 MON TIENG ANH DE 16 20.pdf
DE PHAT TRIEN THEO CAU TRUC DE MINH HOA 2022 MON TIENG ANH DE 16 20.pdf
DE PHAT TRIEN THEO CAU TRUC DE MINH HOA 2022 MON TIENG ANH DE 16 20.pdf
DE PHAT TRIEN THEO CAU TRUC DE MINH HOA 2022 MON TIENG ANH DE 16 20.pdf
DE PHAT TRIEN THEO CAU TRUC DE MINH HOA 2022 MON TIENG ANH DE 16 20.pdf
DE PHAT TRIEN THEO CAU TRUC DE MINH HOA 2022 MON TIENG ANH DE 16 20.pdf
DE PHAT TRIEN THEO CAU TRUC DE MINH HOA 2022 MON TIENG ANH DE 16 20.pdf
DE PHAT TRIEN THEO CAU TRUC DE MINH HOA 2022 MON TIENG ANH DE 16 20.pdf
DE PHAT TRIEN THEO CAU TRUC DE MINH HOA 2022 MON TIENG ANH DE 16 20.pdf
DE PHAT TRIEN THEO CAU TRUC DE MINH HOA 2022 MON TIENG ANH DE 16 20.pdf
DE PHAT TRIEN THEO CAU TRUC DE MINH HOA 2022 MON TIENG ANH DE 16 20.pdf
DE PHAT TRIEN THEO CAU TRUC DE MINH HOA 2022 MON TIENG ANH DE 16 20.pdf
DE PHAT TRIEN THEO CAU TRUC DE MINH HOA 2022 MON TIENG ANH DE 16 20.pdf
DE PHAT TRIEN THEO CAU TRUC DE MINH HOA 2022 MON TIENG ANH DE 16 20.pdf
DE PHAT TRIEN THEO CAU TRUC DE MINH HOA 2022 MON TIENG ANH DE 16 20.pdf
DE PHAT TRIEN THEO CAU TRUC DE MINH HOA 2022 MON TIENG ANH DE 16 20.pdf
DE PHAT TRIEN THEO CAU TRUC DE MINH HOA 2022 MON TIENG ANH DE 16 20.pdf
DE PHAT TRIEN THEO CAU TRUC DE MINH HOA 2022 MON TIENG ANH DE 16 20.pdf
DE PHAT TRIEN THEO CAU TRUC DE MINH HOA 2022 MON TIENG ANH DE 16 20.pdf
DE PHAT TRIEN THEO CAU TRUC DE MINH HOA 2022 MON TIENG ANH DE 16 20.pdf
DE PHAT TRIEN THEO CAU TRUC DE MINH HOA 2022 MON TIENG ANH DE 16 20.pdf
DE PHAT TRIEN THEO CAU TRUC DE MINH HOA 2022 MON TIENG ANH DE 16 20.pdf
DE PHAT TRIEN THEO CAU TRUC DE MINH HOA 2022 MON TIENG ANH DE 16 20.pdf
DE PHAT TRIEN THEO CAU TRUC DE MINH HOA 2022 MON TIENG ANH DE 16 20.pdf
DE PHAT TRIEN THEO CAU TRUC DE MINH HOA 2022 MON TIENG ANH DE 16 20.pdf
DE PHAT TRIEN THEO CAU TRUC DE MINH HOA 2022 MON TIENG ANH DE 16 20.pdf
DE PHAT TRIEN THEO CAU TRUC DE MINH HOA 2022 MON TIENG ANH DE 16 20.pdf
DE PHAT TRIEN THEO CAU TRUC DE MINH HOA 2022 MON TIENG ANH DE 16 20.pdf
DE PHAT TRIEN THEO CAU TRUC DE MINH HOA 2022 MON TIENG ANH DE 16 20.pdf
DE PHAT TRIEN THEO CAU TRUC DE MINH HOA 2022 MON TIENG ANH DE 16 20.pdf
DE PHAT TRIEN THEO CAU TRUC DE MINH HOA 2022 MON TIENG ANH DE 16 20.pdf
DE PHAT TRIEN THEO CAU TRUC DE MINH HOA 2022 MON TIENG ANH DE 16 20.pdf
DE PHAT TRIEN THEO CAU TRUC DE MINH HOA 2022 MON TIENG ANH DE 16 20.pdf
DE PHAT TRIEN THEO CAU TRUC DE MINH HOA 2022 MON TIENG ANH DE 16 20.pdf
DE PHAT TRIEN THEO CAU TRUC DE MINH HOA 2022 MON TIENG ANH DE 16 20.pdf
DE PHAT TRIEN THEO CAU TRUC DE MINH HOA 2022 MON TIENG ANH DE 16 20.pdf
DE PHAT TRIEN THEO CAU TRUC DE MINH HOA 2022 MON TIENG ANH DE 16 20.pdf
DE PHAT TRIEN THEO CAU TRUC DE MINH HOA 2022 MON TIENG ANH DE 16 20.pdf
DE PHAT TRIEN THEO CAU TRUC DE MINH HOA 2022 MON TIENG ANH DE 16 20.pdf
DE PHAT TRIEN THEO CAU TRUC DE MINH HOA 2022 MON TIENG ANH DE 16 20.pdf
DE PHAT TRIEN THEO CAU TRUC DE MINH HOA 2022 MON TIENG ANH DE 16 20.pdf
DE PHAT TRIEN THEO CAU TRUC DE MINH HOA 2022 MON TIENG ANH DE 16 20.pdf
DE PHAT TRIEN THEO CAU TRUC DE MINH HOA 2022 MON TIENG ANH DE 16 20.pdf
DE PHAT TRIEN THEO CAU TRUC DE MINH HOA 2022 MON TIENG ANH DE 16 20.pdf
DE PHAT TRIEN THEO CAU TRUC DE MINH HOA 2022 MON TIENG ANH DE 16 20.pdf
DE PHAT TRIEN THEO CAU TRUC DE MINH HOA 2022 MON TIENG ANH DE 16 20.pdf
DE PHAT TRIEN THEO CAU TRUC DE MINH HOA 2022 MON TIENG ANH DE 16 20.pdf
DE PHAT TRIEN THEO CAU TRUC DE MINH HOA 2022 MON TIENG ANH DE 16 20.pdf
DE PHAT TRIEN THEO CAU TRUC DE MINH HOA 2022 MON TIENG ANH DE 16 20.pdf
DE PHAT TRIEN THEO CAU TRUC DE MINH HOA 2022 MON TIENG ANH DE 16 20.pdf
DE PHAT TRIEN THEO CAU TRUC DE MINH HOA 2022 MON TIENG ANH DE 16 20.pdf
DE PHAT TRIEN THEO CAU TRUC DE MINH HOA 2022 MON TIENG ANH DE 16 20.pdf

More Related Content

What's hot

50 ĐỀ PHÁT TRIỂN THEO CẤU TRÚC ĐỀ MINH HỌA BGD NGÀY 22-3-2024 KỲ THI TỐT NGHI...
50 ĐỀ PHÁT TRIỂN THEO CẤU TRÚC ĐỀ MINH HỌA BGD NGÀY 22-3-2024 KỲ THI TỐT NGHI...50 ĐỀ PHÁT TRIỂN THEO CẤU TRÚC ĐỀ MINH HỌA BGD NGÀY 22-3-2024 KỲ THI TỐT NGHI...
50 ĐỀ PHÁT TRIỂN THEO CẤU TRÚC ĐỀ MINH HỌA BGD NGÀY 22-3-2024 KỲ THI TỐT NGHI...Nguyen Thanh Tu Collection
 
Đề Tiếng Anh 12 cơ bản unit 12 có đáp án - VipLam.Net
Đề Tiếng Anh 12 cơ bản unit 12 có đáp án - VipLam.NetĐề Tiếng Anh 12 cơ bản unit 12 có đáp án - VipLam.Net
Đề Tiếng Anh 12 cơ bản unit 12 có đáp án - VipLam.NetThùy Linh
 
Bài tập tiếng anh lớp 7 (phát âm)
Bài tập tiếng anh lớp 7 (phát âm)Bài tập tiếng anh lớp 7 (phát âm)
Bài tập tiếng anh lớp 7 (phát âm)Học Tập Long An
 
Giải chi tiết đề tiếng anh THPTQG 2018 mã 403
Giải chi tiết đề tiếng anh THPTQG 2018 mã 403Giải chi tiết đề tiếng anh THPTQG 2018 mã 403
Giải chi tiết đề tiếng anh THPTQG 2018 mã 403Viet Le
 
Bai giai chi tiet mon anh tn thpt 2013
Bai giai chi tiet mon anh tn thpt 2013Bai giai chi tiet mon anh tn thpt 2013
Bai giai chi tiet mon anh tn thpt 2013Tommy Bảo
 
191 câu trắc nghiệm chương 1, 2 - Sinh học 12
191 câu trắc nghiệm chương 1, 2 - Sinh học 12191 câu trắc nghiệm chương 1, 2 - Sinh học 12
191 câu trắc nghiệm chương 1, 2 - Sinh học 12Van-Duyet Le
 
Bai tap-tieng-anh-12unit-10
Bai tap-tieng-anh-12unit-10Bai tap-tieng-anh-12unit-10
Bai tap-tieng-anh-12unit-10Huyen Tran
 
BỘ ĐỀ DỰ ĐOÁN - PHÁT TRIỂN ĐỀ MINH HỌA BGD KỲ THI TỐT NGHIỆP THPT NĂM 2023 MÔ...
BỘ ĐỀ DỰ ĐOÁN - PHÁT TRIỂN ĐỀ MINH HỌA BGD KỲ THI TỐT NGHIỆP THPT NĂM 2023 MÔ...BỘ ĐỀ DỰ ĐOÁN - PHÁT TRIỂN ĐỀ MINH HỌA BGD KỲ THI TỐT NGHIỆP THPT NĂM 2023 MÔ...
BỘ ĐỀ DỰ ĐOÁN - PHÁT TRIỂN ĐỀ MINH HỌA BGD KỲ THI TỐT NGHIỆP THPT NĂM 2023 MÔ...Nguyen Thanh Tu Collection
 
50 ĐỀ PHÁT TRIỂN THEO CẤU TRÚC ĐỀ MINH HỌA BGD NGÀY 22-3-2024 KỲ THI TỐT NGHI...
50 ĐỀ PHÁT TRIỂN THEO CẤU TRÚC ĐỀ MINH HỌA BGD NGÀY 22-3-2024 KỲ THI TỐT NGHI...50 ĐỀ PHÁT TRIỂN THEO CẤU TRÚC ĐỀ MINH HỌA BGD NGÀY 22-3-2024 KỲ THI TỐT NGHI...
50 ĐỀ PHÁT TRIỂN THEO CẤU TRÚC ĐỀ MINH HỌA BGD NGÀY 22-3-2024 KỲ THI TỐT NGHI...Nguyen Thanh Tu Collection
 
50 ĐỀ THI THỬ TỐT NGHIỆP THPT TIẾNG ANH 2024 CÓ GIẢI CHI TIẾT - GIỚI HẠN KHO...
50 ĐỀ THI THỬ TỐT NGHIỆP THPT TIẾNG ANH 2024 CÓ GIẢI CHI TIẾT - GIỚI HẠN KHO...50 ĐỀ THI THỬ TỐT NGHIỆP THPT TIẾNG ANH 2024 CÓ GIẢI CHI TIẾT - GIỚI HẠN KHO...
50 ĐỀ THI THỬ TỐT NGHIỆP THPT TIẾNG ANH 2024 CÓ GIẢI CHI TIẾT - GIỚI HẠN KHO...Nguyen Thanh Tu Collection
 
Trắc nghiệm từng unit tiếng Anh 12 (16 unit) + key
Trắc nghiệm từng unit tiếng Anh 12 (16 unit) + keyTrắc nghiệm từng unit tiếng Anh 12 (16 unit) + key
Trắc nghiệm từng unit tiếng Anh 12 (16 unit) + keyThùy Linh
 
20 ĐỀ DỰ ĐOÁN - PHÁT TRIỂN ĐỀ MINH HỌA BGD KỲ THI TỐT NGHIỆP THPT NĂM 2023 MÔ...
20 ĐỀ DỰ ĐOÁN - PHÁT TRIỂN ĐỀ MINH HỌA BGD KỲ THI TỐT NGHIỆP THPT NĂM 2023 MÔ...20 ĐỀ DỰ ĐOÁN - PHÁT TRIỂN ĐỀ MINH HỌA BGD KỲ THI TỐT NGHIỆP THPT NĂM 2023 MÔ...
20 ĐỀ DỰ ĐOÁN - PHÁT TRIỂN ĐỀ MINH HỌA BGD KỲ THI TỐT NGHIỆP THPT NĂM 2023 MÔ...Nguyen Thanh Tu Collection
 
NGỮ PHÁP VÀ BÀI TẬP ÔN 10, ÔN TỐT NGHIỆP THPT MÔN TIẾNG ANH (22 CHUYÊN ĐỀ) (...
NGỮ PHÁP VÀ BÀI TẬP ÔN 10, ÔN TỐT NGHIỆP THPT MÔN TIẾNG ANH (22 CHUYÊN ĐỀ) (...NGỮ PHÁP VÀ BÀI TẬP ÔN 10, ÔN TỐT NGHIỆP THPT MÔN TIẾNG ANH (22 CHUYÊN ĐỀ) (...
NGỮ PHÁP VÀ BÀI TẬP ÔN 10, ÔN TỐT NGHIỆP THPT MÔN TIẾNG ANH (22 CHUYÊN ĐỀ) (...Nguyen Thanh Tu Collection
 
BÀI TẬP BỔ TRỢ 4 KĨ NĂNG TIẾNG ANH LỚP 11 - CẢ NĂM - GLOBAL SUCCESS - NĂM HỌC...
BÀI TẬP BỔ TRỢ 4 KĨ NĂNG TIẾNG ANH LỚP 11 - CẢ NĂM - GLOBAL SUCCESS - NĂM HỌC...BÀI TẬP BỔ TRỢ 4 KĨ NĂNG TIẾNG ANH LỚP 11 - CẢ NĂM - GLOBAL SUCCESS - NĂM HỌC...
BÀI TẬP BỔ TRỢ 4 KĨ NĂNG TIẾNG ANH LỚP 11 - CẢ NĂM - GLOBAL SUCCESS - NĂM HỌC...Nguyen Thanh Tu Collection
 
Hệ thống kiến thức hình học THCS (cấp 2)
Hệ thống kiến thức hình học THCS (cấp 2)Hệ thống kiến thức hình học THCS (cấp 2)
Hệ thống kiến thức hình học THCS (cấp 2)Lớp học thầy Tài
 
TỔNG HỢP HƠN 100 ĐỀ THI THỬ TỐT NGHIỆP THPT TIẾNG ANH 2024 - TỪ CÁC TRƯỜNG, ...
TỔNG HỢP HƠN 100 ĐỀ THI THỬ TỐT NGHIỆP THPT TIẾNG ANH 2024 - TỪ CÁC TRƯỜNG, ...TỔNG HỢP HƠN 100 ĐỀ THI THỬ TỐT NGHIỆP THPT TIẾNG ANH 2024 - TỪ CÁC TRƯỜNG, ...
TỔNG HỢP HƠN 100 ĐỀ THI THỬ TỐT NGHIỆP THPT TIẾNG ANH 2024 - TỪ CÁC TRƯỜNG, ...Nguyen Thanh Tu Collection
 
TỔNG HỢP ĐỀ THI ĐỀ NGHỊ KỲ THI CHỌN HỌC SINH GIỎI CÁC TRƯỜNG THPT CHUYÊN KHU ...
TỔNG HỢP ĐỀ THI ĐỀ NGHỊ KỲ THI CHỌN HỌC SINH GIỎI CÁC TRƯỜNG THPT CHUYÊN KHU ...TỔNG HỢP ĐỀ THI ĐỀ NGHỊ KỲ THI CHỌN HỌC SINH GIỎI CÁC TRƯỜNG THPT CHUYÊN KHU ...
TỔNG HỢP ĐỀ THI ĐỀ NGHỊ KỲ THI CHỌN HỌC SINH GIỎI CÁC TRƯỜNG THPT CHUYÊN KHU ...Nguyen Thanh Tu Collection
 
Nhung cong thuc luong giac co ban
Nhung cong thuc luong giac co banNhung cong thuc luong giac co ban
Nhung cong thuc luong giac co banNguyễn Hoành
 
Đề thi trắc nghiệm Xác suất thống kê có lời giải
Đề thi trắc nghiệm Xác suất thống kê có lời giảiĐề thi trắc nghiệm Xác suất thống kê có lời giải
Đề thi trắc nghiệm Xác suất thống kê có lời giải希夢 坂井
 
Công thức vật lý lớp 11
Công thức vật lý lớp 11Công thức vật lý lớp 11
Công thức vật lý lớp 11Vô Ngã
 

What's hot (20)

50 ĐỀ PHÁT TRIỂN THEO CẤU TRÚC ĐỀ MINH HỌA BGD NGÀY 22-3-2024 KỲ THI TỐT NGHI...
50 ĐỀ PHÁT TRIỂN THEO CẤU TRÚC ĐỀ MINH HỌA BGD NGÀY 22-3-2024 KỲ THI TỐT NGHI...50 ĐỀ PHÁT TRIỂN THEO CẤU TRÚC ĐỀ MINH HỌA BGD NGÀY 22-3-2024 KỲ THI TỐT NGHI...
50 ĐỀ PHÁT TRIỂN THEO CẤU TRÚC ĐỀ MINH HỌA BGD NGÀY 22-3-2024 KỲ THI TỐT NGHI...
 
Đề Tiếng Anh 12 cơ bản unit 12 có đáp án - VipLam.Net
Đề Tiếng Anh 12 cơ bản unit 12 có đáp án - VipLam.NetĐề Tiếng Anh 12 cơ bản unit 12 có đáp án - VipLam.Net
Đề Tiếng Anh 12 cơ bản unit 12 có đáp án - VipLam.Net
 
Bài tập tiếng anh lớp 7 (phát âm)
Bài tập tiếng anh lớp 7 (phát âm)Bài tập tiếng anh lớp 7 (phát âm)
Bài tập tiếng anh lớp 7 (phát âm)
 
Giải chi tiết đề tiếng anh THPTQG 2018 mã 403
Giải chi tiết đề tiếng anh THPTQG 2018 mã 403Giải chi tiết đề tiếng anh THPTQG 2018 mã 403
Giải chi tiết đề tiếng anh THPTQG 2018 mã 403
 
Bai giai chi tiet mon anh tn thpt 2013
Bai giai chi tiet mon anh tn thpt 2013Bai giai chi tiet mon anh tn thpt 2013
Bai giai chi tiet mon anh tn thpt 2013
 
191 câu trắc nghiệm chương 1, 2 - Sinh học 12
191 câu trắc nghiệm chương 1, 2 - Sinh học 12191 câu trắc nghiệm chương 1, 2 - Sinh học 12
191 câu trắc nghiệm chương 1, 2 - Sinh học 12
 
Bai tap-tieng-anh-12unit-10
Bai tap-tieng-anh-12unit-10Bai tap-tieng-anh-12unit-10
Bai tap-tieng-anh-12unit-10
 
BỘ ĐỀ DỰ ĐOÁN - PHÁT TRIỂN ĐỀ MINH HỌA BGD KỲ THI TỐT NGHIỆP THPT NĂM 2023 MÔ...
BỘ ĐỀ DỰ ĐOÁN - PHÁT TRIỂN ĐỀ MINH HỌA BGD KỲ THI TỐT NGHIỆP THPT NĂM 2023 MÔ...BỘ ĐỀ DỰ ĐOÁN - PHÁT TRIỂN ĐỀ MINH HỌA BGD KỲ THI TỐT NGHIỆP THPT NĂM 2023 MÔ...
BỘ ĐỀ DỰ ĐOÁN - PHÁT TRIỂN ĐỀ MINH HỌA BGD KỲ THI TỐT NGHIỆP THPT NĂM 2023 MÔ...
 
50 ĐỀ PHÁT TRIỂN THEO CẤU TRÚC ĐỀ MINH HỌA BGD NGÀY 22-3-2024 KỲ THI TỐT NGHI...
50 ĐỀ PHÁT TRIỂN THEO CẤU TRÚC ĐỀ MINH HỌA BGD NGÀY 22-3-2024 KỲ THI TỐT NGHI...50 ĐỀ PHÁT TRIỂN THEO CẤU TRÚC ĐỀ MINH HỌA BGD NGÀY 22-3-2024 KỲ THI TỐT NGHI...
50 ĐỀ PHÁT TRIỂN THEO CẤU TRÚC ĐỀ MINH HỌA BGD NGÀY 22-3-2024 KỲ THI TỐT NGHI...
 
50 ĐỀ THI THỬ TỐT NGHIỆP THPT TIẾNG ANH 2024 CÓ GIẢI CHI TIẾT - GIỚI HẠN KHO...
50 ĐỀ THI THỬ TỐT NGHIỆP THPT TIẾNG ANH 2024 CÓ GIẢI CHI TIẾT - GIỚI HẠN KHO...50 ĐỀ THI THỬ TỐT NGHIỆP THPT TIẾNG ANH 2024 CÓ GIẢI CHI TIẾT - GIỚI HẠN KHO...
50 ĐỀ THI THỬ TỐT NGHIỆP THPT TIẾNG ANH 2024 CÓ GIẢI CHI TIẾT - GIỚI HẠN KHO...
 
Trắc nghiệm từng unit tiếng Anh 12 (16 unit) + key
Trắc nghiệm từng unit tiếng Anh 12 (16 unit) + keyTrắc nghiệm từng unit tiếng Anh 12 (16 unit) + key
Trắc nghiệm từng unit tiếng Anh 12 (16 unit) + key
 
20 ĐỀ DỰ ĐOÁN - PHÁT TRIỂN ĐỀ MINH HỌA BGD KỲ THI TỐT NGHIỆP THPT NĂM 2023 MÔ...
20 ĐỀ DỰ ĐOÁN - PHÁT TRIỂN ĐỀ MINH HỌA BGD KỲ THI TỐT NGHIỆP THPT NĂM 2023 MÔ...20 ĐỀ DỰ ĐOÁN - PHÁT TRIỂN ĐỀ MINH HỌA BGD KỲ THI TỐT NGHIỆP THPT NĂM 2023 MÔ...
20 ĐỀ DỰ ĐOÁN - PHÁT TRIỂN ĐỀ MINH HỌA BGD KỲ THI TỐT NGHIỆP THPT NĂM 2023 MÔ...
 
NGỮ PHÁP VÀ BÀI TẬP ÔN 10, ÔN TỐT NGHIỆP THPT MÔN TIẾNG ANH (22 CHUYÊN ĐỀ) (...
NGỮ PHÁP VÀ BÀI TẬP ÔN 10, ÔN TỐT NGHIỆP THPT MÔN TIẾNG ANH (22 CHUYÊN ĐỀ) (...NGỮ PHÁP VÀ BÀI TẬP ÔN 10, ÔN TỐT NGHIỆP THPT MÔN TIẾNG ANH (22 CHUYÊN ĐỀ) (...
NGỮ PHÁP VÀ BÀI TẬP ÔN 10, ÔN TỐT NGHIỆP THPT MÔN TIẾNG ANH (22 CHUYÊN ĐỀ) (...
 
BÀI TẬP BỔ TRỢ 4 KĨ NĂNG TIẾNG ANH LỚP 11 - CẢ NĂM - GLOBAL SUCCESS - NĂM HỌC...
BÀI TẬP BỔ TRỢ 4 KĨ NĂNG TIẾNG ANH LỚP 11 - CẢ NĂM - GLOBAL SUCCESS - NĂM HỌC...BÀI TẬP BỔ TRỢ 4 KĨ NĂNG TIẾNG ANH LỚP 11 - CẢ NĂM - GLOBAL SUCCESS - NĂM HỌC...
BÀI TẬP BỔ TRỢ 4 KĨ NĂNG TIẾNG ANH LỚP 11 - CẢ NĂM - GLOBAL SUCCESS - NĂM HỌC...
 
Hệ thống kiến thức hình học THCS (cấp 2)
Hệ thống kiến thức hình học THCS (cấp 2)Hệ thống kiến thức hình học THCS (cấp 2)
Hệ thống kiến thức hình học THCS (cấp 2)
 
TỔNG HỢP HƠN 100 ĐỀ THI THỬ TỐT NGHIỆP THPT TIẾNG ANH 2024 - TỪ CÁC TRƯỜNG, ...
TỔNG HỢP HƠN 100 ĐỀ THI THỬ TỐT NGHIỆP THPT TIẾNG ANH 2024 - TỪ CÁC TRƯỜNG, ...TỔNG HỢP HƠN 100 ĐỀ THI THỬ TỐT NGHIỆP THPT TIẾNG ANH 2024 - TỪ CÁC TRƯỜNG, ...
TỔNG HỢP HƠN 100 ĐỀ THI THỬ TỐT NGHIỆP THPT TIẾNG ANH 2024 - TỪ CÁC TRƯỜNG, ...
 
TỔNG HỢP ĐỀ THI ĐỀ NGHỊ KỲ THI CHỌN HỌC SINH GIỎI CÁC TRƯỜNG THPT CHUYÊN KHU ...
TỔNG HỢP ĐỀ THI ĐỀ NGHỊ KỲ THI CHỌN HỌC SINH GIỎI CÁC TRƯỜNG THPT CHUYÊN KHU ...TỔNG HỢP ĐỀ THI ĐỀ NGHỊ KỲ THI CHỌN HỌC SINH GIỎI CÁC TRƯỜNG THPT CHUYÊN KHU ...
TỔNG HỢP ĐỀ THI ĐỀ NGHỊ KỲ THI CHỌN HỌC SINH GIỎI CÁC TRƯỜNG THPT CHUYÊN KHU ...
 
Nhung cong thuc luong giac co ban
Nhung cong thuc luong giac co banNhung cong thuc luong giac co ban
Nhung cong thuc luong giac co ban
 
Đề thi trắc nghiệm Xác suất thống kê có lời giải
Đề thi trắc nghiệm Xác suất thống kê có lời giảiĐề thi trắc nghiệm Xác suất thống kê có lời giải
Đề thi trắc nghiệm Xác suất thống kê có lời giải
 
Công thức vật lý lớp 11
Công thức vật lý lớp 11Công thức vật lý lớp 11
Công thức vật lý lớp 11
 

Similar to DE PHAT TRIEN THEO CAU TRUC DE MINH HOA 2022 MON TIENG ANH DE 16 20.pdf

đáP án và giải thích đề 20
đáP án và giải thích đề 20đáP án và giải thích đề 20
đáP án và giải thích đề 20Huynh ICT
 
đáP án và giải thích đề 31
đáP án và giải thích đề 31đáP án và giải thích đề 31
đáP án và giải thích đề 31Huynh ICT
 
đáP án và giải thích đề 25
đáP án và giải thích đề 25đáP án và giải thích đề 25
đáP án và giải thích đề 25Huynh ICT
 
đáP án và giải thích đề 6
đáP án và giải thích đề 6đáP án và giải thích đề 6
đáP án và giải thích đề 6Huynh ICT
 
đáP án và giải thích đề 22
đáP án và giải thích đề 22đáP án và giải thích đề 22
đáP án và giải thích đề 22Huynh ICT
 
đáP án và giải thích đề 8
đáP án và giải thích đề 8đáP án và giải thích đề 8
đáP án và giải thích đề 8Huynh ICT
 
đáP án và giải thích đề 9
đáP án và giải thích đề 9đáP án và giải thích đề 9
đáP án và giải thích đề 9Huynh ICT
 
đáP án và giải thích đề 4
đáP án và giải thích đề 4đáP án và giải thích đề 4
đáP án và giải thích đề 4Huynh ICT
 
đáP án và giải thích đề 23
đáP án và giải thích đề 23đáP án và giải thích đề 23
đáP án và giải thích đề 23Huynh ICT
 
đáP án và giải thích đề 17
đáP án và giải thích đề 17đáP án và giải thích đề 17
đáP án và giải thích đề 17Huynh ICT
 
đáP án và giải thích đề 24
đáP án và giải thích đề 24đáP án và giải thích đề 24
đáP án và giải thích đề 24Huynh ICT
 
Đề thi thử Ôn thi Tiếng Anh vào Cao Đẳng , Đại học năm 2013 - Đề 110
Đề thi thử  Ôn thi Tiếng Anh vào Cao Đẳng , Đại học năm 2013 - Đề 110Đề thi thử  Ôn thi Tiếng Anh vào Cao Đẳng , Đại học năm 2013 - Đề 110
Đề thi thử Ôn thi Tiếng Anh vào Cao Đẳng , Đại học năm 2013 - Đề 110phamnhakb
 
đáP án và giải thích đề 15
đáP án và giải thích đề 15đáP án và giải thích đề 15
đáP án và giải thích đề 15Huynh ICT
 
đáP án và giải thích đề 10
đáP án và giải thích đề 10đáP án và giải thích đề 10
đáP án và giải thích đề 10Huynh ICT
 
đáP án và giải thích đề 16
đáP án và giải thích đề 16đáP án và giải thích đề 16
đáP án và giải thích đề 16Huynh ICT
 
đáP án và giải thích đề 14
đáP án và giải thích đề 14đáP án và giải thích đề 14
đáP án và giải thích đề 14Huynh ICT
 
đáP án và giải thích đề 32
đáP án và giải thích đề 32đáP án và giải thích đề 32
đáP án và giải thích đề 32Huynh ICT
 
Giai chi tiet de thi dh khoi d t anh 2011
Giai chi tiet de thi dh khoi d t anh 2011Giai chi tiet de thi dh khoi d t anh 2011
Giai chi tiet de thi dh khoi d t anh 2011Hua Tran Phuong Thao
 
đáP án và giải thích đề 13
đáP án và giải thích đề 13đáP án và giải thích đề 13
đáP án và giải thích đề 13Huynh ICT
 
đáP án và giải thích đề 5
đáP án và giải thích đề 5đáP án và giải thích đề 5
đáP án và giải thích đề 5Huynh ICT
 

Similar to DE PHAT TRIEN THEO CAU TRUC DE MINH HOA 2022 MON TIENG ANH DE 16 20.pdf (20)

đáP án và giải thích đề 20
đáP án và giải thích đề 20đáP án và giải thích đề 20
đáP án và giải thích đề 20
 
đáP án và giải thích đề 31
đáP án và giải thích đề 31đáP án và giải thích đề 31
đáP án và giải thích đề 31
 
đáP án và giải thích đề 25
đáP án và giải thích đề 25đáP án và giải thích đề 25
đáP án và giải thích đề 25
 
đáP án và giải thích đề 6
đáP án và giải thích đề 6đáP án và giải thích đề 6
đáP án và giải thích đề 6
 
đáP án và giải thích đề 22
đáP án và giải thích đề 22đáP án và giải thích đề 22
đáP án và giải thích đề 22
 
đáP án và giải thích đề 8
đáP án và giải thích đề 8đáP án và giải thích đề 8
đáP án và giải thích đề 8
 
đáP án và giải thích đề 9
đáP án và giải thích đề 9đáP án và giải thích đề 9
đáP án và giải thích đề 9
 
đáP án và giải thích đề 4
đáP án và giải thích đề 4đáP án và giải thích đề 4
đáP án và giải thích đề 4
 
đáP án và giải thích đề 23
đáP án và giải thích đề 23đáP án và giải thích đề 23
đáP án và giải thích đề 23
 
đáP án và giải thích đề 17
đáP án và giải thích đề 17đáP án và giải thích đề 17
đáP án và giải thích đề 17
 
đáP án và giải thích đề 24
đáP án và giải thích đề 24đáP án và giải thích đề 24
đáP án và giải thích đề 24
 
Đề thi thử Ôn thi Tiếng Anh vào Cao Đẳng , Đại học năm 2013 - Đề 110
Đề thi thử  Ôn thi Tiếng Anh vào Cao Đẳng , Đại học năm 2013 - Đề 110Đề thi thử  Ôn thi Tiếng Anh vào Cao Đẳng , Đại học năm 2013 - Đề 110
Đề thi thử Ôn thi Tiếng Anh vào Cao Đẳng , Đại học năm 2013 - Đề 110
 
đáP án và giải thích đề 15
đáP án và giải thích đề 15đáP án và giải thích đề 15
đáP án và giải thích đề 15
 
đáP án và giải thích đề 10
đáP án và giải thích đề 10đáP án và giải thích đề 10
đáP án và giải thích đề 10
 
đáP án và giải thích đề 16
đáP án và giải thích đề 16đáP án và giải thích đề 16
đáP án và giải thích đề 16
 
đáP án và giải thích đề 14
đáP án và giải thích đề 14đáP án và giải thích đề 14
đáP án và giải thích đề 14
 
đáP án và giải thích đề 32
đáP án và giải thích đề 32đáP án và giải thích đề 32
đáP án và giải thích đề 32
 
Giai chi tiet de thi dh khoi d t anh 2011
Giai chi tiet de thi dh khoi d t anh 2011Giai chi tiet de thi dh khoi d t anh 2011
Giai chi tiet de thi dh khoi d t anh 2011
 
đáP án và giải thích đề 13
đáP án và giải thích đề 13đáP án và giải thích đề 13
đáP án và giải thích đề 13
 
đáP án và giải thích đề 5
đáP án và giải thích đề 5đáP án và giải thích đề 5
đáP án và giải thích đề 5
 

More from Nguyen Thanh Tu Collection

30 ĐỀ PHÁT TRIỂN THEO CẤU TRÚC ĐỀ MINH HỌA BGD NGÀY 22-3-2024 KỲ THI TỐT NGHI...
30 ĐỀ PHÁT TRIỂN THEO CẤU TRÚC ĐỀ MINH HỌA BGD NGÀY 22-3-2024 KỲ THI TỐT NGHI...30 ĐỀ PHÁT TRIỂN THEO CẤU TRÚC ĐỀ MINH HỌA BGD NGÀY 22-3-2024 KỲ THI TỐT NGHI...
30 ĐỀ PHÁT TRIỂN THEO CẤU TRÚC ĐỀ MINH HỌA BGD NGÀY 22-3-2024 KỲ THI TỐT NGHI...Nguyen Thanh Tu Collection
 
30 ĐỀ PHÁT TRIỂN THEO CẤU TRÚC ĐỀ MINH HỌA BGD NGÀY 22-3-2024 KỲ THI TỐT NGHI...
30 ĐỀ PHÁT TRIỂN THEO CẤU TRÚC ĐỀ MINH HỌA BGD NGÀY 22-3-2024 KỲ THI TỐT NGHI...30 ĐỀ PHÁT TRIỂN THEO CẤU TRÚC ĐỀ MINH HỌA BGD NGÀY 22-3-2024 KỲ THI TỐT NGHI...
30 ĐỀ PHÁT TRIỂN THEO CẤU TRÚC ĐỀ MINH HỌA BGD NGÀY 22-3-2024 KỲ THI TỐT NGHI...Nguyen Thanh Tu Collection
 
30 ĐỀ PHÁT TRIỂN THEO CẤU TRÚC ĐỀ MINH HỌA BGD NGÀY 22-3-2024 KỲ THI TỐT NGHI...
30 ĐỀ PHÁT TRIỂN THEO CẤU TRÚC ĐỀ MINH HỌA BGD NGÀY 22-3-2024 KỲ THI TỐT NGHI...30 ĐỀ PHÁT TRIỂN THEO CẤU TRÚC ĐỀ MINH HỌA BGD NGÀY 22-3-2024 KỲ THI TỐT NGHI...
30 ĐỀ PHÁT TRIỂN THEO CẤU TRÚC ĐỀ MINH HỌA BGD NGÀY 22-3-2024 KỲ THI TỐT NGHI...Nguyen Thanh Tu Collection
 
TỔNG HỢP ĐỀ THI CHÍNH THỨC KỲ THI TUYỂN SINH VÀO LỚP 10 THPT MÔN NGỮ VĂN NĂM ...
TỔNG HỢP ĐỀ THI CHÍNH THỨC KỲ THI TUYỂN SINH VÀO LỚP 10 THPT MÔN NGỮ VĂN NĂM ...TỔNG HỢP ĐỀ THI CHÍNH THỨC KỲ THI TUYỂN SINH VÀO LỚP 10 THPT MÔN NGỮ VĂN NĂM ...
TỔNG HỢP ĐỀ THI CHÍNH THỨC KỲ THI TUYỂN SINH VÀO LỚP 10 THPT MÔN NGỮ VĂN NĂM ...Nguyen Thanh Tu Collection
 
TUYỂN TẬP 20 ĐỀ THI KHẢO SÁT HỌC SINH GIỎI MÔN TIẾNG ANH LỚP 6 NĂM 2020 (CÓ Đ...
TUYỂN TẬP 20 ĐỀ THI KHẢO SÁT HỌC SINH GIỎI MÔN TIẾNG ANH LỚP 6 NĂM 2020 (CÓ Đ...TUYỂN TẬP 20 ĐỀ THI KHẢO SÁT HỌC SINH GIỎI MÔN TIẾNG ANH LỚP 6 NĂM 2020 (CÓ Đ...
TUYỂN TẬP 20 ĐỀ THI KHẢO SÁT HỌC SINH GIỎI MÔN TIẾNG ANH LỚP 6 NĂM 2020 (CÓ Đ...Nguyen Thanh Tu Collection
 
TUYỂN TẬP 25 ĐỀ THI HỌC SINH GIỎI MÔN TIẾNG ANH LỚP 6 NĂM 2023 CÓ ĐÁP ÁN (SƯU...
TUYỂN TẬP 25 ĐỀ THI HỌC SINH GIỎI MÔN TIẾNG ANH LỚP 6 NĂM 2023 CÓ ĐÁP ÁN (SƯU...TUYỂN TẬP 25 ĐỀ THI HỌC SINH GIỎI MÔN TIẾNG ANH LỚP 6 NĂM 2023 CÓ ĐÁP ÁN (SƯU...
TUYỂN TẬP 25 ĐỀ THI HỌC SINH GIỎI MÔN TIẾNG ANH LỚP 6 NĂM 2023 CÓ ĐÁP ÁN (SƯU...Nguyen Thanh Tu Collection
 
BỘ ĐỀ PHÁT TRIỂN THEO CẤU TRÚC ĐỀ MINH HỌA BGD NGÀY 22-3-2024 KỲ THI TỐT NGHI...
BỘ ĐỀ PHÁT TRIỂN THEO CẤU TRÚC ĐỀ MINH HỌA BGD NGÀY 22-3-2024 KỲ THI TỐT NGHI...BỘ ĐỀ PHÁT TRIỂN THEO CẤU TRÚC ĐỀ MINH HỌA BGD NGÀY 22-3-2024 KỲ THI TỐT NGHI...
BỘ ĐỀ PHÁT TRIỂN THEO CẤU TRÚC ĐỀ MINH HỌA BGD NGÀY 22-3-2024 KỲ THI TỐT NGHI...Nguyen Thanh Tu Collection
 
Sáng kiến Dạy học theo định hướng STEM một số chủ đề phần “vật sống”, Khoa họ...
Sáng kiến Dạy học theo định hướng STEM một số chủ đề phần “vật sống”, Khoa họ...Sáng kiến Dạy học theo định hướng STEM một số chủ đề phần “vật sống”, Khoa họ...
Sáng kiến Dạy học theo định hướng STEM một số chủ đề phần “vật sống”, Khoa họ...Nguyen Thanh Tu Collection
 
Sáng kiến “Sử dụng ứng dụng Quizizz nhằm nâng cao chất lượng ôn thi tốt nghiệ...
Sáng kiến “Sử dụng ứng dụng Quizizz nhằm nâng cao chất lượng ôn thi tốt nghiệ...Sáng kiến “Sử dụng ứng dụng Quizizz nhằm nâng cao chất lượng ôn thi tốt nghiệ...
Sáng kiến “Sử dụng ứng dụng Quizizz nhằm nâng cao chất lượng ôn thi tốt nghiệ...Nguyen Thanh Tu Collection
 
30 ĐỀ PHÁT TRIỂN THEO CẤU TRÚC ĐỀ MINH HỌA BGD NGÀY 22-3-2024 KỲ THI TỐT NGHI...
30 ĐỀ PHÁT TRIỂN THEO CẤU TRÚC ĐỀ MINH HỌA BGD NGÀY 22-3-2024 KỲ THI TỐT NGHI...30 ĐỀ PHÁT TRIỂN THEO CẤU TRÚC ĐỀ MINH HỌA BGD NGÀY 22-3-2024 KỲ THI TỐT NGHI...
30 ĐỀ PHÁT TRIỂN THEO CẤU TRÚC ĐỀ MINH HỌA BGD NGÀY 22-3-2024 KỲ THI TỐT NGHI...Nguyen Thanh Tu Collection
 
10 ĐỀ KIỂM TRA + 6 ĐỀ ÔN TẬP CUỐI KÌ 2 VẬT LÝ 11 - KẾT NỐI TRI THỨC - THEO C...
10 ĐỀ KIỂM TRA + 6 ĐỀ ÔN TẬP CUỐI KÌ 2 VẬT LÝ 11 - KẾT NỐI TRI THỨC - THEO C...10 ĐỀ KIỂM TRA + 6 ĐỀ ÔN TẬP CUỐI KÌ 2 VẬT LÝ 11 - KẾT NỐI TRI THỨC - THEO C...
10 ĐỀ KIỂM TRA + 6 ĐỀ ÔN TẬP CUỐI KÌ 2 VẬT LÝ 11 - KẾT NỐI TRI THỨC - THEO C...Nguyen Thanh Tu Collection
 
BỘ ĐỀ KIỂM TRA CUỐI KÌ 2 VẬT LÝ 11 - KẾT NỐI TRI THỨC - THEO CẤU TRÚC ĐỀ MIN...
BỘ ĐỀ KIỂM TRA CUỐI KÌ 2 VẬT LÝ 11 - KẾT NỐI TRI THỨC - THEO CẤU TRÚC ĐỀ MIN...BỘ ĐỀ KIỂM TRA CUỐI KÌ 2 VẬT LÝ 11 - KẾT NỐI TRI THỨC - THEO CẤU TRÚC ĐỀ MIN...
BỘ ĐỀ KIỂM TRA CUỐI KÌ 2 VẬT LÝ 11 - KẾT NỐI TRI THỨC - THEO CẤU TRÚC ĐỀ MIN...Nguyen Thanh Tu Collection
 
SÁNG KIẾN “THIẾT KẾ VÀ SỬ DỤNG INFOGRAPHIC TRONG DẠY HỌC ĐỊA LÍ 11 (BỘ SÁCH K...
SÁNG KIẾN “THIẾT KẾ VÀ SỬ DỤNG INFOGRAPHIC TRONG DẠY HỌC ĐỊA LÍ 11 (BỘ SÁCH K...SÁNG KIẾN “THIẾT KẾ VÀ SỬ DỤNG INFOGRAPHIC TRONG DẠY HỌC ĐỊA LÍ 11 (BỘ SÁCH K...
SÁNG KIẾN “THIẾT KẾ VÀ SỬ DỤNG INFOGRAPHIC TRONG DẠY HỌC ĐỊA LÍ 11 (BỘ SÁCH K...Nguyen Thanh Tu Collection
 
ĐỀ THAM KHẢO THEO HƯỚNG MINH HỌA 2025 KIỂM TRA CUỐI HỌC KÌ 2 NĂM HỌC 2023-202...
ĐỀ THAM KHẢO THEO HƯỚNG MINH HỌA 2025 KIỂM TRA CUỐI HỌC KÌ 2 NĂM HỌC 2023-202...ĐỀ THAM KHẢO THEO HƯỚNG MINH HỌA 2025 KIỂM TRA CUỐI HỌC KÌ 2 NĂM HỌC 2023-202...
ĐỀ THAM KHẢO THEO HƯỚNG MINH HỌA 2025 KIỂM TRA CUỐI HỌC KÌ 2 NĂM HỌC 2023-202...Nguyen Thanh Tu Collection
 
HỌC TỐT TIẾNG ANH 11 THEO CHƯƠNG TRÌNH GLOBAL SUCCESS ĐÁP ÁN CHI TIẾT - CẢ NĂ...
HỌC TỐT TIẾNG ANH 11 THEO CHƯƠNG TRÌNH GLOBAL SUCCESS ĐÁP ÁN CHI TIẾT - CẢ NĂ...HỌC TỐT TIẾNG ANH 11 THEO CHƯƠNG TRÌNH GLOBAL SUCCESS ĐÁP ÁN CHI TIẾT - CẢ NĂ...
HỌC TỐT TIẾNG ANH 11 THEO CHƯƠNG TRÌNH GLOBAL SUCCESS ĐÁP ÁN CHI TIẾT - CẢ NĂ...Nguyen Thanh Tu Collection
 
50 ĐỀ ĐỀ XUẤT THI VÀO 10 THPT SỞ GIÁO DỤC THANH HÓA MÔN TIẾNG ANH 9 CÓ TỰ LUẬ...
50 ĐỀ ĐỀ XUẤT THI VÀO 10 THPT SỞ GIÁO DỤC THANH HÓA MÔN TIẾNG ANH 9 CÓ TỰ LUẬ...50 ĐỀ ĐỀ XUẤT THI VÀO 10 THPT SỞ GIÁO DỤC THANH HÓA MÔN TIẾNG ANH 9 CÓ TỰ LUẬ...
50 ĐỀ ĐỀ XUẤT THI VÀO 10 THPT SỞ GIÁO DỤC THANH HÓA MÔN TIẾNG ANH 9 CÓ TỰ LUẬ...Nguyen Thanh Tu Collection
 
ĐỀ THAM KHẢO THEO HƯỚNG MINH HỌA 2025 KIỂM TRA GIỮA HỌC KÌ + CUỐI HỌC KÌ 2 NĂ...
ĐỀ THAM KHẢO THEO HƯỚNG MINH HỌA 2025 KIỂM TRA GIỮA HỌC KÌ + CUỐI HỌC KÌ 2 NĂ...ĐỀ THAM KHẢO THEO HƯỚNG MINH HỌA 2025 KIỂM TRA GIỮA HỌC KÌ + CUỐI HỌC KÌ 2 NĂ...
ĐỀ THAM KHẢO THEO HƯỚNG MINH HỌA 2025 KIỂM TRA GIỮA HỌC KÌ + CUỐI HỌC KÌ 2 NĂ...Nguyen Thanh Tu Collection
 
TỔNG HỢP 30 ĐỀ THI CHỌN HSG CÁC TRƯỜNG THPT CHUYÊN VÙNG DUYÊN HẢI & ĐỒNG BẰNG...
TỔNG HỢP 30 ĐỀ THI CHỌN HSG CÁC TRƯỜNG THPT CHUYÊN VÙNG DUYÊN HẢI & ĐỒNG BẰNG...TỔNG HỢP 30 ĐỀ THI CHỌN HSG CÁC TRƯỜNG THPT CHUYÊN VÙNG DUYÊN HẢI & ĐỒNG BẰNG...
TỔNG HỢP 30 ĐỀ THI CHỌN HSG CÁC TRƯỜNG THPT CHUYÊN VÙNG DUYÊN HẢI & ĐỒNG BẰNG...Nguyen Thanh Tu Collection
 
ĐỀ CƯƠNG + TEST ÔN TẬP CUỐI KÌ 2 TIẾNG ANH 11 - GLOBAL SUCCESS (THEO CHUẨN MI...
ĐỀ CƯƠNG + TEST ÔN TẬP CUỐI KÌ 2 TIẾNG ANH 11 - GLOBAL SUCCESS (THEO CHUẨN MI...ĐỀ CƯƠNG + TEST ÔN TẬP CUỐI KÌ 2 TIẾNG ANH 11 - GLOBAL SUCCESS (THEO CHUẨN MI...
ĐỀ CƯƠNG + TEST ÔN TẬP CUỐI KÌ 2 TIẾNG ANH 11 - GLOBAL SUCCESS (THEO CHUẨN MI...Nguyen Thanh Tu Collection
 
VẬN DỤNG KIẾN THỨC LIÊN MÔN TRONG GIẢI BÀI TẬP ÔN THI THPTQG MÔN SINH HỌC - H...
VẬN DỤNG KIẾN THỨC LIÊN MÔN TRONG GIẢI BÀI TẬP ÔN THI THPTQG MÔN SINH HỌC - H...VẬN DỤNG KIẾN THỨC LIÊN MÔN TRONG GIẢI BÀI TẬP ÔN THI THPTQG MÔN SINH HỌC - H...
VẬN DỤNG KIẾN THỨC LIÊN MÔN TRONG GIẢI BÀI TẬP ÔN THI THPTQG MÔN SINH HỌC - H...Nguyen Thanh Tu Collection
 

More from Nguyen Thanh Tu Collection (20)

30 ĐỀ PHÁT TRIỂN THEO CẤU TRÚC ĐỀ MINH HỌA BGD NGÀY 22-3-2024 KỲ THI TỐT NGHI...
30 ĐỀ PHÁT TRIỂN THEO CẤU TRÚC ĐỀ MINH HỌA BGD NGÀY 22-3-2024 KỲ THI TỐT NGHI...30 ĐỀ PHÁT TRIỂN THEO CẤU TRÚC ĐỀ MINH HỌA BGD NGÀY 22-3-2024 KỲ THI TỐT NGHI...
30 ĐỀ PHÁT TRIỂN THEO CẤU TRÚC ĐỀ MINH HỌA BGD NGÀY 22-3-2024 KỲ THI TỐT NGHI...
 
30 ĐỀ PHÁT TRIỂN THEO CẤU TRÚC ĐỀ MINH HỌA BGD NGÀY 22-3-2024 KỲ THI TỐT NGHI...
30 ĐỀ PHÁT TRIỂN THEO CẤU TRÚC ĐỀ MINH HỌA BGD NGÀY 22-3-2024 KỲ THI TỐT NGHI...30 ĐỀ PHÁT TRIỂN THEO CẤU TRÚC ĐỀ MINH HỌA BGD NGÀY 22-3-2024 KỲ THI TỐT NGHI...
30 ĐỀ PHÁT TRIỂN THEO CẤU TRÚC ĐỀ MINH HỌA BGD NGÀY 22-3-2024 KỲ THI TỐT NGHI...
 
30 ĐỀ PHÁT TRIỂN THEO CẤU TRÚC ĐỀ MINH HỌA BGD NGÀY 22-3-2024 KỲ THI TỐT NGHI...
30 ĐỀ PHÁT TRIỂN THEO CẤU TRÚC ĐỀ MINH HỌA BGD NGÀY 22-3-2024 KỲ THI TỐT NGHI...30 ĐỀ PHÁT TRIỂN THEO CẤU TRÚC ĐỀ MINH HỌA BGD NGÀY 22-3-2024 KỲ THI TỐT NGHI...
30 ĐỀ PHÁT TRIỂN THEO CẤU TRÚC ĐỀ MINH HỌA BGD NGÀY 22-3-2024 KỲ THI TỐT NGHI...
 
TỔNG HỢP ĐỀ THI CHÍNH THỨC KỲ THI TUYỂN SINH VÀO LỚP 10 THPT MÔN NGỮ VĂN NĂM ...
TỔNG HỢP ĐỀ THI CHÍNH THỨC KỲ THI TUYỂN SINH VÀO LỚP 10 THPT MÔN NGỮ VĂN NĂM ...TỔNG HỢP ĐỀ THI CHÍNH THỨC KỲ THI TUYỂN SINH VÀO LỚP 10 THPT MÔN NGỮ VĂN NĂM ...
TỔNG HỢP ĐỀ THI CHÍNH THỨC KỲ THI TUYỂN SINH VÀO LỚP 10 THPT MÔN NGỮ VĂN NĂM ...
 
TUYỂN TẬP 20 ĐỀ THI KHẢO SÁT HỌC SINH GIỎI MÔN TIẾNG ANH LỚP 6 NĂM 2020 (CÓ Đ...
TUYỂN TẬP 20 ĐỀ THI KHẢO SÁT HỌC SINH GIỎI MÔN TIẾNG ANH LỚP 6 NĂM 2020 (CÓ Đ...TUYỂN TẬP 20 ĐỀ THI KHẢO SÁT HỌC SINH GIỎI MÔN TIẾNG ANH LỚP 6 NĂM 2020 (CÓ Đ...
TUYỂN TẬP 20 ĐỀ THI KHẢO SÁT HỌC SINH GIỎI MÔN TIẾNG ANH LỚP 6 NĂM 2020 (CÓ Đ...
 
TUYỂN TẬP 25 ĐỀ THI HỌC SINH GIỎI MÔN TIẾNG ANH LỚP 6 NĂM 2023 CÓ ĐÁP ÁN (SƯU...
TUYỂN TẬP 25 ĐỀ THI HỌC SINH GIỎI MÔN TIẾNG ANH LỚP 6 NĂM 2023 CÓ ĐÁP ÁN (SƯU...TUYỂN TẬP 25 ĐỀ THI HỌC SINH GIỎI MÔN TIẾNG ANH LỚP 6 NĂM 2023 CÓ ĐÁP ÁN (SƯU...
TUYỂN TẬP 25 ĐỀ THI HỌC SINH GIỎI MÔN TIẾNG ANH LỚP 6 NĂM 2023 CÓ ĐÁP ÁN (SƯU...
 
BỘ ĐỀ PHÁT TRIỂN THEO CẤU TRÚC ĐỀ MINH HỌA BGD NGÀY 22-3-2024 KỲ THI TỐT NGHI...
BỘ ĐỀ PHÁT TRIỂN THEO CẤU TRÚC ĐỀ MINH HỌA BGD NGÀY 22-3-2024 KỲ THI TỐT NGHI...BỘ ĐỀ PHÁT TRIỂN THEO CẤU TRÚC ĐỀ MINH HỌA BGD NGÀY 22-3-2024 KỲ THI TỐT NGHI...
BỘ ĐỀ PHÁT TRIỂN THEO CẤU TRÚC ĐỀ MINH HỌA BGD NGÀY 22-3-2024 KỲ THI TỐT NGHI...
 
Sáng kiến Dạy học theo định hướng STEM một số chủ đề phần “vật sống”, Khoa họ...
Sáng kiến Dạy học theo định hướng STEM một số chủ đề phần “vật sống”, Khoa họ...Sáng kiến Dạy học theo định hướng STEM một số chủ đề phần “vật sống”, Khoa họ...
Sáng kiến Dạy học theo định hướng STEM một số chủ đề phần “vật sống”, Khoa họ...
 
Sáng kiến “Sử dụng ứng dụng Quizizz nhằm nâng cao chất lượng ôn thi tốt nghiệ...
Sáng kiến “Sử dụng ứng dụng Quizizz nhằm nâng cao chất lượng ôn thi tốt nghiệ...Sáng kiến “Sử dụng ứng dụng Quizizz nhằm nâng cao chất lượng ôn thi tốt nghiệ...
Sáng kiến “Sử dụng ứng dụng Quizizz nhằm nâng cao chất lượng ôn thi tốt nghiệ...
 
30 ĐỀ PHÁT TRIỂN THEO CẤU TRÚC ĐỀ MINH HỌA BGD NGÀY 22-3-2024 KỲ THI TỐT NGHI...
30 ĐỀ PHÁT TRIỂN THEO CẤU TRÚC ĐỀ MINH HỌA BGD NGÀY 22-3-2024 KỲ THI TỐT NGHI...30 ĐỀ PHÁT TRIỂN THEO CẤU TRÚC ĐỀ MINH HỌA BGD NGÀY 22-3-2024 KỲ THI TỐT NGHI...
30 ĐỀ PHÁT TRIỂN THEO CẤU TRÚC ĐỀ MINH HỌA BGD NGÀY 22-3-2024 KỲ THI TỐT NGHI...
 
10 ĐỀ KIỂM TRA + 6 ĐỀ ÔN TẬP CUỐI KÌ 2 VẬT LÝ 11 - KẾT NỐI TRI THỨC - THEO C...
10 ĐỀ KIỂM TRA + 6 ĐỀ ÔN TẬP CUỐI KÌ 2 VẬT LÝ 11 - KẾT NỐI TRI THỨC - THEO C...10 ĐỀ KIỂM TRA + 6 ĐỀ ÔN TẬP CUỐI KÌ 2 VẬT LÝ 11 - KẾT NỐI TRI THỨC - THEO C...
10 ĐỀ KIỂM TRA + 6 ĐỀ ÔN TẬP CUỐI KÌ 2 VẬT LÝ 11 - KẾT NỐI TRI THỨC - THEO C...
 
BỘ ĐỀ KIỂM TRA CUỐI KÌ 2 VẬT LÝ 11 - KẾT NỐI TRI THỨC - THEO CẤU TRÚC ĐỀ MIN...
BỘ ĐỀ KIỂM TRA CUỐI KÌ 2 VẬT LÝ 11 - KẾT NỐI TRI THỨC - THEO CẤU TRÚC ĐỀ MIN...BỘ ĐỀ KIỂM TRA CUỐI KÌ 2 VẬT LÝ 11 - KẾT NỐI TRI THỨC - THEO CẤU TRÚC ĐỀ MIN...
BỘ ĐỀ KIỂM TRA CUỐI KÌ 2 VẬT LÝ 11 - KẾT NỐI TRI THỨC - THEO CẤU TRÚC ĐỀ MIN...
 
SÁNG KIẾN “THIẾT KẾ VÀ SỬ DỤNG INFOGRAPHIC TRONG DẠY HỌC ĐỊA LÍ 11 (BỘ SÁCH K...
SÁNG KIẾN “THIẾT KẾ VÀ SỬ DỤNG INFOGRAPHIC TRONG DẠY HỌC ĐỊA LÍ 11 (BỘ SÁCH K...SÁNG KIẾN “THIẾT KẾ VÀ SỬ DỤNG INFOGRAPHIC TRONG DẠY HỌC ĐỊA LÍ 11 (BỘ SÁCH K...
SÁNG KIẾN “THIẾT KẾ VÀ SỬ DỤNG INFOGRAPHIC TRONG DẠY HỌC ĐỊA LÍ 11 (BỘ SÁCH K...
 
ĐỀ THAM KHẢO THEO HƯỚNG MINH HỌA 2025 KIỂM TRA CUỐI HỌC KÌ 2 NĂM HỌC 2023-202...
ĐỀ THAM KHẢO THEO HƯỚNG MINH HỌA 2025 KIỂM TRA CUỐI HỌC KÌ 2 NĂM HỌC 2023-202...ĐỀ THAM KHẢO THEO HƯỚNG MINH HỌA 2025 KIỂM TRA CUỐI HỌC KÌ 2 NĂM HỌC 2023-202...
ĐỀ THAM KHẢO THEO HƯỚNG MINH HỌA 2025 KIỂM TRA CUỐI HỌC KÌ 2 NĂM HỌC 2023-202...
 
HỌC TỐT TIẾNG ANH 11 THEO CHƯƠNG TRÌNH GLOBAL SUCCESS ĐÁP ÁN CHI TIẾT - CẢ NĂ...
HỌC TỐT TIẾNG ANH 11 THEO CHƯƠNG TRÌNH GLOBAL SUCCESS ĐÁP ÁN CHI TIẾT - CẢ NĂ...HỌC TỐT TIẾNG ANH 11 THEO CHƯƠNG TRÌNH GLOBAL SUCCESS ĐÁP ÁN CHI TIẾT - CẢ NĂ...
HỌC TỐT TIẾNG ANH 11 THEO CHƯƠNG TRÌNH GLOBAL SUCCESS ĐÁP ÁN CHI TIẾT - CẢ NĂ...
 
50 ĐỀ ĐỀ XUẤT THI VÀO 10 THPT SỞ GIÁO DỤC THANH HÓA MÔN TIẾNG ANH 9 CÓ TỰ LUẬ...
50 ĐỀ ĐỀ XUẤT THI VÀO 10 THPT SỞ GIÁO DỤC THANH HÓA MÔN TIẾNG ANH 9 CÓ TỰ LUẬ...50 ĐỀ ĐỀ XUẤT THI VÀO 10 THPT SỞ GIÁO DỤC THANH HÓA MÔN TIẾNG ANH 9 CÓ TỰ LUẬ...
50 ĐỀ ĐỀ XUẤT THI VÀO 10 THPT SỞ GIÁO DỤC THANH HÓA MÔN TIẾNG ANH 9 CÓ TỰ LUẬ...
 
ĐỀ THAM KHẢO THEO HƯỚNG MINH HỌA 2025 KIỂM TRA GIỮA HỌC KÌ + CUỐI HỌC KÌ 2 NĂ...
ĐỀ THAM KHẢO THEO HƯỚNG MINH HỌA 2025 KIỂM TRA GIỮA HỌC KÌ + CUỐI HỌC KÌ 2 NĂ...ĐỀ THAM KHẢO THEO HƯỚNG MINH HỌA 2025 KIỂM TRA GIỮA HỌC KÌ + CUỐI HỌC KÌ 2 NĂ...
ĐỀ THAM KHẢO THEO HƯỚNG MINH HỌA 2025 KIỂM TRA GIỮA HỌC KÌ + CUỐI HỌC KÌ 2 NĂ...
 
TỔNG HỢP 30 ĐỀ THI CHỌN HSG CÁC TRƯỜNG THPT CHUYÊN VÙNG DUYÊN HẢI & ĐỒNG BẰNG...
TỔNG HỢP 30 ĐỀ THI CHỌN HSG CÁC TRƯỜNG THPT CHUYÊN VÙNG DUYÊN HẢI & ĐỒNG BẰNG...TỔNG HỢP 30 ĐỀ THI CHỌN HSG CÁC TRƯỜNG THPT CHUYÊN VÙNG DUYÊN HẢI & ĐỒNG BẰNG...
TỔNG HỢP 30 ĐỀ THI CHỌN HSG CÁC TRƯỜNG THPT CHUYÊN VÙNG DUYÊN HẢI & ĐỒNG BẰNG...
 
ĐỀ CƯƠNG + TEST ÔN TẬP CUỐI KÌ 2 TIẾNG ANH 11 - GLOBAL SUCCESS (THEO CHUẨN MI...
ĐỀ CƯƠNG + TEST ÔN TẬP CUỐI KÌ 2 TIẾNG ANH 11 - GLOBAL SUCCESS (THEO CHUẨN MI...ĐỀ CƯƠNG + TEST ÔN TẬP CUỐI KÌ 2 TIẾNG ANH 11 - GLOBAL SUCCESS (THEO CHUẨN MI...
ĐỀ CƯƠNG + TEST ÔN TẬP CUỐI KÌ 2 TIẾNG ANH 11 - GLOBAL SUCCESS (THEO CHUẨN MI...
 
VẬN DỤNG KIẾN THỨC LIÊN MÔN TRONG GIẢI BÀI TẬP ÔN THI THPTQG MÔN SINH HỌC - H...
VẬN DỤNG KIẾN THỨC LIÊN MÔN TRONG GIẢI BÀI TẬP ÔN THI THPTQG MÔN SINH HỌC - H...VẬN DỤNG KIẾN THỨC LIÊN MÔN TRONG GIẢI BÀI TẬP ÔN THI THPTQG MÔN SINH HỌC - H...
VẬN DỤNG KIẾN THỨC LIÊN MÔN TRONG GIẢI BÀI TẬP ÔN THI THPTQG MÔN SINH HỌC - H...
 

Recently uploaded

QUẢN LÝ HOẠT ĐỘNG GIÁO DỤC KỸ NĂNG SỐNG CHO HỌC SINH CÁC TRƯỜNG TRUNG HỌC CƠ ...
QUẢN LÝ HOẠT ĐỘNG GIÁO DỤC KỸ NĂNG SỐNG CHO HỌC SINH CÁC TRƯỜNG TRUNG HỌC CƠ ...QUẢN LÝ HOẠT ĐỘNG GIÁO DỤC KỸ NĂNG SỐNG CHO HỌC SINH CÁC TRƯỜNG TRUNG HỌC CƠ ...
QUẢN LÝ HOẠT ĐỘNG GIÁO DỤC KỸ NĂNG SỐNG CHO HỌC SINH CÁC TRƯỜNG TRUNG HỌC CƠ ...ThunTrn734461
 
Kiểm tra chạy trạm lí thuyết giữa kì giải phẫu sinh lí
Kiểm tra chạy trạm lí thuyết giữa kì giải phẫu sinh líKiểm tra chạy trạm lí thuyết giữa kì giải phẫu sinh lí
Kiểm tra chạy trạm lí thuyết giữa kì giải phẫu sinh líDr K-OGN
 
Chàm - Bệnh án (da liễu - bvdlct ctump) .pptx
Chàm - Bệnh án (da liễu - bvdlct ctump) .pptxChàm - Bệnh án (da liễu - bvdlct ctump) .pptx
Chàm - Bệnh án (da liễu - bvdlct ctump) .pptxendkay31
 
Thong bao 337-DHPY (24.4.2024) thi sat hach Ngoai ngu dap ung Chuan dau ra do...
Thong bao 337-DHPY (24.4.2024) thi sat hach Ngoai ngu dap ung Chuan dau ra do...Thong bao 337-DHPY (24.4.2024) thi sat hach Ngoai ngu dap ung Chuan dau ra do...
Thong bao 337-DHPY (24.4.2024) thi sat hach Ngoai ngu dap ung Chuan dau ra do...hoangtuansinh1
 
Sơ đồ tư duy môn sinh học bậc THPT.pdf
Sơ đồ tư duy môn sinh học bậc THPT.pdfSơ đồ tư duy môn sinh học bậc THPT.pdf
Sơ đồ tư duy môn sinh học bậc THPT.pdftohoanggiabao81
 
bài 5.1.docx Sinh học di truyền đại cương năm nhất của học sinh y đa khoa
bài 5.1.docx Sinh học di truyền đại cương năm nhất của học sinh y đa khoabài 5.1.docx Sinh học di truyền đại cương năm nhất của học sinh y đa khoa
bài 5.1.docx Sinh học di truyền đại cương năm nhất của học sinh y đa khoa2353020138
 
Trích dẫn trắc nghiệm tư tưởng HCM5.docx
Trích dẫn trắc nghiệm tư tưởng HCM5.docxTrích dẫn trắc nghiệm tư tưởng HCM5.docx
Trích dẫn trắc nghiệm tư tưởng HCM5.docxnhungdt08102004
 
ôn tập lịch sử hhhhhhhhhhhhhhhhhhhhhhhhhh
ôn tập lịch sử hhhhhhhhhhhhhhhhhhhhhhhhhhôn tập lịch sử hhhhhhhhhhhhhhhhhhhhhhhhhh
ôn tập lịch sử hhhhhhhhhhhhhhhhhhhhhhhhhhvanhathvc
 
NQA Lợi ích Từ ISO và ESG Tăng Trưởng và Bền Vững ver01.pdf
NQA Lợi ích Từ ISO và ESG Tăng Trưởng và Bền Vững ver01.pdfNQA Lợi ích Từ ISO và ESG Tăng Trưởng và Bền Vững ver01.pdf
NQA Lợi ích Từ ISO và ESG Tăng Trưởng và Bền Vững ver01.pdfNguyễn Đăng Quang
 
Chuong trinh dao tao Su pham Khoa hoc tu nhien, ma nganh - 7140247.pdf
Chuong trinh dao tao Su pham Khoa hoc tu nhien, ma nganh - 7140247.pdfChuong trinh dao tao Su pham Khoa hoc tu nhien, ma nganh - 7140247.pdf
Chuong trinh dao tao Su pham Khoa hoc tu nhien, ma nganh - 7140247.pdfhoangtuansinh1
 

Recently uploaded (10)

QUẢN LÝ HOẠT ĐỘNG GIÁO DỤC KỸ NĂNG SỐNG CHO HỌC SINH CÁC TRƯỜNG TRUNG HỌC CƠ ...
QUẢN LÝ HOẠT ĐỘNG GIÁO DỤC KỸ NĂNG SỐNG CHO HỌC SINH CÁC TRƯỜNG TRUNG HỌC CƠ ...QUẢN LÝ HOẠT ĐỘNG GIÁO DỤC KỸ NĂNG SỐNG CHO HỌC SINH CÁC TRƯỜNG TRUNG HỌC CƠ ...
QUẢN LÝ HOẠT ĐỘNG GIÁO DỤC KỸ NĂNG SỐNG CHO HỌC SINH CÁC TRƯỜNG TRUNG HỌC CƠ ...
 
Kiểm tra chạy trạm lí thuyết giữa kì giải phẫu sinh lí
Kiểm tra chạy trạm lí thuyết giữa kì giải phẫu sinh líKiểm tra chạy trạm lí thuyết giữa kì giải phẫu sinh lí
Kiểm tra chạy trạm lí thuyết giữa kì giải phẫu sinh lí
 
Chàm - Bệnh án (da liễu - bvdlct ctump) .pptx
Chàm - Bệnh án (da liễu - bvdlct ctump) .pptxChàm - Bệnh án (da liễu - bvdlct ctump) .pptx
Chàm - Bệnh án (da liễu - bvdlct ctump) .pptx
 
Thong bao 337-DHPY (24.4.2024) thi sat hach Ngoai ngu dap ung Chuan dau ra do...
Thong bao 337-DHPY (24.4.2024) thi sat hach Ngoai ngu dap ung Chuan dau ra do...Thong bao 337-DHPY (24.4.2024) thi sat hach Ngoai ngu dap ung Chuan dau ra do...
Thong bao 337-DHPY (24.4.2024) thi sat hach Ngoai ngu dap ung Chuan dau ra do...
 
Sơ đồ tư duy môn sinh học bậc THPT.pdf
Sơ đồ tư duy môn sinh học bậc THPT.pdfSơ đồ tư duy môn sinh học bậc THPT.pdf
Sơ đồ tư duy môn sinh học bậc THPT.pdf
 
bài 5.1.docx Sinh học di truyền đại cương năm nhất của học sinh y đa khoa
bài 5.1.docx Sinh học di truyền đại cương năm nhất của học sinh y đa khoabài 5.1.docx Sinh học di truyền đại cương năm nhất của học sinh y đa khoa
bài 5.1.docx Sinh học di truyền đại cương năm nhất của học sinh y đa khoa
 
Trích dẫn trắc nghiệm tư tưởng HCM5.docx
Trích dẫn trắc nghiệm tư tưởng HCM5.docxTrích dẫn trắc nghiệm tư tưởng HCM5.docx
Trích dẫn trắc nghiệm tư tưởng HCM5.docx
 
ôn tập lịch sử hhhhhhhhhhhhhhhhhhhhhhhhhh
ôn tập lịch sử hhhhhhhhhhhhhhhhhhhhhhhhhhôn tập lịch sử hhhhhhhhhhhhhhhhhhhhhhhhhh
ôn tập lịch sử hhhhhhhhhhhhhhhhhhhhhhhhhh
 
NQA Lợi ích Từ ISO và ESG Tăng Trưởng và Bền Vững ver01.pdf
NQA Lợi ích Từ ISO và ESG Tăng Trưởng và Bền Vững ver01.pdfNQA Lợi ích Từ ISO và ESG Tăng Trưởng và Bền Vững ver01.pdf
NQA Lợi ích Từ ISO và ESG Tăng Trưởng và Bền Vững ver01.pdf
 
Chuong trinh dao tao Su pham Khoa hoc tu nhien, ma nganh - 7140247.pdf
Chuong trinh dao tao Su pham Khoa hoc tu nhien, ma nganh - 7140247.pdfChuong trinh dao tao Su pham Khoa hoc tu nhien, ma nganh - 7140247.pdf
Chuong trinh dao tao Su pham Khoa hoc tu nhien, ma nganh - 7140247.pdf
 

DE PHAT TRIEN THEO CAU TRUC DE MINH HOA 2022 MON TIENG ANH DE 16 20.pdf

  • 1. Question 1: Robert Mugabe has ruled Zimbabwe since the _________ achieved independence in 1980. A. national B. nation C. nationality D. nationally - nation (n): đất nước - national (adj): thuộc đất nước - nationally (adv) - nationality (n): quốc tịch Sau “the” cần danh từ và nghĩa câu B hợp lý nhất
  • 2. Question 2: It is time to begin, _________? A. isn’t it B. are they C. do I D. is time Cấu trúc câu hỏi đuôi, Subject + verb + (object) + (…) + , + trợ động từ + subject/ đại từ tương ứng với subject + ? Question 3: Otherwise, you give a(n) ________ impression that you are too anxious for a settlement. A. wrong B. false C. improper D. incorrect - give a false impression (collo): để lại ấn tượng sai lầm Question 4: Grinder, a 35-year military veteran, was named to the post ______ Thursday, Jan. 13 A. on B. in C. of D. with
  • 3. Question 5: Poteet will receive a $100 cash ________ and $200 for his school to purchase books. A. bounty B. award C. gift D. prize - prize : giải thưởng, phần thưởng (ít trang trọng, dùng cho người có hành động tốt) - award: giải thưởng, phần thưởng ( dùng cho người nào đó làm hành động lớn lao ) - gift: món quà - bonus: phần thưởng thêm Question 6: Kids without guidance getting into trouble __ there was nothing else to do A. because B. though C. because of D. despite - because + clause: bởi vì - because of + N: bởi vì - though + clause: mặc dù - despite + N: mặc dù
  • 4. Question 7: Tonight, Hana wore a __________ dress at the party A. gorgeous red Chinese B. gorgeous Chinese red C. red Chinese gorgeous D. Chinese red gorgeous Trật tự: opinion -> color -> origin Question 8: Don't _________ when he is telling the story. A. stop in B. stop off C. break in D. break into - stop in: ở nhà - stop off: ngừng chuyến đi/hành trình - break in: chen lời - break into: đột nhập
  • 5. Question 9: When my father ________ TV, my friend suddenly came to my house. A. has watched B. watches C. watched D. was watching Hành động đang xảy ra liên tiếp trong quá khứ thì có hành động khác chen ngang Question 10: When I see him, I ___________ him your regards. A. will give B. give C. would give D. gave Sau mệnh đề chỉ thời gian dùng hiện tại ( vế trước dùng tương lai)
  • 6. Question 11: She said to the mouse, you must __________ me a favor, and once more manage the house for a day alone. A. make B. do C. put D. go - do sb a favor: giúp đỡ ai đó việc gì Question 12: The doctor said that she might never be able to _________ children. A. care B. tolerate C. bear D. feed - bear (v): chịu đựng/sinh ra - tolerate (v): chịu đựng - care (v): quan tâm - feed (v): cho ai ăn => về nghĩa thì sẽ chọn bear (sinh ra)
  • 7. Question 13: This house ____________ in the 18th century. A. is building B. were build C. was built D. is built - Câu bị động trong quá khứ: S + was/were + V3/ed Question 14: _______ her homework, she watched her favourite movie A. Having finished B. finish C. being finished D. she finish Nếu 2 hành động ở 2 mệnh đề xảy ra theo thứ tự trước sau, ta lược bỏ chủ từ của mệnh đề có hành động xảy ra trước rồi sau đó chia động từ trong mệnh đề đó ở dạng Having + V3/ Ved. Question 15: The sooner JK take your medicine,_________ he will feel A. better B. the better C. the good D. well So sánh tăng tiến: The + comparative + S + V + the + comparative + S + V
  • 8. Question 16: Ann and Peter are doing homework together. – Ann: “Could you lend me that book” – Peter: “____________” A. No, I’m busy B. That’s what I think C. Why did you say that? D. Of course, here it is Câu giao tiếp: lời nhờ đưa đồ và cách đáp lại (theo nghĩa) Question 17: Linda is thanking Daniel for his birthday present. Linda: “Thanks for the book. I’ve been looking for it for months.” Daniel: “____________” A. You can say that again. B. Thank you for looking at it C. I like reading books. D. I’m glad you like it. Câu giao tiếp: cảm ơn quà và đáp lại (theo nghĩa)
  • 9. Question 18: A. permission B. suggestion C. refusal D. possible A,B,C: âm 2 D: âm 1 Question 19: A. forest B. succeed C. homeless C. nation B: âm 2 A,C,D: âm 1
  • 10. Question 20: A. stopped B. wanted C. decided D. visited Question 21: A. head B. please C. heavy C. measure
  • 11. Question 22: They tell us she isn't allowed to leave unless she gets the doctor's permission A. disapproval B. objection C. agreement D. refusal - Permission = agreement (n): sự cho phép - Disapproval (n): sự không chấp thuận - Objection = refusal (n): sự từ chối- abide = obey (v): nghe lời Question 23: Public schools must abide by these rules, so why shouldn't charter schools, too? A. obey B. tolerate C. dislike D. allow - abide = obey (v): nghe lời - tolerate (v): chịu đựng - allow (v): cho phép
  • 12. Question 24: I would love to go back to college, but unfortunately, that ship has sailed. A. it was late B. it was timely C. it was inconvenient D. it was traditional That ship has sailed: đã quá trễ rồi Question 25: A comfortable working environment will increase productivity. A. promote B. raise C. decrease D. go up Increase = go up = raise = promote (v): tăng >< decrease (v): giảm
  • 13. Question 26: It was not until they had reached Dak Lak that they realized how far they had gone. A. Not until they had reached Dak Lak did they realize how far they had gone. B. Not until had they reached Dak Lak they realized how far they had gone. C. Not until they reached Dak Lak had they realized how far they had gone. D. Not until they realized how far they had gone had they reached Dak Lak Đảo ngữ Not until + clause ………… trợ động từ + S + V - Dịch: vừa mới…….thì……. Question 27: They had a lot of homework last Monday. A. They wish they had not a lot of homework last Monday B. They wish they didn’t have a lot of homework last Monday C. They wish they had had little homework last Monday D. They wish they wouldn’t have a lot of homework last Monday Điều ước không có thật ở quá khứ: S + wish + S + had + V3/ed
  • 14. Question 28: Let's start by reviewing what we do with those friends last week. A. start B. reviewing C. do D. friend “do” sai: thì quá khứ, có trạng ngữ chỉ thời gian “last week” cuối câu Sửa: did Question 29: Lily and her mother went to hospital last week to check our health. A. her B. went C. last D. our health “our health” sai: tính từ sở hữu không đúng theo nghĩa Sửa: their health Question 30: I must complement you on your handling of a very difficult situation. A. must B. complement C. handling D. very Complement: bổ ngữ => sai vì nhầm lẫn về nghĩa Sửa: Compliment (khen)
  • 15. Question 31: She advised him to go to the dentist immediately. A. He would go to the dentist immediately B. She should go to the dentist immediately C. He should go to the dentist immediately D. He must go to the dentist immediately Lời khuyên => should Question 32: She said, “ Don’t laugh, Jessica. Be serious.” A. She said not to laugh and be serious. B. She said Jessica not to laugh and be serious. C. She told Jessica not to laugh and asked him to be serious. D. She told Jessica do not laugh and be serious Câu yêu cầu trong tường thuật: tell sb (not) to do something Question 33: The last time I played the piano was 10 years ago. A. I have not played the piano for 10 years ago B. I have not played the piano for 10 years C. I have not played the piano since 10 years D. I did not play the piano since 10 years Quá khứ đơn chuyển thành hiện tại hoàn thành Bỏ “ago” sau thời gian, và đây là khoảng thời gian nên dùng “for”
  • 16. Question 34: Plants and animals will find it difficult to escape from or adjust to the effect of global warming. Scientists have already observed shifts in the life cycles of (34) ________plants and animals A. many B. each C. much C. little Plants và animals là số nhiều đếm được => many Question 35: such as flowers (35)______________earlier and birds hatching earlier in the spring. A. increasing B. blooming C. growing D. swelling - bloom (v): nở (dùng cho hoa) - grow (v): lớn lên - swell (v): phồng lên
  • 17. Question 36: Many species have begun shifting (36)_____________ they live or their annual migration patterns due to warmer temperatures. A. that B. what C. which D. where Mệnh đề chỉ nơi chốn => hợp nghĩa và ngữ cảnh Question 37: Species living in unique ecosystems, such as those found in polar and mountaintop regions, are especially at risk (37)___________ migration to new habitats is not possible. A. because B. but C. and D. although Question 38: For example, polar bears and marine mammals in the Arctic are already threatened by(38) ________________ sea ice but have nowhere farther to go. A. decreasing B. falling C. deepening D. dwindling Dwindling sea ice: suy giảm băng biển
  • 18. Question 39: What is the passage mainly about? A. Brain Waves Tell the Story B. People with fixed mindsets C. The feedbacks are important D. People with a growth mindset Có thể dùng phương pháp loại trừ: B: Sai vì có nói thêm people with a growth mindset C: Sai vì trong bài không đề cập D. Sai vì nói có nói thêm people with a fixed mindset => Chọn A Question 40: According to paragraph 3, what is the result when people with a fixed mindset were presented with information that could help them learn? A. They paid close attention to B. They were reluctant C. They were angry D. They showed no interest Dẫn chứng: “But when………..sign of interest” ở câu đầu đoạn 3
  • 19. Question 41: Which of the following is NOT mentioned in paragraph 2 and 3 as for people with a fixed mindset? A. They were not interested in new knowledge B. They only paid attention to the the feedbacks C. Learning made them get a headache D. They were inattentive to information that helps them learn Dùng phương pháp dò từng câu có trong bài: A: Có ở câu đầu đoạn 3 “ but when they were presented…… no sign of interest” B: Có ở câu đầu đoạn 2 “ people with fixed mindset….ability” C. Không đề cập (chọn) D. như câu A Question 42: The word growth in paragraph 4 is closest in meaning to _______. A. decline B. development C. height D. reduction Growth (n) = development : phát triển Question 43: The word they in paragraph 1 refers to _______. A. People with both mindsets B. Hard questions C. Brainwaves D. feedback Dẫn chứng: “they” trong đoạn 1 chỉ người ở số nhiều => đứng trước nó chỉ có “people with both mindsets”.
  • 20. Question 44: Which of the following is the best title for the passage? A. The open-water swimming record of Diana Nyad B. Every Sport is a Team Sport C. Swimming is difficult D. The strength of trainers to athletes Dẫn chứng: Câu đầu đoạn 1 và xuyên suốt bài nói về vấn đề sự hợp tác của nhiều người dù là môn thể thao cá nhân Question 45: According to paragraph 1, what type of sports has a team? A. individual sports B. team sportsC. underwater sports D. both A&B Dẫn chứng: câu đầu đoạn 1 có cụm “every sport”, tức là chỉ tất cả các loại môn thể thao
  • 21. Question 46: The word arduous in paragraph 2 is closest in meaning to _______. A. heavy B. easy C. strenuous D. quick Arduous (adj) = strenuous: cật lực, tốn nhiều công sức Question 47: The word it in paragraph 1 refers to _______. A. every sport B. tennis C. golf D. a team sport “it” trong đoạn văn đứng sau 2 danh từ số ít : “every sport” và “a team sport” =>Loại B,C - Xét về nghĩa của câu chứa “it” => chọn every sport
  • 22. Question 48: The word coaches in paragraph 2 is closest in meaning to _______. A. partners B. instructors C. pupils D. carriages Coach (n) = instructor: người hướng dẫn, huấn luyện viên Question 49: Which of the following is NOT true according to the passage? A. A team of guides measure the winds and the current, and watch for obstacles B. Divers teach her how to swim properly C. NASA experts guide on nutrition and endurance D. Trainers who talked her through uncontrollable shivers, nausea, hallucinations, and despair A. đúng ( … a team of guides ( for meassuring….), câu 6 từ dưới lên Divers look for sharks => Câu B sai C. NASA experts…. dòng 4 từ dưới lên D. Trainers who talked…………….. (dòng 2 từ dưới lên)
  • 23. Question 50: Which of the following can be referred from the passage? A. Diana Nyad lost so much weight, which led to hallucinations B. A few months of training were not enough for her to be ready C. Diana Nyad’s trainers contributed mostly to her achievements D. Diana Nyad were supported by over 50 people Dẫn chứng: câu cuối đoạn văn: “but it took fifty-one other people to do it: 51 người khác làm nó.
  • 24.
  • 25.
  • 26.
  • 27.
  • 28.
  • 29.
  • 30.
  • 31.
  • 32.
  • 33.
  • 34.
  • 35.
  • 36.
  • 37.
  • 38.
  • 39.
  • 40.
  • 41.
  • 42.
  • 43.
  • 44.
  • 45.
  • 46.
  • 47.
  • 48. Question 1. Children can be encouraged to work together as a team by playing _______ sports. A. compete B. competitively C. competitive D. competition Kiến thức: Từ loại Giải thích: - Ta thấy chỗ trống ở trước danh từ nên từ cần điền là một tính từ. Xét các đáp án : A. compete (v): thi/ thi đấu B. Competitively (adv): cạnh tranh C. competitive (a): cạnh tranh D. competition (n): cuộc thi Vậy đáp án là C Tạm dịch: Trẻ em có thể được khuyến khích làm việc cùng nhau như một đội bằng cách chơi các môn thể thao cạnh tranh.
  • 49. Question 2. Michael rarely returns to his hometown, _______? A. doesn’t heB. hasn’t he C. does he D. has he Kiến thức: Câu hỏi đuôi Giải thích: Ta có chú ý trong khi thành lập câu hỏi đuôi như sau: Nếu trong câu dạng khẳng định có rarely, barely, hardly, never thì thành lập câu hỏi đuôi như đối với câu phủ định. Như vậy ở đây ta mượn trợ động từ là does. Chọn đáp án đúng là C. Tạm dịch. Michael hiếm khi về quê, có phải không?
  • 50. Question 3. Pesticide residues in fruit and vegetable can be _______ to health. A. crucial B. supportive C. receptive D. destructive Kiến thức: Từ vựng Giải thích: Xét các đáp án: A. crucial (adj): cực kỳ quan trọng B. supportive (adj): khuyến khích, cổ vũ C. receptive (adj): dễ tiếp thu D. destructive (adj): phá hoại, gây hại Phù hợp ngữ nghĩa nhất của câu là D. Tạm dịch: Dư lượng thuốc trừ sâu trong trái cây và rau quả có thể gây hại sức khoẻ.
  • 51. Question 4. When you grow up, you must learn how to become independent _______ your parents. A. withB. in C. of D. on Kiến thức: Giới từ Giải thích: Ta có cấu trúc: be/ get/ become independent of: tự lập/ không phụ thuộc vào Vậy đáp án đúng là C Tạm dịch: Khi lớn lên, bạn phải học cách tự lập không phụ thuộc vào cha mẹ.
  • 52. Question 5. I don’t feel like buying a _______ in a poke; we’d better check the content. A. pig B. cattle C. buffalo D. ox Kiến thức: Thành ngữ Giải thích: Ta có thành ngữ: buy a pig in a poke: mua vô tội vạ Vậy đáp án đúng là A Tạm dịch: Tôi không muốn mua vô tội vạ; tốt hơn chúng ta nên kiểm tra nội dung.
  • 53. Question 6. Dang Van Lam was absent from Vietnam’s World Cup qualifiers in June _______ his contact with a Covid-19 case. A. despite B. because C. although D. because of Kiến thức: Liên từ Giải thích: Xét các đáp án: A. despite (+ V-ing/ noun phrase): mặc dù B. because (+ clause): bởi vì C. although (+ clause): mặc dù D. because of (+ V-ing/ noun phrase): bởi vì Ta thấy sau chỗ trống là là một cụm danh từ nên loại đáp án B và C Dựa vào nghĩa của câu, đáp án D là đúng Tạm dịch: Đặng Văn Lâm đã vắng mặt ở vòng loại World Cup của Việt Nam vào tháng Sáu vì nhiễm Covid-19.
  • 54. Question 7. I remember she wore a _______ dress to go out with her boyfriend last week. A. cotton white Vietnamese B. Vietnamese white cotton C. white Vietnamese cotton D. white cotton Vietnamese Kiến thức: Trật tự tính từ Giải thích: Theo quy tắc trật tự tính từ trong câu: OSASCOMP white – Color; Vietnamese – Origin; cotton – Material Vậy đáp án đúng là C Tạm dịch: Tôi nhớ cô ấy đã mặc một chiếc váy vải cotton Việt Nam màu trắng để đi ra ngoài với bạn trai tuần trước.
  • 55. Question 8. She _______ quite often as a guest on popular television shows, as well as in television and bigscreen movies. A. looked up B. looked after C. turned up D. turned on Kiến thức: Cụm động từ Giải thích: Xét các đáp án: A. look up: tra cứu (từ điển) B. look after: chăm sóc C. turn up: đến/ xuất hiện D. turn on: bật (điện/ công tắc điện) Dựa vào nghĩa và ngữ cảnh, đáp án C là đúng Tạm dịch: Cô xuất hiện khá thường xuyên với tư cách khách mời trên các chương trình truyền hình nổi tiếng, cũng như trong các bộ phim truyền hình và màn ảnh rộng.
  • 56. Question 9. The students _______ about their summer vacation when their teacher came in. A. talked B. were talking C. have talked D. talk Kiến thức: Thì quá khứ tiếp diễn Giải thích: Ta thấy mệnh đề trạng ngữ chỉ thời gian bắt đầu bằng WHEN và động từ CAME ở quá khứ đơn nên động từ ở mệnh đề chính ở thì quá khứ tiếp diễn (một hành động đang diễn ra thì bị một hành động khác xen vào). Công thức: S1 + was/ were + V1-ing + when + S2 + V2-simple past Vậy đáp án đúng là: B (were talking) Tạm dịch: Các học sinh đang nói về kỳ nghỉ hè của họ khi giáo viên của họ đến
  • 57. Question 10. _______, they will have done their homework. A. By the time you finish cooking B. By the time you finished cooking C. When you finish cooking D. While you finish cooking Question 10. Đáp án A Kiến thức: Sự phối hợp các thì (Tương lai & hiện tại) Giải thích: - Mệnh đề chính chia ở thì tương lai hoàn thành nên chọn đáp án A. * Chú ý : By the time + S + V(htt), S + will have + Vp2 By the time + S + V(qkđ), S + had + Vp2 Tạm dịch : Đến lúc bạn hoàn thành việc nấu ăn, chúng sẽ đã hoàn thành bài tập của chúng.
  • 58. Question 11. Due to the nature of the earthquake, a much larger _______ of the population might be affected. A. density B. totality C. segment D. division Kiến thức: Cụm từ cố định Giải thích: Ta có các cụm từ: - density of the population = population density: mật độ dân số (danh từ không đếm được) - segment of the population = population segment: phân khúc dân số/ bộ phận dân số (danh từ đếm được) Vậy đáp án đúng là C Tạm dịch: Do tính chất của trận động đất, một bộ phận dân số lớn hơn nhiều có thể bị ảnh hưởng.
  • 59. Question 12. Volunteers may be required to obtain Red Cross _______ in order to serve through hospitals and healthcare organizations or provide disaster relief. A. diploma B. certification C. license D. degree Kiến thức: Từ vựng Giải thích: A. Diploma /dɪˈpləʊ.mə/ (n): chứng chỉ, bằng cấp (dành cho các khóa học kéo dài từ 1-2 năm) B. Certification /ˌsɜː.tɪ.fɪˈkeɪ.ʃən/ (n): giấy chứng nhận C. Licence /ˈlaɪ.səns/ (n): giấy phép D. Degree /dɪˈɡriː/ (n): bằng cấp (dành cho các khóa học kéo dài từ 3-4 năm) Dựa vào nghĩa và ngữ cảnh đáp án đúng là B Tạm dịch: Các tình nguyện viên có thể được yêu cầu có giấy chứng nhận Hội chữ thập đỏ để mà phục vụ ở bệnh viện và các tổ chức chăm sóc sức khỏe hoặc là cung cấp sự cứu nạn.
  • 60. Question 13. Last night Laura _______ a big diamond ring by her husband-to-be at her birthday party. A. was givingB. gave C. gives D. was given Kiến thức: Câu bị động Giải thích: Ta thấy trạng ngữ chỉ thời gian LAST NIGHT nên động từ chia ở thì quá khứ đơn nên loại đáp án A và C. Ta có công thức : give Sb St : cho/ tặng ai cái gì Hơn nữa, ta thấy trong câu có BY HER HUSBAND-TO-BE nên động từ chia ở bị động – loại B Vậy đáp án đúng là D Tạm dịch : Đêm qua, Laura đã được chồng sắp cưới tặng một chiếc nhẫn kim cương lớn trong bữa tiệc sinh nhật.
  • 61. Question 14. ________ the homework, he was allowed to go out with his friends. A. Finishing B. Finish C. To finish D. Having finished Kiến thức : Rút gọn mệnh đề trạng ngữ Giải thích: Khi hai mệnh đề cùng chủ ngữ, và câu muốn nhấn mạnh hành động phía trước được hoàn thành xong trước rồi hành động phía sau mới xảy ra thì chúng ta dùng công thức: Having + P2, S+Ved. Vậy đáp án đúng D. Tạm dịch: Sau khi hoàn thành xong bài tập về nhà, anh ấy được phép đi chơi với những người bạn của mình.
  • 62. Question 15. _______the time passes, _______I feel! The deadline of my thesis is coming, but I have just finished half of it. A. The faster / the nervous B. The more fast / the nervous C. The fast / the more nervous D. The faster / the more nervous Kiến thức: So sánh kép Giải thích: A. The faster / the nervous: sai công thức B. The more fast / the nervous: sai công thức C. The fast / the more nervous: sai công thức D. The faster / the more nervous: đúng công thức so sánh kép The + comparative + S1+ V1, the comparative + S2 + V2: càng ……, càng …… Tạm dịch: Thời gian trôi qua càng nhanh, tôi càng cảm thấy hồi hộp.
  • 63. Question 16. Tom is in Ho Chi Minh city and asks a passer-by the way to the railway station. - Tom: “Can you show me the way to the railway station, please?” - Passer-by: “_______” A. No way. B. Just round the comer over there. C. Look it up in a dictionary! D. There’s no traffic near here. Kiến thức: Tình huống giao tiếp (Đáp lại lời đề nghị) Giải thích: Tình huống giao tiếp: Tom đang ở thành phố Hồ Chí Minh và hỏi người qua đường đường tới nhà ga xe lửa. - Tom: Làm ơn chỉ cho tôi đường đến nhà ga xe lửa. - Người đi đường: _______. Xét các đáp án: A. Không đời nào B. Nó chỉ ở quanh góc kia thôi C. Hãy tra cứu nó trong cuốn từ điển! D. Không có giao thông ở gần đây Xét về nghĩa và tình huống giao tiếp thì chọn được B Bản word phát hành từ website Tailieuchuan.vn
  • 64. Question 17. Marry is talking to Linda over the phone. - Mary: “Thank you for helping me prepare for the party.” - Linda: “_______” A. My pleasure B. The meal was out of this world C. Never mention me D. Of course not Kiến thức: Tình huống giao tiếp (Đáp lại lời cảm ơn) Giải thích: Tình huống giao tiếp: Marry đang nói chuyện với Linda qua điện thoại. - Mary: "Cảm ơn bạn đã giúp tôi chuẩn bị cho bữa tiệc." - Linda: “_______” Xét các đáp án: A. Niềm vui của tôi B. Bữa ăn đã ra khỏi thế giới này C. Không bao giờ đề cập đến tôi D. Tất nhiên là không Xét về nghĩa và tình huống giao tiếp thì chọn được A
  • 65. Question 18. A. uncertain B. arrogant C. familiar D. impatient Kiến thức: Trọng âm của từ 3 âm tiết Giải thích: - uncertain /ʌnˈsɜːtn/: trọng âm rơi vào âm tiết thứ hai vì theo quy tắc tiền tố UN- không làm thay đổi trọng âm chính. - arrogant /ˈærəɡənt/ : trọng âm rơi vào âm tiết thứ nhất vì theo quy tắc trọng âm ưu tiên rơi vào âm chứa nguyên âm dài. - familiar /fəˈmɪlɪə/: trọng âm rơi vào âm tiết thứ hai - impatient /ɪmˈpeɪʃnt/: trọng âm rơi vào âm tiết thứ hai vì theo quy tắc tiền tố IM- không làm thay đổi trọng âm chính. Vậy đáp án đúng là B
  • 66. Question 19. A. precede B. offer C. visit D. finish Question 19. Đáp án: A Kiến thức: Trọng âm của từ 2 âm tiết Giải thích: A. precede (v) /prɪˈsiːd/ : động từ nên trọng âm rơi vào thứ 2 B. offer (v) /ˈofə/ : động từ 2 âm tiết nhưng trọng âm rơi vào 1 vì có đuôi -ER C. visit (v) /ˈvizit/ : động từ 2 âm tiết mà cả 2 âm tiết đều là /I/ nên trọng âm rơi vào 1 D. finish (v) /ˈfiniʃ/ : động từ 2 âm tiết nhưng trọng âm rơi vào 1 vì có đuôi -ISH Vậy đáp án là A
  • 67. Question 20. A. looked B. laughed C. opened D. stepped Kiến thức : Cách phát âm đuôi -ED Giải thích: – Phát âm là /t/ khi từ có tận cùng bằng các phụ âm vô thanh: /θ/, /p/, /k/, /f/, /s/, /ʃ/, /tʃ/ – Phát âm là /id/ khi từ có tận cùng là các âm: /t/, /d/ – Phát âm là /d/ khi các từ có tận cùng là nguyên âm và các phụ âm hữu thanh còn lại Xét các đáp án: A. looked /lʊkt/ B. laughed /lɑː / C. opened /ˈəʊ.pənd/ D. stepped /stept/ Vậy đáp án đúng là C
  • 68. Question 21. A. tangle B. dangerous C. battle D. calculate Question 21. Đáp án B Kiến thức: Cách phát âm của nguyên âm Giải thích: A. tangle /ˈtæŋɡl/ B. dangerous /ˈdeɪndʒərəs/ C. battle /ˈbætl/ D. calculate /ˈkælkjuleɪt/ Vậy đáp án đúng là B
  • 69. Question 22. I knew she was only flattering me because she wanted to borrow some money. A. teasing B. threatening C. praising D. helping Kiến thức: Đồng nghĩa (từ đơn) Giải thích: Ta có: flatter (v): xu nịnh, tâng bốc, khen ngợi Xét các đáp án: - tease (v): trêu chọc - threaten (v): đe dọa - praise (v): khen ngợi, tán dương - help (v): giúp đỡ Vậy: flatter = praise Dịch: Tôi biết cô ấy chỉ đang nịnh tôi vì cô ấy muốn mượn tiền.
  • 70. Question 23. he student was ecstatic when he found out he received the highest achievable score on the test. A. excited B. worried C. tragic D. disappointed Kiến thức: Đồng nghĩa (từ đơn) Giải thích: Ta có: - ecstatic (a): ngất ngây/ vui mừng/ hào hứng Xét các đáp án: A. excited (a): hào hứng/ vui mừng B. worried (a): lo lắng C. tragic (a) bi kịch/ thảm kịch D. disappointed (a): thất vọng Vậy đáp án đúng là A ecstatic ~ excited Tạm dịch: Sinh viên vui mừng khi biết mình nhận được số điểm cao nhất có thể đạt được trong bài kiểm tra.
  • 71. Question 24. If you want to be a successful gardener, of course you’ve got to have green fingers. A. lack skills in gardening B. have a good head for gardening C. be better at doing the gardening D. make plants grow well Kiến thức: Trái nghĩa (cụm từ hoặc thành ngữ) Giải thích: Ta có thành ngữ: have green fingers: mát tay/ có tài làm vườn/ có khiếu làm vườn Xét các đáp án: A. lack skills in gardening: thiếu kỹ năng làm vườn B. have a good head for gardening: có tài/ khiếu làm vườn C. be better at doing the gardening: giỏi làm vườn hơn D. make plants grow well: làm cho cây cối phát triển tốt Vậy đáp án đúng: A have green fingers >< lack skills in gardening Tạm dịch: Nếu bạn muốn trở thành một người làm vườn thành công, tất nhiên bạn phải có khiếu làm vườn.
  • 72. Question 25. 2008 is a memorable year for people around the world because of the global financial recession that hit practically every sector of world economy. A. prosperity B. downturn C. crisis D. depression Kiến thức: Trái nghĩa (từ đơn) Giải thích: Ta có: Recession (n) sự khủng hoảng Xét các đáp án: A. Prosperity (n): sự thịnh vượng (về kinh tế) B. crisis (n) cơn khủng hoảng C. Downturn (n): sự suy sụp (trog hoạt động kinh tế) D. depression (n) sự trì trệ Vậy đáp án đúng là A recession = crisis >< prosperity Tạm dịch: 2008 là năm đáng nhớ với mọi người trê toàn thế giới vì sự khủng hoảng kinh tế toàn cầu, cái mà đã đánh thẳng vào các ngành kinh tế thế giới.
  • 73. Question 26. Linda's final exam is important. She realizes little of it A. Under no circumstances does Linda realize how important her final exam is. B. Rarely does Linda realize how important her final exam is. C. Little does Linda realize how important her final exam is. D. Never does Linda realize how important her final exam is. Kiến thức: Kết hợp câu – đảo ngữ Giải thích: Cấu trúc: Under no circumstances/rarely/little/never + mệnh đề đảo ngữ Mệnh đề đảo ngữ: trợ động từ/tobe/ động từ khuyết thiếu/ have/has + S + V Đề bài: Linda's final exam is important. She realizes little of it. Bài kiểm tra cuối khóa của Linda rất quan trọng. Cô ấy gần như không hiểu được điều đó. = C. Little does Linda realize how important her final exam is. Hầu như Linda không nhận ra được kỳ thi cuối cùng của cô quan trọng như thế nào.
  • 74. Question 27. I deeply regret having spoken to her so severely yesterday. She was badly hurt. A. If only I could apologize to her for having spoken to her so severely yesterday. B. I wish I hadn’t spoken to her so severely yesterday. C. She must have been badly hurt because I had spoken to her so severely yesterday. D. If I hadn’t spoken to her so severely yesterday, she wouldn’t be badly hurt. Kiến thức: Kết hợp câu – câu ước Giải thích: Câu đề bài: Tôi vô cùng hối hận vì đã nói rất nặng lời với cô ấy đêm qua. Cô ấy bị tổn thương nặng nề. Các phương án: A. Giá mà tôi có thể xin lỗi cô ấy về việc đã nói rất nặng lời với cô ấy đêm qua. (sai về cấu trúc - ở đây cần cấu trúc lời ước cho quá khứ nhưng đây lại là lời ước ở hiện tại.) B. Tôi ước tôi đã không nói rất nặng lời với cô ấy đêm qua. (đáp án này đúng - cấu trúc lời ước cho quá khứ) C. Cô ấy chắc chắn bị tổn thương nặng nề bởi vì tôi đã nói rất nặng lời với cô ấy đêm qua. (sai về thì của động từ ở mệnh đề trạng ngữ chỉ lý do – YESTERDAY – thì động từ chia ở quá khứ đơn) D. Nếu tôi đã không nói rất nặng lời với cô ấy đêm qua, cô ấy không bị tổn thương nặng nề. (Sai về cấu trúc – việc tổn thương này đã xảy ra nên ko thể là câu điều kiện hỗn hợp) Vậy đáp án đúng là B
  • 75. Question 28. We are going to visit our grandparents when we will finish our final exams. A. are going to B. our C. when D. will finish Kiến thức: Lỗi sai – Thì của động từ Giải thích: Động từ của mệnh đề chính chia ở thì tương lai gần, thì động từ của mệnh đề trạng ngữ chỉ thời gian chia ở thì hiện tại nên sai ở đáp án D. Sửa: will finish → finish Tạm dịch: Chúng tôi sẽ đi thăm ông bà khi chúng tôi hoàn thành kì thi cuối cùng.
  • 76. Question 29. Ozone has his origin in a number of sources, a prime one being the automobile engine. A. his B. prime C. being D. the Kiến thức: Lỗi sai – Đại từ nhân xưng/ tính từ sở hữu Giải thích: - Tính từ sở hữu thay thế cho danh từ “Ozone” (không phải danh từ chỉ người) là “ITS” nên sai ở đáp án A. Sửa lỗi: his → its Tạm dịch: Ozone có nguồn gốc từ một số nguồn, nguồn chính là động cơ ô tô.
  • 77. Question 30. It took the spouses five years to discover that their tastes were diverging and their tempers were incompatible. A. spouses B. tastes C. diverging D. incompatible Kiến thức: Lỗi sai – Từ vựng Giải thích: Ta có: diverging (V-ing): khác nhau/ bất đồng ý kiến divergent (a): khác nhau/ bất đồng ý kiến Dùng sau động từ TASTE + adj (linking verb + adj/n) Vậy đáp án đúng là C Sửa lỗi: diverging → divergent Tạm dịch: Hai vợ chồng đã mất 5 năm để phát hiện ra rằng sở thích của họ khác nhau và tính khí của họ không tương thích.
  • 78. Question 31. I'm sure Luisa was very disappointed when she failed the exam. A. Luisa must be very disappointed when she failed the exam. B. Luisa must have been very disappointed when she failed the exam. C. Luisa may be very disappointed when she failed the exam. D. Luisa could have been very disappointed when she failed the exam. Kiến thức: Câu đồng nghĩa – Động từ khuyết thiếu Giải thích: Must be... : dùng để diễn tả những suy luận ở hiện tại Must + have + Vp2: dùng để diễn tả những suy luận ở trong quá khứ Could + have + Vp2: dùng để diễn tả những điều có thể đã xảy ra nhưng trên thực tế là không Đề bài: Tôi chắc chắn Luisa đã rất thất vọng khi cô ấy trượt kỳ thi. = B. Luisa hẳn đã rất thất vọng khi cô thi trượt.
  • 79. Question 32. “Would you like to come out to dinner with me tonight, Jenny?” Paul said. A. Paul suggested that Jenny go out to dinner with him that night. B. Paul insisted on Jenny going out to dinner with him that night. C. Paul invited Jenny to go out to dinner with him that night. D. Pau offered Jenny to go out to dinner with him that night Question 32. Đáp án: C Kiến thức: Câu đồng nghĩa – Câu tường thuật Giải thích: Câu đề bài: “Em có muốn ra ngoài ăn tối với anh hôm nay không Jenny?” Paul hỏi. (Đây là cấu trúc dùng để mời). Xét các đáp án: A. Paul gợi ý rằng Jenny nên ra ngoài ăn tối với anh ấy hôm đó. → sai ý B. Paul nằng nặc muốn Jenny ra ngoài ăn tối với anh ấy hôm đó. → sai ý C. Paul mời Jenny ra ngoài ăn tối với anh ấy hôm đó. → đúng D. Không dịch vì sai cấu trúc: Động từ “offer” không có cấu trúc “offer sb to do sth”. Đáp án C đúng ý đề bài cho. Các đáp án còn lại sai ý hoặc sai cấu trúc.
  • 80. Question 33. he last time I ate spaghetti was five months ago. A. I didn’t eat spaghetti five months ago. B. I would eat spaghetti in five months. C. I have eaten spaghetti for five months. D. I haven’t eaten spaghetti for five months. Kiến thức: Câu đồng nghĩa – Thì của động từ Giải thích: Ta có công thức: S + haven’t/ hasn’t + V- pp + O + for + time = S + (last) + V-simple past + O + time + ago = It’s + time + since + S + (last) + V-simple past + O = The last time + S + V-simple past + O + was + time + ago Vậy đáp án đúng là D The last time I ate spaghetti was five months ago. = D. I haven’t eaten spaghetti for five months. Tạm dịch: Lần cuối cùng tôi ăn mì Ý là năm tháng trước. = D. Tôi đã không ăn mì Ý được năm tháng.
  • 81. Question 34. The 2020 Summer Games were postponed due to the coronavirus pandemic but organizers have been determined to (34) _______ they go ahead in July. A. acclaim B. ensure C. remain D. assure Kiến thức: Đọc điền từ Giải thích: Xét các đáp án: A. acclaim (v): hoan nghênh B. ensure (v): đảm bảo (+ st/ that + clause) C. remain (v): còn lại D. assure (v): đảm bảo (+ st) Dựa vào nghĩa và ngữ cảnh đáp án B là đúng Thông tin: The 2020 Summer Games were postponed due to the coronavirus pandemic but organizers have been determined to (34) ___ ensure ___ they go ahead in July. Tạm dịch: Thế vận hội mùa hè 2020 đã bị hoãn do đại dịch coronavirus nhưng các nhà tổ chức vẫn quyết tâm đảm bảo rằng họ sẽ diễn ra vào tháng 7.
  • 82. Question 35 It began in Fukushima Prefecture, (35) _______ was devastated by the triple disasters of the 2011 earthquake, tsunami and nuclear meltdown. A. who B. where C. which D. whom Kiến thức: Đọc điền từ Giải thích: A. who: thay thế cho danh từ chỉ người có chức năng làm chủ ngữ hoặc tân ngữ trong mệnh đề quan hệ. B. where: thay thế cho danh từ chỉ nơi chốn và có chức năng làm trạng từ chỉ nơi chốn trong mệnh đề quan hệ. C. which: thay thế cho danh từ chỉ vật có chức năng làm chủ ngữ hoặc tân ngữ trong mệnh đề quan hệ. D. whom: thay thế cho danh từ chỉ người có chức năng làm tân ngữ trong mệnh đề quan hệ. Ta cần đại từ quan hệ thay thế cho danh từ “Fukushima Prefecture” – danh từ chỉ nơi chốn và làm chủ ngữ trong mệnh đề quan hệ không xác định. Thông tin: It began in Fukushima Prefecture, (32) ___ which ___ was devastated by the triple disasters of the 2011 earthquake, tsunami and nuclear meltdown. Tạm dịch: Nó bắt đầu ở tỉnh Fukushima, nơi bị tàn phá bởi ba thảm họa động đất, sóng thần và thảm họa hạt nhân năm 2011.
  • 83. Question 36. although (36) _______ people say claims of recovery are a little premature. A. much B. every C. a lot D. many Question 36. Đáp án: D Kiến thức: Đọc điền từ Giải thích: A. much: nhiều (dùng với danh từ không đếm được) B. every: mọi (dùng với danh từ không đếm được) C. a lot (+ of : dùng với cả danh từ số nhiều và danh từ không đếm được) D. many: nhiều (dùng với danh từ số nhiều) Ta thấy, sau chỗ trống là danh từ số nhiều (people) nên đáp án D là đúng
  • 84. Question 37 . (37) _______ must wear face masks, and are being urged to clap rather than cheer. . A. Audiences B. Viewers C. Spectators D. Watchers Question 37. Đáp án: C Kiến thức: Đọc điền từ Giải thích: Xét các đáp án: A. Audiences: khán giả (xem ở rạp) B. Viewers: người xem/ khán giả (xem TV) C. Spectators: khán giả (xem ở sân vận động) D. Watchers: người xem Ta thấy, đoạn văn nói về thế vận hội mùa hè được tổ chức ở Nhật, nên đáp án C là đúng (spectators: khan giả xem ở sân vận động) Thông tin: Japanese people are being encouraged to look at live broadcasts of the relay and refrain from travelling to watch it amid fears of a spike in COVID 19 cases. (37) ___ Spectators ___ must wear face masks, and are being urged to clap rather than cheer. Tạm dịch: Người dân Nhật Bản đang được khuyến khích xem các chương trình phát sóng trực tiếp của cuộc tiếp sóng và hạn chế đi lại để xem nó trong bối cảnh lo ngại về sự gia tăng đột biến các trường hợp mắc COVID 19. Khán giả phải đeo khẩu trang và được khuyến khích vỗ tay hơn là cổ vũ.
  • 85. Question 38. A. in spite of B. because of C. due to D. with a view to Question 38. Đáp án: A Kiến thức: Đọc điền từ Giải thích: Xét các đáp án: A. in spite of (+ V-ing/ N.P): mặc dù B. because of (+ V- ing/ N.P): bởi vì C. due to (+ V-ing/ N.P): bởi vì D. with a view to (+ V- ing/ N.P): với ý định Dựa vào nghĩa và ngữ cảnh, đáp án đúng là A Thông tin: Japan's Prime Minister Yoshihide Suga reiterated his commitment to stage a “safe and secure” Olympics (38) ___ in spite of ___ the pandemic. Tạm dịch: Thủ tướng Nhật Bản Yoshihide Suga nhắc lại cam kết tổ chức một Thế vận hội “an toàn và bảo mật” bất chấp đại dịch.
  • 86. Question 39. What is the passage mainly about? A. The natural homes for zoos’ animals. B. The suffering zoos bring captured animals. C. The things zoo officials concern. D. The comfort for zoo visitors. Question 39: Đáp án B Kiến thức: Đọc hiểuGiải thích: Ý chính của bài là gì? Xét các đáp án: A. Ngôi nhà tự nhiên của các loài động vật trong vườn thú. B. Các vườn thú đau khổ mang theo những con vật bị bắt. C. Những điều các quan chức sở thú quan tâm. D. Sự thoải mái cho khách tham quan vườn thú. Thông tin: Zoo officials say that they are concerned about animals. However, most zoos remain "collections" of interesting "items" rather than protective habitats. Zoos teach people that it is acceptable to keep animals in captivity. However, animals in zoos are bored, limited, lonely, and far from their natural homes. Tạm dịch: Các quan chức sở thú nói rằng họ lo ngại về động vật. Tuy nhiên, hầu hết các vườn thú vẫn là "bộ sưu tập" các "vật phẩm" thú vị hơn là môi trường sống bảo vệ. Các vườn thú dạy mọi người rằng việc nuôi nhốt động vật được chấp nhận. Tuy nhiên, các loài động vật trong vườn thú cảm thấy buồn chán, hạn chế, cô đơn và xa nhà tự nhiên của chúng.
  • 87. Question 40. The word “observe” in paragraph 2 is closest in meaning to ______. A. watch B. feed C. raise D. touch Kiến thức: Đọc hiểu Giải thích: Từ “observe” trong đoạn 2 gần nghĩa nhất với ______. Observe: xem/ quan sát Xét các đáp án: A. watch: xem B. feed: nuôi C. raise: nuôi D. touch: chạm vào Vậy đáp án đúng là A Từ đồng nghĩa: observe = watch Thông tin: Most zoo areas are quite small, and visitors can rarely observe animals’ normal behavior in these unnatural spaces. Tạm dịch: Hầu hết các khu vực vườn thú đều khá nhỏ và du khách hiếm khi có thể quan sát hành vi bình thường của động vật trong những không gian phi tự nhiên này.
  • 88. Question 41. According to the passage, zoochosis is popular among animals in ______. A. the wild B. natural homes C. cages D. large spaces Question 41: Đáp án C Kiến thức: Đọc hiểu Giải thích: Theo đoạn văn, zoochosis phổ biến ở các loài động vật trong ______. Xét các đáp án: A. hoang dã B. ngôi nhà tự nhiên C. lồng D. không gian rộng lớn Thông tin: This results in abnormal and self-destructive behavior called zoochosis. A worldwide study of zoos found that zoochosis is common among animals in small spaces or cages. For example, Bears spend 30 percent of their time walking back and forth. These are all signs of distress. Tạm dịch: Điều này dẫn đến hành vi bất thường và tự hủy hoại được gọi là zoochosis. Một nghiên cứu trên toàn thế giới về các vườn thú cho thấy rằng bệnh zoochosis là phổ biến ở các loài động vật trong không gian nhỏ hoặc lồng. Ví dụ: Gấu dành 30 phần trăm thời gian của chúng để đi đi lại lại. Đây là tất cả các dấu hiệu của sự đau khổ.
  • 89. Question 42. The word “them” in paragraph 4 refers to ______. A. signs B. their habitats C. endangered animals D. zoos Kiến thức: Đọc hiểu Giải thích: Từ “them” trong đoạn 4 đề cập đến ______. Xét các đáp án: A. dấu hiệu B. môi trường sống của chúng C. động vật có nguy cơ tuyệt chủng D. vườn thú Thông tin: However, zoos that breed endangered animals, such as big cats and Asian elephants, often do not release them to the wild. Tạm dịch: Tuy nhiên, các vườn thú nuôi các loài động vật có nguy cơ tuyệt chủng, chẳng hạn như mèo lớn và voi châu Á, thường không thả chúng về tự nhiên.
  • 90. Question 43. Which of the following is TRUE according to the passage? A. Zoos educate people by showing them animals’ natural behavior. B. Zoos remain animals’ protective habitats. C. Zoos always release endangered animals to the wild. D. Zoos keep animals in small spaces Question 43: Đáp án: D Kiến thức: Đọc hiểu Giải thích: Điều nào sau đây là ĐÚNG theo đoạn văn? Xét các đáp án: A. Các vườn thú giáo dục con người bằng cách cho họ thấy hành vi tự nhiên của động vật. B. Các vườn thú vẫn là môi trường sống bảo vệ của động vật. C. Các vườn thú luôn thả các loài động vật có nguy cơ tuyệt chủng về tự nhiên. D. Vườn thú nuôi nhốt động vật trong không gian nhỏ Thông tin: The animals are kept together in small spaces, with no privacy and little opportunity for mental stimulation or physical exercise. Tạm dịch: Các con vật được nhốt chung với nhau trong những không gian nhỏ, không có sự riêng tư và ít có cơ hội để kích thích tinh thần hoặc rèn luyện thể chất.
  • 91. Question 44. Which of the following could be the main idea of the passage? A. Being optimistic is an effective way to get over bad situations. B. Keeping positive or negative thoughts is the own choice of each person. C. Positive thoughts are necessary conditions to be successful. D. There seems to have more pessimists than optimists. Kiến thức: Đọc hiểu Giải thích: Câu nào trong các câu sau đây có thể là ý chính của đoạn văn? A. Lạc quan là một cách rất hiệu quả để vượt qua các hoàn cảnh khó khăn. B. Giữ suy nghĩ tích cực hay tiêu cực là lựa chọn riêng của mỗi người. C. Suy nghĩ tích cực là điều kiện cần thiết để thành công. D. Dường như là có nhiều người bi quan hơn lạc quan. Thông tin 1: Have you ever seen people who have just endured an awful situation? Some focus on what they have lost, and this is easy to understand. But other people focus on what they did not lose, and they start thinking about a better future. Tạm dịch: Bạn đã từng thấy những người phải trải qua một tình huống tồi tệ? Vài người chỉ chăm chú vào những gì họ đã mất, và điều này cũng dễ hiểu. Nhưng những người khác chỉ tập trung vào những điều họ chưa mất, và họ bắt đầu nghĩ về 1 tương lai tốt hơn. Thông tin 2: It may sound like a cliché. While a positive attitude may not be the answer to every problem, it can certainly give you an advantage in surviving most of life’s minor setbacks. Tạm dịch: Nó nghe như 1 câu sáo rỗng. Mặc dù thái độ tích cực không phải là chìa khóa cho mọi vấn đề, nhưng nó chắc chắn sẽ cho bạn cơ hội vượt qua mọi khó
  • 92. sốngQuestion 45. The word “grief” in paragraph 3 is closest in meaning to _______. A. problem B. sorrow C. disappointment D. damage Kiến thức: Đọc hiểu Giải thích: Từ “grief” trong đoạn 3 gần nghĩa nhất với từ ___________. A. vấn đề B. nỗi buồn C. sự thất vọng D. sự thiệt hại Từ đồng nghĩa: grief (nỗi đau buồn) = sorrow Thông tin: Both have lost their homes and all their belongings in a devastating storm. One family cannot mask their grief. They feel that everything they hold dear has been destroyed. Tạm dịch: Cả 2 gia đình đều mất nhà và toàn bộ của cải trong trận bão kinh hoàng. Một gia đình không thể che giấu nỗi buồn của họ. Họ cảm thấy rằng mọi thứ họ có đã bị phá hủy hoàn toàn.
  • 93. Question 46. What does the word “they” in paragraph 3 refer to? A. tragic events B. all of the people C. the first family D. the second family Kiến thức: Đọc hiểu Giải thích: Từ “they” trong đoạn 3 đề cập đến từ nào? A. thảm kịch B. tất cả mọi người C. gia đình thứ nhất D. gia đình thứ hai Thông tin: However, the second family certainly seems to be better off. They are thinking about making progress rather than focusing on the tragic events. Tạm dịch: Tuy nhiên, gia đình thứ 2 chắc chắn có vẻ tốt hơn. Họ đang suy nghĩ về việc làm sao để cải thiện tình hình chứ không tập trung vào thảm kịch.
  • 94. Question 47. It can be inferred from the third paragraph that _______. A. disappointment and sadness are all people’s common emotions in terrible situations. B. your attitude in terrible situations is more important than how serious the problems are. C. optimists often suffer less terrible situations than pessimists. D. your attitude will decide the way you react to terrible situations. Kiến thức: Đọc hiểu Giải thích: Có thể suy ra từ đoạn 3 rằng _______. A. Sự thất vọng và nỗi buồn là cảm xúc chung của tất cả mọi người trong hoàn cảnh khó khăn. B. Thái độ của bạn trong các hoàn cảnh khó khăn thì quan trọng hơn là việc vấn đề đó nghiêm trọng đến mức nào. C. Người lạc quan thường ít gặp phải các hoàn cảnh khó khăn hơn người bi quan. D. Thái độ của bạn sẽ quyết định cách bạn phản ứng lại các hoàn cảnh khó khăn. Thông tin: Imagine two families: Both have lost their homes and all their belongings in a devastating storm. One family cannot mask their grief. They feel that everything they hold dear has been destroyed. They cannot imagine how they will ever be able to replace things and start over again. Their normal life seems to have been completely lost. In contrast, a second family is crying with joy. All of the people in their family are unharmed and safe. This family is just happy that everyone has survived. This family is already trying to figure out how they can recover. You can’t really blame the first family for experiencing a very normal reaction to a terrible situation. However, the second family certainly seems to be better off. They are thinking about making progress rather
  • 95. Question 48. The word “scenario” in paragraph 4 mostly means _______. A. trouble B. background C. circumstance D. imagination Question 48. Đáp án C Kiến thức: Đọc hiểu Giải thích: Từ “scenario” trong đoạn 4 có nghĩa là _______. A. khó khăn B. bối cảnh C. tình huống D. sự tưởng tượng Từ đồng nghĩa: scenario (kịch bản, hoàn cảnh, tình huống) = circumstance Thông tin: Though this scenario is extreme, everyone experiences setbacks that seem just awful at the time. Tạm dịch: Mặc dù tình huống này hơi cực đoan, nhưng dường như ai cũng có lúc trải qua khó khăn trong cuộc đời.
  • 96. Question 49. According to the passage, which of the following is NOT true? A. The thing people have to remember is managing their own reaction to bad situations. B. Everyone will suffer some terrible experiences in their life. C. Paying attention to the solutions of the setback is better than focusing on the damage it causes. D. To have a good foundation for the future, you should not undergo bad situations today. Question 49. Đáp án D Kiến thức: Đọc hiểu Giải thích: Theo đoạn văn, câu nào là KHÔNG đúng? A. Điều mà mọi người phải nhớ là kiểm soát phản ứng của mình trong các hoàn cảnh khó khăn. B. Mỗi người cũng sẽ trải qua các khó khăn trong cuộc đời của họ. C. Tập trung vào các giải pháp giải quyết khó khăn tốt hơn là chăm chú vào các thiệt hại do khó khăn gây ra. D. Để có 1 nền tảng tốt trong tương lai, bạn không nên trải qua các hoàn cảnh khó khăn hôm nay. Thông tin 1: One good piece of advice to remember is that you cannot always control situations or other people. The only thing you can control is your own personal reaction to bad situations. Thông tin 2: Nobody gets through life without having some bad things happen. In these situations, try to focus on the steps you can take to remedy the situation, instead of how awful the setback is. By doing this, you will be laying the foundation for a better tomorrow. And you will not suffer as much pain today. (Đoạn 4)
  • 97. Question 50. According to paragraph 5, what is the major thing you should do when you have troubles? A. Be optimistic and make out a plan. B. Ask other people for help when necessary. C. Control your emotions. D. Determine how serious the problem is. Kiến thức: Đọc hiểu Giải thích: Theo đoạn 5, điều cốt yếu bạn nên làm khi gặp khó khăn là gì? A. Hãy lạc quan và vạch ra 1 kế hoạch. B. Hãy nhờ người khác giúp đỡ khi cần thiết. C. Hãy kiểm soát cảm xúc của bạn. D. Hãy xác định xem khó khăn của bạn nghiêm trọng tới mức nào. Thông tin: The bottom line is, no matter what the problem is, you are more likely to fix it if you can stay positive and work out a plan. Tạm dịch: Điểm mấu chốt là, cho dù khó khăn của bạn là gì đi nữa, bạn đều có thể vượt qua được nếu bạn luôn lạc quan và vạch ra 1 kế hoạch.
  • 98. Question 1: Lan wants to learn English because it is an____________ language. A. internationally B. international C. internationalize D. internationalism A. Internationally /inte’næ∫nəli/ (adv): cấp độ toàn thế giới B. International /,intə’næ∫nəl/ (a): thuộc về quốc tế C. Internationalize /intə’næʃnəlaiz/ (v): quốc tế hóa D. Internationalism /intə’næ∫nəlizəm/ (n): chủ nghĩa quốc tế Căn cứ vào danh từ “language” thì vị trí còn trống cần một tính từ để tạo thành một cụm danh từ. Ta loại được các phương án A, C, D.
  • 99. Question 2: Let’s go to the cinema to watch “End game”, _______? A. will you B. don’t you C. shall we D. don’t we Dịch nghĩa: “Chúng ta cùng đi đến rạp chiếu phim để xem bộ phim “End game”, phải không nào?” Câu bắt đầu với let’s có ý rủ rê thì thành lập hỏi đuôi sẽ dùng shall we.
  • 100. Question 3. He will apply for a job ________ . A. when he is graduating from university B. until he graduated from university. C. after he had graduated from university D. as soon as he graduates from university. Kiến thức về mệnh đề trạng ngữ Xét các đáp án ta thấy: - Động từ của vế câu cho trước ở trong đề bài ở thì tương lai → diễn tả một hành động chưa xảy ra hay sẽ xảy ra ở trong tương lai nên ở vế sau, không thể chia động từ ở thì quá khứ, quá khứ tiếp diễn hay quá khứ hoàn thành. → loại A, B, C → 4 đáp án D đúng (phối thì hiện tại đơn với tương lai đơn) Dịch: Anh ta sẽ đi xin việc ngay sau khi anh ta tốt nghiệp
  • 101. Question 4: Not many people are aware ____ male preference in this company. A. with B. on C. about D. of * Dịch nghĩa: Không nhiều người nhận thức được sự ưu tiên nam giới trong công ty này. * Căn cứ vào cấu trúc: - to be aware of: nhận thức về.
  • 102. Question 5: They had to wait ten minutes for the anesthetic to take________ before they stitched up the cut. A. effect B. impact C. influence D. affect A. effect /1 fekt/ (n): có kết quả, có hiệu lực, có tác dụng ánh huởng B. impact /impækt/ (n). có lác động, ảnh huờng mạnh mé (thường là cái gi mơi mé) C. influence/ influans/ (n): sức mạnh, ảnh huởng, tác dộng đến ai D. affect /e Tekl/ (v) tác động, ảnh hưởng dễn ai *Cụm từ: Take offect. phát tác, có tác dung (thuốc thang) hoặc tao ra.dat dược kết quả như mong quôn Tạm dịch: Họ phải dợi mười phút để thuốc mê có hiệu luc trước khi khâu vét cát.
  • 103. Question 6: ____ many international agreements affirming their human rights, women are still much more likely than men to be poor and illiterate. A. Although B. Despite C. Because D. Since Dịch nghĩa: Mặc dù nhiều xác nhận các thỏa thuận quốc tế quyền con người của họ, nữ giới vẫn nhiều hơn nam giới về số lượng nghèo và mù chữ. Xét các đáp án: A. Although + clause: mặc dù B. Despite + N/Ving: mặc dù C. Because + clause: bởi vì D. Since = because + clause: bởi vì *Note: “affirming” là danh động từ, có chức năng là một danh từ chứ không phải Ving, cùng với “agreements” tạo thành cụm danh từ, và cả cụm “many….rights” là một cụm danh từ hoàn thiện, do đó ta chọn B.
  • 104. Question 7: “What is the groom wearing?” “He dresses in a _______________.” A. light suit summer B. light summer suit C. suit summer light D. summer suit light *Theo quy tắc trật tự tính từ trong câu: OSASCOMP light (a) - C (color) *Note: summer suit (phr.v): trang phục/ bộ com-lê mùa hè - Tính từ luôn đứng trước danh từ để bổ nghĩa cho danh từ Tạm dịch: “Chú rể sẽ mặc gì?” – “Anh ấy mặc bộ com-lê mùa hè màu nhạt”
  • 105. Question 8: Can you tell me how the problem _________________ in the first place? A. fell through B. brought round C. got out D. came about A. fell through: thất bại B. brought round: làm cho tỉnh C. got out: rời nhà, đi chơi với D. came about: xảy đến, xảy ra Dịch nghĩa: Bạn có thể cho tôi biết ngay từ đầu vấn đề đã xảy ra như thế nào không?
  • 106. Question 9: When we went out, the sun _______. A. was shining B. is shining C. shines D. shone Kiến thức: Thì quá khứ tiếp diễn Giải thích: Dấu hiệu: “when” (khi), động từ ở mệnh đề trạng ngữ chỉ thời gian ở thì quá khứ đơn => loại B, C (vì chia thì hiện tại). Thì quá khứ tiếp diễn dùng để diễn tả một hành động đang diễn ra tại một thời điểm trong quá khứ (chia quá khứ tiếp diễn), nếu có hành động khác xen vào thì chia hành động đó ở quá khứ đơn. Công thức: S + was/were Ving. Phối hợp thì: When + S + V-quá khứ đơn, S + was/were + V_ing. Tạm dịch: Khi chúng tôi đi ra ngoài, mặt trời đã chói chang.
  • 107. Question 10: She will have finished the preparations for the meeting _______. A. by the time her boss arrives B. after her boss had arrived C. as soon as her boss had arrived D. when her boss arrived Kiến thức: Mệnh đề trạng ngữ chỉ thời gian Giải thích: Do mệnh đề chính thì tương lai nên mệnh đề trạng ngữ chỉ thời gian ở thì hiện tại => loại B, C, D (vì chia quá khứ). Công thức: By the time S + V(s/es), S + will have Ved/V3. Tạm dịch: cô ta sẽ hoàn thành công việc chuẩn bị cho cuộc họp vào lúc sếp đến.
  • 108. Question 11: The two people________badminton seemed to be at it quite intensely. A. going B. playing C. doing D. practicing A. go /gou/ (v) dùng trước tên của các môn thể thao thuờng có dang "Ving B. play /pler/ (v); dùng trước tên của các môn thể thao đồng đội C. do /du / dùng trước tên của các môn thể thao cá nhân D. practise / præktıs/ (v): thực hành, luyện tập Tạm dịch: Hai người đang chơi cầu lông ở đó có vẻ khá mãnh liệt.
  • 109. Question 12. She is a kind of woman who does not care much of work but generally ________ meals, movies or late nights at a club with her colleagues. A. supposes B. discusses C. attends D. socializes Kiến thức về từ vựng A. suppose (v): giả sử B. discuss (v): bàn luận C. attend (v): tham dự D. socialize (v): hoà nhập với ai + with Dịch: Cô ấy là kiểu phụ nữ không quan tâm nhiều đến công việc mà thường giao lưu ăn uống, xem phim hay đi ăn khuya tại câu lạc bộ với đồng nghiệp.
  • 110. Question 13: Many people ______ against COVID-19 last year. A. vaccinated B. were vaccinating C. were vaccinated D. vaccinate Kiến thức: Câu bị động Giải thích: Người không thể tự thực hiện hành động “vaccinate” (tiêm vắc xin), mà là do bác sĩ tiêm => động từ ở dạng bị động => loại A, B, D Câu bị động thì quá khứ đơn: S + was/were Ved/V3. vaccinate => vaccinated Tạm dịch: Nhiều người đã được tiêm vắc xin phòng chống COVID-19 năm ngoái.
  • 111. Question 14: _______ all the exercises, I went out with my best friend. A. To do B. Having done C. Being done D. Had done Kiến thức: Rút gọn mệnh đề đồng ngữ Giải thích: 2 mệnh đề có cùng chủ ngữ (she), có thể rút gọn một mệnh đề về dạng: - Ving: khi mệnh đề dạng chủ động và hai hành động xảy ra liên tiếp - Ved: khi mệnh đề dạng bị động - Having Ved/V3: khi mệnh đề dạng chủ động, hành động được rút gọn xảy ra trước và là nguyên nhân dẫn đến hành động còn lại => Dạng bị động: Having been Ved/V3. Câu đầy đủ: I had done all the exercises. I. went out with my best friend. Câu rút gọn: Having done all the exercises, I went out with my best friend. Tạm dịch: Sau khi hoàn thành tất cả các bài tập, Tôi đi chơi với bạn thân của mình.
  • 112. Question 15: My neighbor is driving me mad! It seems that the later it is at night, __________ he plays his music! A. the more loud B. less C.the more loudly D. the louder Dịch nghĩa: Hàng xóm của tôi đang khiến tôi phát điên! Có vẻ như càng về đêm, anh ấy càng chơi nhạc to hơn! Xét các đáp án: A. the more loud → Sai cấu trúc so sánh hơn (loud là từ 1 âm tiết) B. less → Dựa vào nghĩa ta thấy câu B sai C. the more loudly → Sai cấu trúc so sánh hơn (late là từ 1 âm tiết) D.the louder → Cấu trúc so sánh kép khi nói về 2 người hoặc sự vật: The more/-er + S + V, the more/-er + S + V
  • 113. Question 16: - Kate: How lovely your cats are! - David: “ ________ ” A. I love them, too B. Thank you, it is nice of you to say so C. Can you say it again D. Really? They are Kiến thức: Hội thoại giao tiếp A. Tôi cũng yêu chúng. B. Cảm ơn bạn, bạn thật tử tế khi nói như vậy. C. Bạn có thể nói lại được không. D. Thật sao? Chúng dễ thương thật. Tạm dịch: - Kate :”Lũ mèo của bạn đáng yêu quá!”. - David: “Cảm ơn bạn, bạn thật tử tế khi nói như vậy”.
  • 114. Question 17: -John: “Everyone should learn more about how to treat the environment well." - Jack: “________ ” A. That's not true B. I am sure about that. C. I don't think so. D. It's not true Kiến thức: Hội thoại giao tiếp A. Điều đó không đúng B. Tôi chắc chắn về điều đó C. Tôi không nghĩ vậy D. Nó không đúng Tạm dịch: - John: “Mỗi người nên tìm hiểu nhiều hơn về cách đối xử tử tế với môi trường” - Jack: “Tôi không nghĩ vậy”.
  • 115. Question 18: A. sacrifice B. understand C. integrate D. recognize Kiến thức: Trọng âm A. /'sækrɪfaɪs/, trọng âm rơi vào âm tiết thứ 1 B. /,ʌdə'stænd/, trọng âm rơi vào âm tiết thứ 2 C. /’ɪntɪgreɪt/, trọng âm rơi vào âm tiết thứ 1 D. /’rekəgnaɪz/, trọng âm rơi vào âm tiết thứ 1 Question 19: A. social B. mature C. secure D. polite Kiến thức: Trọng âm A. /'səufəl/ trọng âm rơi vào âm tiết thứ 1 B. /mə’tʃuə/ trọng âm rơi vào âm tiết thứ 2 C. /sɪ’kuə/ trọng âm rơi vào âm tiết thứ 2 D. /pə’laɪt/ trọng âm rơi vào âm tiết thứ 2
  • 116. Question 20: A. coughed B. crooked C. cooked D. laughed Kiến thức: phát âm đuôi ed A. /kɔft/ B. /'krukɪd/ C. /kukt/ D. /lɑ:ft/ Question 21: A. band B. hand C. sand D. bank Kiến thức: Phát âm A. /bænd/ B. /hænd/ C. /sænd/ D. /bæŋk/
  • 117. Question 22: The guards were ordered to get to the king's room on the double. A. in a larger number B. very quickly C. on the second floor D. every two hours Question 23: We can use either verbal or non-verbal forms of communication. A. using verbs B. using facial expressions C. using speech D. using gesture Kiến thức: Cụm từ • on the double: nhanh chóng, không trì hoãn = very quickly Tạm dịch: lính canh được lệnh đến phòng vua ngay lập tức Kiến thức: Từ vựng A. sử dụng động từ B. sử dụng nét mặt C. sử dụng lời nói D. sử dụng cử chỉ • Verbal (adj): (thuộc) lời nói = using speech Tạm dịch: Chúng ta có thể sử dụng các hình thức giao tiếp bằng lời nói hoặc không lời.
  • 118. Question 24: You should put yourself on the back for having achieved such a high score in the graduation exam. A. wear a backpack B. praise yourself C. criticize yourself D. check up your back Kiến thức: Cụm từ A. đeo ba lô B. khen ngợi bản thân C. phê bình bản thân D. kiểm tra lưng của bạn • Put yourself on the back: tự hào về bản thân >< criticize yourself Tạm dịch: Bạn nên tự hào về bản thân vì đã đạt được số điểm cao như vậy trong kỳ thi tốt nghiệp. Question 25: The new policy will help generate more jobs. A. produce B. bring out C. form D. terminate. Kiến thức: Từ vựng A. Produce = bring out = form: sản xuất • Generate (v): Sản xuất, tạo ra >< Terminate (v): chấm dứt Tạm dịch: Chính sách mới sẽ giúp tạo ra nhiều việc làm hơn
  • 119. Question 26: She didn’t read the reference books. She wouldn’t be able to finish the test. A. Had she read the reference books, she would have been able to finish the test. B. If she had read the reference books, she could finish the test. C. Although she didn’t read the reference books, she was able to finish the test. D. Not having read the reference books, she couldn’t finish the test. Dịch nghĩa: Cô đã không đọc những cuốn sách tham khảo. Cô ấy sẽ không thể hoàn thành bài kiểm tra. Xét các đáp án: A. Had she read the reference books, she would have been able to finish the test. → Trong đảo ngữ với câu điều kiện loại 3 ta sử dụng cấu trúc: If-clause = Had + S + Vp2. B. If she had read the reference books, she could finish the test. → Sai cấu trúc đảo ngữ với câu điều kiện loại 3. C. Although she didn’t read the reference books, she was able to finish the test. → Mặc dù cô ấy không đọc sách tham khảo, cô ấy đã có thể hoàn thành bài kiểm tra, sai nghĩa. D. Not having read the reference books, she couldn’t finish the test. → Sai về cách dùng của của couldn’t. Vì “could” là quá khứ của “can”-dùng để diễn tả khả năng có thể làm gì như một bản năng, thường là do bẩm sinh mà có. Trong khi đó, câu gốc đang dùng “be able to Vo”- dùng để diễn tả một khả năng có thể làm gì do cố gắng, nỗ lực mới có được
  • 120. Question 27. She helped us a lot with our project. We couldn’t continue without her. A. Unless we had her contribution, we could continue with the project. B. But for her contribution, we could have continued with the project. C. If she hadn’t contributed positively, we couldn’t have continued with the project. D. Provided her contribution wouldn’t come, we couldn’t continue with the project. Kiến thức: Câu điều kiện loại 3 Giải thích: Câu điều kiện loại 3 diễn tả điều giả định ngược với quá khứ Cấu trúc: If + S + had Ved/ V3, S + would have Ved/ V3 = But for/ Without + noun phrase, S + would have Ved/ V3 Tạm dịch: Cô ấy giúp chúng tôi rất nhiều với dự án. Chúng tôi không thể thành công nếu không có cô ấy. A. sai ngữ pháp: câu điều kiện loại 2 => loại 3 B. Nếu không vì những giúp đỡ của cô ấy, chúng tôi sẽ đã tiếp tục với dự án. => sai nghĩa C. Nếu cô ấy không cống hiến 1 cách tích cực, chúng tôi sẽ đã không thể tiếp tục dự án. => đúng D. sai ngữ pháp: câu điều kiện loại 2 => loại 3
  • 121. Question 28: The wooden fence surrounded the factory is beginning to fall down because of the rain A. wooden B. surrounded C. to fall down D. the rain Kiến thức: rút gọn mệnh đề quan hệ • Vì câu mang nghĩa chủ động nên rút gọn mệnh đề quan hệ về dạng Ving. Sửa: Surrounded → surrounding Tạm dịch: Hàng rào gỗ bao quanh nhà máy đang bắt đầu đổ vì trời mưa Question 29: Experts in climatology and other scientists are becoming extreme concerned about the changes to our climate which are taking place. A. in B. extreme C. about D. are Kiến thức: Từ loại • Trạng từ đứng trước tính tử để bổ nghĩa cho tính từ đó. Sai: extreme → extremely Tạm dịch: Chuyên gia khí hậu học và những nhà khoa học khác đang cực kì lo ngại về những thay đổi đang diễn ra đối với khí hậu của chúng ta.
  • 122. Question 30: Her passion for helping people has motivated her to found his own charity organization. A. for B. has C. his D. organization Kiến thức: sự hòa hợp giữa chủ ngữ và động từ • Chủ ngữ là “her passion” → chia ngôi thứ 3 số ít. Sửa: his → her Tạm dịch: Niềm đam mê giúp đỡ người khác đã thúc đẩy cô ấy thành lập tổ chức từ thiện của riêng mình.
  • 123. S + have/ has + not + Ved/3………before  It/This/That is the first time + S + have/ has + Ved/3 Question 31: I have never read such a good book before. A. This is the first time I read a good book . B. This is the first time I have read a good book. C. This book is the best I have never read. D. This book is the best I read . Kiến thức: biến đổi thì Sử dụng công thức:
  • 124. Question 32: “He should to go to bed before 11p: m everyday”, I said to him. A. I advised him to go to bed before 11p: m everyday B. He shouldn’t go to bed before 11p: m everyday C. I prayed him to to go to bed before 11 p: m everyday D. I ordered him you should stay in bed. Kiến thức câu trực tiếp ở trần thuật Advise sb to V = had better + V+ St / should + V +St Question 33: Smoking is not allowed in the museum. A. You mustn’t smoke in the museum. B. You can smoke in the museum C. You don’t have to smoke in the museum D. You may smoke in the museum Kiến thức Modal verbs với inf to V Do “is not allowed” = must not
  • 125. Question 34: According to a new report, today's generation of children are in danger of getting so little sleep that they are putting their mental and physical health at (34) ________ . A. risk B. threat C. danger D. jeopardy Kiến thức: Cụm từ • put sth at risk: đặt cái gì vào tình trạng nguy hiểm Tạm dịch: Theo một báo cáo mới, thế hệ trẻ em ngày nay có nguy cơ ngủ ít đến mức khiến sức khỏe tinh thần và thể chất của chúng gặp nguy hiểm
  • 126. Question 35: Adults can easily survive on seven to eight hours' sleep a night, (35) ________ teenagers require nine or ten hours. A. so B. or C. whereas D. because Kiến thức: Từ vựng A. vì thế B. hoặc C. trong khi D. bởi vì Tạm dịch: Người lớn có thể dễ dàng tồn tại khi ngủ từ bảy đến tám giờ mỗi đêm, trong khi thanh thiếu niên cần chín hoặc mười giờ
  • 127. Question 36: Research has shown that losing as little as half an hour's sleep a night can have profound (36) ________ on how children perform the next day A. effective B. effectively C. affect D. effect Kiến thức: Từ vựng Vị trí cần điền đứng sau tính từ nên sẽ là một danh từ • Effect (n): tác động, ảnh hưởng Tạm dịch: Nghiên cứu đã chỉ ra rằng mất ngủ ít nhất nửa tiếng mỗi đêm có thể ảnh hưởng sâu sắc đến hoạt động của trẻ vào ngày hôm sau
  • 128. Question 37: A good night's sleep is also crucial for teenagers because it is while they are asleep (37) ________ they release a hormone that is essential for their 'growth spurt' (the period during teenage years when the body grows at a rapid rate). A. that B. which C. when D. where Kiến thức: Câu chẻ • Câu chẻ hay còn được gọi là câu nhấn mạnh. Chúng được sử dụng khi muốn nhấn mạnh vào một đối tượng hay sự việc nào đó. • It is/was + trạng ngữ + that + S + V + O Tạm dịch: Một giấc ngủ ngon cũng rất quan trọng đối với thanh thiếu niên vì chính trong khi ngủ, chúng tiết ra một loại hormone cần thiết cho quá trình tăng trưởng nhanh (giai đoạn ở tuổi thiếu niên khi cơ thể phát triển với tốc độ nhanh).
  • 129. Question 38: It's true that they can, to(38) ________ extent, catch up on sleep at weekends, but that won't help them when they are dropping off to sleep in class on a Friday afternoon. A. any B. some C. one D. every Kiến thức: Cụm từ • To some extent: ở một phạm vi, mức độ, chừng mực nào đó Tạm dịch: Đúng là ở một mức độ nào đó, chúng có thể ngủ bù vào cuối tuần, nhưng điều đó sẽ không giúp ích gì khi chúng ngủ gật trong lớp vào buổi chiều thứ 6
  • 130. 39. What is the objective of the things Dr. Wray has done? A. To make the environment less polluted and to have children from her marriage with her husband. B. To help people feel less lonely and overcome the bad emotions from climate anxiety. C. To describe and demonstrate the different kinds of eco-distress. D. To provide people in some remote areas with the access to the counseling for climate grief. Kiến thức: Đọc hiểu Giải chi tiết: Mục tiêu của những việc Tiến sĩ Wray đã làm là gì? A. Để môi trường bớt ô nhiễm và có con từ cuộc hôn nhân với chồng. B. Để giúp mọi người cảm thấy bớt cô đơn và vượt qua những cảm xúc tồi tệ do lo lắng về khí hậu. C. Để mô tả và chứng minh các loại đau đớn sinh thái khác nhau. D. Cung cấp cho người dân ở một số vùng sâu vùng xa khả năng tiếp cận với dịch vụ tư vấn về các vấn đề khí hậu. Thông tin: "My overall goal is to help people feel less alone," Dr. Wray said. "We need to restore ourselves so we don't burn out and know how to be in this crisis for the long haul that it is."
  • 131. Question 40. What does the word "prevalence" in paragraph 3 mean? A. impact B. appearance C. popularity D. recognition Kiến thức: Đọc hiểu Giải chi tiết: Từ "prevalence" trong đoạn 3 có nghĩa là gì? A. impact (n): tác động B. appearance (n): sự xuất hiện C. popularity (n): sự phổ biến D. recognition (n): sự công nhận => prevalence (n): sự phổ biến = popularity Thông tin: But as the prevalence of climate anxiety has grown, so has the number of people working to alleviate it, both for themselves and those around them. Tạm dịch: Nhưng khi sự phổ biến của chứng lo âu về khí hậu ngày càng gia tăng, thì số lượng người làm việc để giảm bớt nó, cho cả bản thân và những người xung quanh cũng tăng lên.
  • 132. Question 41. According to the passage, which of the following is TRUE? A. Jennifer Atkinson couldn't sleep for fear of a mass extinction. B. More and more people are worried about the climate change's effect on their mental health C. The anxiety about climate change is not widespread in less wealthy countries. D. People can easily access the consultancy for climate distress in most parts of the world. Kiến thức: Đọc hiểu Giải chi tiết: Theo đoạn văn, điều nào sau đây là ĐÚNG? A. Jennifer Atkinson không thể ngủ vì lo sợ về sự tuyệt chủng hàng loạt. B. Ngày càng có nhiều người lo lắng về ảnh hưởng của biến đổi khí hậu đối với sức khỏe tâm thần của họ. C. Sự lo lắng về biến đổi khí hậu không phổ biến ở các nước kém giàu có hơn. D. Mọi người có thể dễ dàng tiếp cận tư vấn về các vấn đề khí hậu ở hầu hết các nơi trên thế giới. Thông tin: But as the prevalence of climate anxiety has grown, so has the number of people working to alleviate it, both for themselves and those around them.
  • 133. Question 42. What is the main idea of the passage? A. Types of eco-distress B. When eco-distress affects human beings C. The risk of an impending mass extinction D. How specialists help address eco-distress Kiến thức: Đọc hiểu Giải chi tiết: Ý chính của đoạn văn là gì? A. Các dạng khó khăn về sinh thái B. Khi tình trạng khó khăn về hệ sinh thái ảnh hưởng đến con người C. Nguy cơ sắp xảy ra tuyệt chủng hàng loạt D. Cách các chuyên gia giúp giải quyết vấn đề sinh thái Thông tin: Distress over global warming is increasing, but formal and informal support networks are springing up, too… "My overall goal is to help people feel less alone," Dr. Wray said. "We need to restore ourselves so we don't burn out and know how to be in this crisis for the long haul that it is." Dr. Atkinson, in hopes of assuaging her feelings and those of her students, designed a seminar on eco-grief and climate anxiety.
  • 134. Question 43. What does the word "they" in paragraph 1 refer to? A. activists B. children C. humanities D. students Kiến thức: Đọc hiểu Giải chi tiết: Từ "they" trong đoạn 1 chỉ điều gì? A. nhà hoạt động B. trẻ em C. nhân văn D. sinh viên Thông tin: Jennifer Atkinson, an associate professor of environmental humanities at the University of Washington, Bothell, became depressed after students told her they couldn't sleep because they feared social collapse or mass extinction. Tạm dịch: Jennifer Atkinson, phó giáo sư về nhân văn môi trường tại Đại học Washington, Bothell, trở nên trầm cảm sau khi sinh viên nói với cô rằng họ không thể ngủ được vì sợ xã hội sụp đổ hoặc tuyệt chủng hàng loạt.
  • 135. Question 44. What is this passage mainly about? A. gardens in Madrid, New York, and Chicago B. people growing plants on roofs and walls C. private gardens in the city's unused spaces D. how to grow your own food in the city 44. Đoạn này chủ yếu là về? A. những khu vườn ở Madrid, New York và Chicago B. người trồng cây trên mái nhà và tường C. những khu vườn riêng trong những không gian chưa được sử dụng của thành phố D. cách tự trồng lương thực trong thành phố Có thể thấy cả bài viết này nói về những người trồng cây trên mái nhà và trên những bức tường. Các đáp án còn lại không chính xác hoặc không bao quát được ý chính của cả bài.
  • 136. Question 45. According to the passage, people ________ as cities have little space for gardens. A. grow plants in their apartments B. go to the countryside at the weekends C. paint their walls and roofs green D. grow trees and flowers on top of roofs Theo đoạn văn, mọi người ________ vì các thành phố có rất ít không gian cho vườn. A. trồng cây trong căn hộ của họ B. về quê vào cuối tuần C. sơn tường và mái nhà màu xanh lá cây D. trồng cây và hoa trên mái nhà Thông tin này nằm ở câu thứ hai và câu thứ ba của đoạn đầu tiên: "With limited space for parks and gardens... One creative solution is to grow plants on unused areas like walls or rooftops." (Với không gian dành cho công viên và vườn có hạn... Một giải pháp sáng tạo là trồng cây ở những khu vực không sử dụng đến như tường hay mái nhà)
  • 137. Question 46. Why is the green wall in Madrid mentioned in the passage? A. It is the most expensive green wall in the world. B. It is located on the side of a government building. C. It is a famous example of a green wall. D. It was made to provide jobs for homeless people. Tại sao bức tường xanh ở Madrid lại được nhắc đến trong đoạn văn? A. Đó là bức tường xanh đắt nhất thế giới. B. Nó nằm bên hông một tòa nhà chính phủ. C. Đó là một ví dụ nổi tiếng về bức tường xanh. D. Nó được tạo ra để cung cấp việc làm cho những người vô gia cư. Thông tin này có thể được tìm thấy ở câu thứ 3 của đoạn 2: "The CaxiaForum art gallery in Madrid, Spain, is a famous example - one of its walls is covered with 15,000 plants from over 250 different species." (Viện bảo tàng CaxiaForum ở Madrid, Tây Ban Nha là một ví dụ nổi bật - một trong những bức tường ở đó được bao phủ bởi 15,000 cây với hơn 250 loài khác nhau). Do đó "bức tường xanh" ở Madrid được nhắc đến như một ví dụ nổi tiếng về bức tường xanh".
  • 138. Question 47. Which is NOT mentioned as a benefit of a rooftop garden? A. It becomes a park that the community can use. B. Having one might mean paying less taxes. C. Children can use it to learn about the environment. D. Growing plants on a roof keeps buildings cooler. Điều nào KHÔNG được đề cập như một lợi ích của khu vườn trên sân thượng? A. Nó trở thành một công viên mà cộng đồng có thể sử dụng. B. Có một có thể có nghĩa là phải trả ít thuế hơn. C. Trẻ em có thể sử dụng nó để tìm hiểu về môi trường. D. Trồng cây trên mái nhà giúp các tòa nhà mát mẻ hơn. Đáp án B nằm ở thông tin đoạn 3: Many cities offer tax discounts to businesses with these features. (Nhiều thành phố giảm giá thuế cho các doanh nghiệp có các tính năng này.) Đáp án C nằm ở thông tin đoạn 4: In addition to saving the school money, teachers and parents love the gardens because of their educational value - it's a fun and healthy way for their kids to investigate the world around them. (Ngoài việc tiết kiệm tiền học, giáo viên và phụ huynh còn yêu thích những khu vườn vì giá trị giáo dục của chúng - đó là một cách thú vị và lành mạnh để con họ khám phá thế giới xung quanh.) Đáp án D thông tin nằm ở đoạn 3: Using plants to cover walls and rooftops can also keep cities cooler in the summer. (Sử dung thực vật để che phủ các bức tường và mái nhà cũng có thể giữ cho các thành phố mát mẻ hơn vào mùa hè.) → Chọn đáp án A
  • 139. Question 48. Because food can now be grown in cities, ________ . A. the food at expensive restaurants is cheaper B. governments are making many rules about city gardens C. farmers in the countryside are moving to the city D. there's less pollution caused by transporting food 48. Bởi vì lương thực hiện có thể được trồng ở các thành phố, ________ . A. thức ăn ở nhà hàng đắt tiền rẻ hơn B. chính phủ đang đưa ra nhiều quy tắc về vườn thành phố C. nông dân ở nông thôn đang chuyển đến thành phố D. ít ô nhiễm hơn do vận chuyển thực phẩm Thông tin này có thể được tìm thấy ở câu thứ 3 của đoạn 5: "This is based on the concept that locally grown food reduces pollution since it does not have to be transported far." (Điều này dựa trên qian niệm rằng thực phẩm được trồng tại địa phương làm giảm ô nhiễm môi trường bởi nó không cần vận chuyển xa).
  • 140. Question 49. The word "it" in paragraph 5 refers to ________ . A. a rooftop garden B. the food in general C. locally grown food D. green space Từ "it" trong đoạn 5 đề cập đến ________ . A. một khu vườn trên sân thượng B. thức ăn nói chung C. thực phẩm trồng tại địa phương D. không gian xanh It ở đây được dùng thay thế cho "locally grown food" được nhắc đến ở vế trước của cậu.
  • 141. Question 50. Why did Chef Bayless name his dish "Rooftop Salsa"? A. He got the idea while cooking on his rooftop. B. He buys the salsa from other rooftop gardeners. C. It is made from food grown in his rooftop garden. D. The money earned from the dish is given to rooftop gardeners. Tại sao đầu bếp Bayless lại đặt tên món ăn của mình là "Rooftop Salsa"? A. Anh ấy có ý tưởng khi đang nấu ăn trên sân thượng của mình. B. Anh ấy mua salsa từ những người làm vườn trên sân thượng khác. C. Nó được làm từ thực phẩm trồng trong khu vườn trên sân thượng của anh ấy. D. Số tiền kiếm được từ món ăn được đưa cho những người làm vườn trên sân thượng. Thông tin này có thể được suy ra từ câu cuối cùng của đoạn 5: "Chef Rick Bayless serves “Rooftop Salsa” at his restaurant in Chicago, USA, using only ingredients grown in his rooftop garden." (Đầu bếp Rick Bayless phục vụ món “Salsa sân thượng” ở nhà hàng của mình ở Chicago, Mỹ, chỉ sử dụng các nguyên liệu được trồng trong khu vườn trên mái nhà của mình).
  • 142.
  • 143.
  • 144.
  • 145.
  • 146. Câu 1. They are going to _______ the pool to 1.8 meter. A. deepen B. depth C. deep D. deeply Câu 1 Kiến thức về từ loại Khoảng trống cần “động từ” do phía trước khoảng trống là “be going to + Vo” Trong đó: A. deepen (v) B. depth (n) C. deep (a) D. deeply (adv)  nên ta chọn A Question 2. It’s a lovely day__________? A. isn’t it B. was it C. doesn’t it D. did it Kiến thức về câu hỏi đuôi: phía trước khẳng định của “to be” và chủ ngữ là “it”
  • 147. Question 3: The _____ time was a charm for Korea’s Pyeongchang 2018 Winter Olympic bid committee. A. third B. first C. second D. forth A. third /θɜːd/: thứ ba B. first /ˈfɜːst/: đầu tiên C. second /ˈsekənd/: thứ hai D. forth /fɔːθ/: thứ tư Ta có: Third time is a charm: lần thứ ba sẽ thành công, quá tam ba bận Tạm dịch: Lần thứ ba sẽ thành công cho ủy ban đấu thầu Olympic mùa đông Hàn Quốc Pyeongchang 2018. Question 4. Simon Lake drew the inspiration ____ La submarine of undersea travel and exploration from Twenty Thousand Leagues Under the Sea. A. at B. by C. for D. of Kiến thức về giới từ * Ta có cấu trúc sau: Draw the inspiration for sth (to sb) from sth: lấy nguồn cảm hứng cho cái gì (cho ai) từ cái gì
  • 148. Question 5. The final winner will be the one who breaks through ______ and survives till the last minutes. A. obstacles B. difficulty C. hindrance D. impediment Kiến thức về từ vựng A. obstacle /ˈɑːbstəkəl/ (n): khó khăn, trở ngại B. difficulty /ˈdɪ əlt̬i/ (n): khó khăn C. hindrance /'hindrәns/ (n): sự cản trở, ngăn cản, khó khăn D. impediment /ɪmˈpedəmənt/ (n): sự cản trở, trở ngại, khó khăn * Nhận thấy các danh từ đều thuộc cùng một trường nghĩa, tuy nhiên cả 4 đáp án đều là danh từ đếm được, trong khi trước chỗ trống trong đề bài không có mạo từ “a/an” nên vị trí này cần điền một dạng danh từ số nhiều → Loại B, C, D Tạm dịch: Người chiến thắng trong trận chung kết sẽ là người có bước đột phá để vượt qua những khó khăn trở ngại và tiếp tục tồn tại cho đến những phút giây cuối cùng. * Note: Break through sth (phr.v): vượt qua, tạo ra bước đột phá cái gì
  • 149. Question 6: Many students work to earn money ____________ their parents are rich A. because of B. despite C. however D. although Dịch nghĩa: Nhiều sinh viên làm việc để kiếm tiền mặc dù cha mẹ họ giàu có. Xét các đáp án: A. because of → Liên từ chỉ nguyên nhân, sau because of (bởi vì) là noun/noun phrase (DT/cụm DT), không phải clause (mệnh đề) B. despite → Liên từ chỉ sự đối lập, sau Despite (mặc dù) = In spite of là noun/noun phrase/V_ing (DT/cụm DT/V_ing), không phải clause (mệnh đề) C. however → Liên từ chỉ sự đối lập, sau however (tuy nhiên) là clause (mệnh đề), dịch cả câu: Nhiều sinh viên làm việc để kiếm tiền tuy nhiên cha mẹ họ giàu có. (không hợp lý) D. although → Liên từ chỉ sự đối lập, sau Although (mặc dù) = Even though = Though = In spite that là clause (mệnh đề)
  • 150. Question 7: She is going to marry a _______ man next year and they’ll give birth to two children after getting married. A. tall pretty English B. English tall pretty C. tall English pretty D. pretty tall English *Theo quy tắc trật tự tính từ trong câu: OSASCOMP : pretty - Opinion; tall - Shape; English - Origin Dịch: Cô ấy sẽ cưới một anh chàng người Anh cao đẹp năm tới và họ sẽ sinh 2 con sau khi kết hôn.
  • 151. Question 8. There are other problems of city life which I don't propose to ____ at the moment. A. go into B. go around C. go for D. go up Kiến thức về cụm động từ A. go into (phr.v): bắt đầu làm gì; thảo luận, kiểm chứng, mô tả, giải thích một cách chi tiết và thận trọng B. go around (phr.v): đủ cho mọi người trong nhóm; đến thăm ai; cư xử tệ C. go for (phr.v): chọn, say mê, cố gắng D. go up (phr.v): tăng lên Tạm dịch: Còn nhiều vấn đề khác về cuộc sống thành thị cái mà tôi không muốn đi vào sâu để thảo luận kỹ ngay lúc này.
  • 152. Question 9: When I last saw him, he________in the living room A. is sitting B. has been sitting C. was sitting D. sitting Kiến thức: Sự phối hợp về thì của động từ Giải thích: Thì quá khứ đơn và quá khứ tiếp diễn kết hợp trong câu: Diễn tả hành động đang xảy ra trong quá khứ thì một hành động khác xen vào, hành động đang xảy ra chi thì quá khứ tiếp diễn, hành động xen vào chia thì quá khứ đơn Cấu trúc: S + was/ were + V_ing + when + S + Ved/bqt Tạm dịch: Khi tôi gặp anh ta, anh ta đang ngồi trong phòng khách.
  • 153. Question 10. Richard will look for a job __________. A. after he had passed his exams B. before he passed his exams C. while he was passing his exams D. as soon as he passes his exams Kiến thức về mệnh đề trạng ngữ chỉ thời gian * Ta có mệnh đề: Richard will look for a job: chia thì tương lai đơn nên mệnh đề chỉ thời gian phải ở thì hiện Question 11: It is always difficult for designer sportswear to _____into the market because there is too much competition from leading brands like Adidas. A. invade B. cut C. break D. interfere A. invade /ɪnˈveɪd/ (v): xâm lược B. cut /kʌt/ (v): cắt C. break /breɪk/ (v): làm gãy, vỡ D. interfere /ˌɪntəˈfɪər/ (v): cản trở => Cấu trúc: break into the market: thâm nhập thị trường Dịch nghĩa: Luôn luôn khó khăn cho các nhà thiết kế đồ thể thao thâm nhập thị trường vì có quá nhiều sự cạnh tranh từ các thương hiệu hàng đầu như Adidas.
  • 154. Question 12. Though she lost her job last month, she still wanted to save_____ so she said that she had left it willingly. A. mouth B. face C. reputation D. fame Kiến thức về cụm từ cố định Ta có cụm từ: Save face (idm): giữ thể diện, danh dự cá nhân, tránh để những người khác không mất đi sự tôn trọng dành cho mình Tạm dịch: Mặc dù cô ta bị mất việc từ tháng trước, nhưng cô ta vẫn muốn gìn giữ thể diện cá nhân nên cô đã nói rằng mình tình nguyện bỏ công việc đó. Question 13. The story................... by Agatha Christie A. were written B. was written C. was written from D. wrote by Phương pháp giải: Kiến thức: Câu bị động: Giải chi tiết: Câu bị động thì Quá khứ đơn. Hành động được nhấn mạnh là câu chuyện được viết bởi Agatha Christie. Chủ ngữ là the story (số ít)=> chon B S + động từ tobe + V-ed/V3 Tạm dịch : Câu chuyện được viết bởi Agatha
  • 155. Question 14: _________the instruction,she started to do the assignment. A. Read B. Having read C.Being read D.Being done Kiến thức: Mệnh đề phân tử / Rút gọn mệnh đề đồng ngữ Giải thích: Khi 2 mệnh đề có cùng chủ ngữ (she) thì có thể rút gọn 1 trong 2 mệnh đề về dạng: - V-ing / Having P2: nếu mệnh đề được rút gọn mang nghĩa chủ động - P2 (quá khứ phân từ): nếu mệnh đề được rút gọn mang nghĩa bị động Tạm dịch: Sau khi đọc hướng dẫn, cô ta bắt đầu thực hiện bài tập.
  • 156. Question 15____ the brushwork is in Stevenson’s landscapes, the more vitality and character the painting seems to possess. A. The loose B. loose C. The loosest D. The looser Dịch nghĩa: Nét vẽ càng mềm mại trong bức tranh phong cảnh của Stevenson, thì nó càng có sức sống và có cá tính. Xét các đáp án: D. The looser the brushwork is → Cấu trúc so sánh kép khi nói về 2 người hoặc sự vật: The more/-er + S + V, the more/-er + S + V
  • 157. Question 16: Joana and David, two lectures, are talking about library skills. - Joana: “ I think we should teach our students how to use the library” - David:”_______’ A. You're absolutely wrong. B. You must be kidding. C. I couldn't agree with you more.D. That's not a good idea. Tình huống: Joana và David, 2 giảng viên, đang nói chuyện về những kỹ năng thư viện - Joana: “Mình nghĩ chúng ta nên dạy học sinh sử dụng thư viện như thế nào.” - David: “______________.” A. Bạn hoàn toàn sai. B. Chắc hẳn bạn đang đùa. C. Tôi hoàn toàn đồng ý với bạn. D. Đó không phải là một ý kiến tốt.
  • 158. Question 17: A: "Have you ever done any volunteer work?" B: “_______.” A. I'm doing a part-time job to support my student life. B. You see, earning money is difficult these days. C. Sure. When I was a student, I helped in the hospital. D. I have been trying to work with all my heart. HD: “Bạn đã từng làm việc tình nguyện chưa?” A. Tôi đang làm một công việc bán thời gian để hỗ trợ cuộc sống sinh viên của mình. B. Bạn thấy đấy, ngày nay thì kiếm tiền rất khó. C. Chắc chắn rồi. Khi tôi còn là học sinh, tôi đã làm giúp trong bệnh viện. D. Tôi đang cố gắng làm việc với tất cả trái tim của mình.
  • 159. Question 18.A. approach B. panda C. income D. current Kiến thức về trọng âm A. approach /əˈproʊtʃ/: từ này có trọng âm rơi vào âm tiết thứ hai. Vì theo quy tắc, trọng âm không bao giờ rơi vào nguyên âm /ə/. B. panda /ˈpændə/: từ này có trọng âm rơi vào âm tiết thứ nhất. Vì theo quy tắc, trọng âm không bao giờ rơi vào nguyên âm /ə/. C. income /ˈɪnkʌm/: từ này có trọng âm rơi vào âm tiết thứ nhất. Vì theo quy tắc, danh từ có hai âm tiết thì trọng âm thường rơi vào âm tiết đầu. D. current /ˈkɝrənt/: từ này có trọng âm rơi vào âm tiết đầu. Vì theo quy tắc, trọng âm không bao giờ rơi vào nguyên âm /ə/.
  • 160. Question 19. A. confidence B. celebrate C. effective D. handicapped Kiến thức về trọng âm A. confidence /ˈkɑːnfədəns/: từ này có trọng âm rơi vào âm tiết đầu. Vì theo quy tắc, trọng âm không bao giờ rơi vào nguyên âm /ə/. B. celebrate /ˈseləbreɪt/: từ này có trọng âm rơi vào âm tiết đầu. Vì theo quy tắc, đuôi –ate làm trọng âm dịch chuyển ba âm tính từ cuối lên. C. effective /əˈfektɪv/: từ này có trọng âm rơi vào âm tiết thứ hai. Vì theo quy tắc, đuôi –tive làm trọng âm rơi vào âm tiết ngay trước nó. D. handicapped /ˈhændɪkæpt/: từ này có trọng âm rơi vào âm tiết thứ nhất. Vì theo quy tắc, nếu tất cả các âm mà ngắn hết thì trọng âm sẽ rơi vào âm tiết thứ nhất.